Sunteți pe pagina 1din 110

Anul XIII, Nr.

Iulie Decembrie 2011

RECREAII
MATEMATICE
REVIST DE MATEMATIC PENTRU ELEVI I PROFESORI

e i = 1

Asociaia Recreaii Matematice


IAI - 2011

ntr-o form concis, formula e = 1 leag


cele patru ramuri fundamentale ale matematicii: ARITMETICA - reprezentat de 1; GEOMETRIA
reprezentat de ; ALGEBRA reprezentat de i ; ANALIZA MATEMATIC reprezentat de e.

Semnificaia formulei de pe copert.

Membrii fondatori :
Temistocle BRSAN
Ctlin CALISTRU

Alexandru CRUU
Constantin COCEA

Adrian CORDUNEANU
Gheorghe IUREA

Membri onorifici :
Acad. Constantin CORDUNEANU
Acad. Radu MIRON

Redactor ef :

Cercet.pr. Dan TIBA


Prof.univ. Vasile OPROIU

Temistocle BRSAN

Redactori principali : Gabriel POPA coordonator al rubricilor de probleme


Gheorghe IUREA, Petru ASAFTEI, Maria RACU

Comitetul de redacie :
Dan BRNZEI
Alexandru CRUU
Constantin CHIRIL
Eugenia COHAL
Adrian CORDUNEANU
Mihai CRCIUN (Pacani)
Paraschiva GALIA

Paul GEORGESCU
Gheorghe ILIE
Gabriel MRANU
Alexandru NEGRESCU
Dan POPESCU (Suceava)
Florin POPOVICI (Braov)
Neculai ROMAN (Mirceti)

Ioan SCLEANU (Hrlu)


Ioan ERDEAN (Ortie)
Marian TETIVA (Brlad)
Lucian TUESCU (Craiova)
Adrian ZANOSCHI
Titu ZVONARU (Comneti)

Materialele vor fi trimise la una dintre adresele: t-birsan@yahoo.com , profgpopa@yahoo.co.uk


COPYRIGHT 2008, ASOCIAIA RECREAII MATEMATICE
Toate drepturile aparin Asociaiei Recreaii Matematice. Reproducerea integral sau parial a
textului sau a ilustraiilor din aceast revist este posibil numai cu acordul prealabil scris al acesteia.
TIPRIT LA BLUE SIM&Co IAI
Bd. Carol I, nr. 3-5
Tel. 0332 111021, 0721 571705; e-mail: simonaslf@yahoo.com
ISSN 1582 1765

Anul XIII, Nr. 2

Iulie Decembrie 2011

RECREAII
MATEMATICE
REVIST DE MATEMATIC PENTRU ELEVI I PROFESORI

e i = 1
Revist cu apariie semestrial

EDITURA RECREAII MATEMATICE

IAI - 2011

PETRU MINUT

(1936 - 2011)
In luna mai a acestui an a plecat dintre noi prof. univ. dr. Petru Minut,
distins cadru didactic universitar, colaborator apropiat si atasat al revistei Recreatii
Matematice.
Elev si student stralucit, dupa terminarea studiilor universitare a fost chemat la
catedra de algebra a Facult
atii de matematica si fizica a Universitatii din Iasi. A
urcat n timp si rand pe rand treptele ierarhiei universitare pana la cea mai nalt
a.
Ca elev, a fost un harnic rezolvitor de probleme propuse n Gazeta Matematic
a.
Liceul l-a f
acut n orasul Piatra Neamt, la matematica fiind ndrumat si format de
profesorul emerit Constantin Bors un didact des
av
arsit, care a d
aruit ntreaga
sa viata matematicii si nvat
am
antului matematic. De profesorul sau si amintea
ntotdeauna, atat n anii de studentie c
at si mai t
arziu, cu recunostinta si veneratie.
In anul 1998, anul stingerii profesorului C. Bors, va evoca personalitatea acestuia n
nr. 3 al Gazetei Matematice, seria A, ntr-un articol-studiu documentat si vibr
and de
emotie n acelasi timp.
In toamna anului 1954, n urma unui concurs de admitere, am devenit colegi la
sectia matematica a Facult
atii de matematica si fizica din Iasi, sectie care la sfarsitul
celor cinci ani de studii a dat opt absolventi; cinci dintre noi au facut cariere universitare, printre care si regretatul Petru Minut.
A fost apreciat si ndragit de prof. dr. doc. Ion Creang
a, seful catedrei de
algebra, care i-a fost si conducatorul stiintific al tezei de doctorat. Domeniile sale
de preferint
a au fost teoria numerelor, teoria probabilit
atilor si statistica matematic
a;
a predat cursuri de acest fel si a realizat lucrari n aceste domenii. Este coautor la
tratatul Introducere n teoria numerelor (1965), elaborat de un colectiv al catedrei
de algebra sub conducerea prof. dr. doc. I. Creanga, tratat deseori citat, actual si
acum.
Era interesat si preocupat de toate evenimentele importante: de la cele literare
sau artistice si p
ana la cele politico-sociale. A fost iubitor de cunoastere si frumos,
un democrat convins si cu dragoste de tara.
Desigur, cele mai importante realizari ale sale sunt legate de cariera universitara si
cercetarea n domeniul matematicii. Nu este locul aici de a detalia aceste aspecte. Ne
vom referi numai la contributia avut
a de Petru Minut n matematica elementar
a.
Pentru elevii si profesorii din nvat
am
antul preuniversitar, a publicat doua c
arti de
introducere n teoria numerelor: Teoria numerelor. Capitole introductive (1997) si
Numere prime. Numere prime speciale (2005, n colaborare cu Cristina Simirad ).
A considerat buna initiativa editarii revistei elementare Recreatii Matematice si a
publicat n paginile acesteia articole de istoria matematicii si teoria numerelor, precum si probleme de aritmetica si calculul probabilit
atilor. A publicat mai multe
portrete ale unor ilustri matematicieni ca: Pierre Fermat, Leonhard Euler, Gustav
Lejeune Dirichlet si N.A. Kolmogorov, c
at si ale unor matematicieni romani consacrati:
Grigore Moisil, Nicolae Teodorescu, Ion Creang
a. De asemenea, popularizeaza n
99

paginile revistei chestiuni de mare interes pentru elevi: ipoteza lui Goldbach, numere
prime din progresii aritmetice s. a.
Cu competent
a si pasiune a contribuit la ridicarea nivelului Recreatiilor Matematice. A fost membru al Asociatiei Recreatii Matematice, asociatie care tuteleaz
a
editarea acestei reviste.
Disparitia profesorului Petru Minut este o pierdere grea pentru toti cei care l-au
cunoscut. Pentru toti colaboratorii Recreatiilor Matematice ram
ane un exemplu de
modestie, constiinciozitate si grija pentru lucrul facut. Cititori si colaboratori, ne
unim ntr-un g
and de recunostinta pentru ceea ce ne-a lasat ca fapta si de p
astrare si
urmare a exemplului sau de d
aruire, abnegatie si omenie .

Prof.dr. Temistocle B
IRSAN

Lista articolelor publicate de Petru Minut n revista Recreatii Matematice:


90 ani de la nasterea prof. Ion Creang
a (nr. 1/2001, pp. 1-2);
Pierre Fermat - patru secole de la nasterea sa (nr. 2/2001, pp. 4-5);
Asupra ipotezei lui Goldbach (nr. 1/2002, pp. 5-6);
Numere prime din progresii aritmetice (nr. 1/2003, pp. 15-18);
A.N. Kolmogorov - 100 de ani de la nastere (nr. 2/2003, pp. 3-4);
200 de ani de la nasterea lui Dirichlet (nr. 1/2005, pp. 1-2);
100 de ani de la nasterea matematicianului Grigore C. Moisil (nr. 1/2006,
pp. 2-4);
300 de ani de la nasterea lui Leonhard Euler (nr. 1/2007, pp. 1-3);
100 de ani de la nasterea academicianului Nicolae Teodorescu (nr. 1/2009,
pp. 1-2);
O extensiune a sirului Fibonacci (nr. 1/2010, pp. 27-30) (n colaborare cu
Cristina Simirad).

100

Congresul al VII-lea
al matematicienilor rom
ani de pretutindeni
Orasul Brasov, cu frumoasa asezare n apropierea Carpatilor, celebre monumente,
precum Biserica Neagr
a, si valoroase traditii pe t
aram cultural-artistic, a fost gazda
primitoare a celui de-Al VII-lea Congres al matematicienilor rom
ani. Precizarea de pretutindeni este acum necesara, c
and - datorita evenimentelor din ultima jum
atate de secol - peste 400 de matematicieni romani, cu activitate creatoare
n acest domeniu, tr
aiesc n afara granitelor Rom
aniei. Ei se afla n toate continentele
Terrei, avand concentrare mai ridicat
a n Europa de Vest si America de Nord.
Urmasii spirituali ai lui Spiru Haret, Gheorghe T
iteica, Dimitrie Pompeiu, Alexandru Myller si ai elevilor acestora s-au rasp
andit n toata lumea unde matematica se
cultiva si este reprezentat
a n universitati sau institute de cercetare. Numai dintre
cei plecati de la Universitatea Al. I. Cuza ne putem mandri cu 7-8 actuali profesori
la c
ateva din marile universitati ale lumii (Paris - 2, Stanford - 2, Univ. California,
Berkeley - 1, Notre Dame Univ., Indiana - 1, Oxford - 1 ). Date similare sunt valabile
si pentru universitatile din Bucuresti, Cluj si Timisoara.
Congresele matematicienilor romani au debutat cu cel organizat la Cluj, n anul
1929, unde sufletul organizator a fost Petre Sergescu. Format la Bucuresti si ocupand
pozitii profesorale la Bucuresti si Cluj, si-a ncheiat cariera la Paris, n calitate de
presedinte al Academiei Internationale de Istoria Stiintei. La acest prim congres
au luat parte Gheorghe T
iteica si Dimitrie Pompeiu, atunci la apogeul carierei lor
academice. Acest nceput a fost urmat de congresele de la Turnu Severin si Bucuresti,
dupa care a urmat o ndelungata pauz
a datorita evenimentelor politice din anii 19301945.
Congresul al IV-lea a fost organizat cu mare fast si deosebit de distins
a participare
la Bucuresti, n anul 1956. Comitetul de organizare a fost alc
atuit din cunoscuti
matematicieni romani din acea perioada (S. Stoilow, M. Nicolescu, N. Teodorescu,
Grigore Moisil s.a.), printre participanti aflandu-se nume ca J. Hadamard, A. Denjoy,
W. Blaschke, L. Vietoris, B. Segre, I.N. Vekua, S. Eilenberg, E. Hille, M. Hukuhara,
K. Kuratowski, T. Wazewski.
Cu totul regretabil, succesul Congresului al IV-lea nu a ajutat la repetarea acestei
valoroase manifest
ari, Congresul al V-lea avand loc dupa 47 de ani, la Pitesti.
Am participat la congresele de la Pitesti (2003), Bucuresti (2007) si la ultimul,
care a avut loc, la Brasov (2011).
Congresul al VII-lea si-a desf
asurat lucrarile n perioada 29 iunie - 5 iulie, 2011,
fiind organizat si sponsorizat de c
atre un mare numar de organizatii si institutii,
101

incluz
and Presedintia Rom
aniei (nalt patronaj), Academia Rom
an
a, Institutul de
Matematic
a S. Stoilow al Academiei Rom
ane, Societatea de S
tiinte Matematice
din Rom
ania, Universitatea Transilvania din Brasov. Congresul a fost organizat si n
parteneriat cu European Mathematical Society (reprezentat
a de presedintele societatii,
prof. Marta Sanz - Sole, de la Universitatea din Barcelona).
Au participat peste 400 de matematicieni, n majoritate din Rom
ania sau plecati
din Rom
ania, comunic
andu-si rezultatele cercetarii lor n cele opt sectii ale congresului. Academicianul Solomon Marcus a prezentat raportul Aspects of the Mathematical
Research in Romania.
In cadrul mesei rotunde Sistemul educational din Romnia (moderator prof. Radu
Gologan, presedintele Societ
atii de S
tiinte Matematice), au participat mai multi vorbitori, iar D-na Marta Sanz-Sole a prezentat succint The European Mathematical
Society.
Am remarcat, cu satisfactie, prezenta multor matematicieni, incluz
and colegi
romani, de la universitatile: Paris, Roma, Tel Aviv, Princeton, Purdue, Sydney, San
Diego, Lund, Lausanne, Chisin
au, Columbus, Madrid, Lyon, Syracuse, N.Y., Florida
- Gainesville, Bologna, Grenoble, Teheran, Nancy, Leiden, K
oln, Lille, Yaroslavl,
Haifa, Hindu, Nice, Orlando (Florida), Toronto, Berlin, Skoplje, Bloomington (Indiana), Philadelphia.
Comitetul local de organizare, format din prof. E. Stoica, prorector al Universitatii Transilvania, si prof. E. P
alt
anea, decan al Facultatii de Matematica si Informatica, au asigurat, mpreuna cu matematicienii brasoveni, o reusita desf
asurare a
ntregii manifest
ari, inclusiv latura sociala (concertul de org
a de la Biserica Neagr
a,
spectacolul artistic cu concursul Operei din Brasov - balet, dansuri, vizite la diverse
obiective).
Desigur, participarea Academiei Rom
ane, prin sase membri ai Sectiei de S
tiinte
Matematice, si a Societ
atii de S
tiinte Matematice din Romania a contribuit la asigurarea prestigiului profesional al Congresului al VII-lea al matematicienilor
rom
ani. Sper
am, aceasta sa reprezinte o garantie pentru organizarea si desf
asurarea
congreselor viitoare.

Constantin CORDUNEANU
Universitatea din Texas (Arlington)
Academia Rom
ana

102

o sut
Profesorul ION CREANGA
a de ani
de la nasterea sa
S-a scurs deja un veac de c
and, la 1 ianuarie
1911, a vazut lumina zilei cel mai mic dintre cei
sapte copii ai familiei Elena si Luca Creang
a, care
n timp avea sa fie matematicianul Ion Creang
a1 ,
profesor la Universitatea Al. I. Cuza si rector al
acesteia.
Asupra familiei Creang
a, statornicita n Nordul
Moldovei (com. Adancata, jud. Dorohoi) se abate
o mare nenorocire: moare Luca Creanga pe c
and
micutul Ion avea doar un an. Familia, ramas
a n
grija mamei, se muta n scurt timp n Iasi. Intrega
viata a lui Ion Creang
a va fi legat
a de acest oras,
de care nu s-a desp
artit si pe care l-a iubit.
Aici, n Iasi, parcurge toate treptele de formare educativ
a si profesional
a: primare (Scoala
V. Alecsandri), secundare (Liceul National) si
superioare (Facultatea de S
tiinte a Universitatii
din Iasi, cu licent
a n matematici obtinuta n 1931). Cu nclinatii deosebite pentru matematica, a beneficiat de ndrumarile Silviei Creang
a , sora sa mai mare, care
este prima femeie din Rom
ania doctor n matematici (1925), cu o tez
a de geometrie
diferentiala sub conducerea lui Alexandru Myller. In facultatea ieseana Ion Creanga
a avut ca profesori nume de mare prestigiu: Alexandru si Vera Myller, Octav Mayer,
Simion Sanielevici, Grigore Moisil s.a.
Incepe cariera universitara la 1 iunie 1931, ca preparator la laboratorul de mecanic
a
condus de S. Sanielevici. Beneficiind de un concediu de studii pentru anul universitar
1938-1939, pleac
a la Roma pentru studii de doctorat; n iulie 1939 obtine doctoratul cu o tez
a de geometrie diferentiala sustinuta n fata unei comisii prezidate de
geometrul Enrico Bompiani.
In momentul reformei nvat
am
antului din 1948, Ion Creang
a ocupa o pozitie
de profesor la Institutul Politehnic din Iasi, dupa care este transferat la Universitatea
din Iasi ca sef al Catedrei de algebra de la Facultatea de matematica si fizica nou
nfiintata. Isi leaga destinul de catedra, facultatea si institutia n care activa.
Ca sef al Catedrei de algebra, pe care a condus-o p
ana n 1981, anul pension
arii,
profesorul Ion Creang
a a ncurajat directii de cercetare noi: cercet
ari operationale,
logic
a matematic
a, lingvistic
a matematic
a etc. Sub conducerea sa si-au sustinut doctoratul cu teme de algebra sau teoria numerelor urmatorii: C. Reischer, I. Enescu,
P. Minut, D. Spulber, Gh. Radu, M. S
tef
anescu, M. Gutan s.a.
Pe tot parcursul vietii active a avut functii de nalt
a raspundere: decan al Facultatii
de matematica si fizica (1949-1953), prorector (1953-1955) si rector al Universitatii din
1
In Recreatii Matematice , nr. 1/ 2001, cititorul poate g
asi evocarea 90 ani de la nasterea
profesorului Ion Creang
a , articol semnat de Petru Minut fost elev si colaborator al acestuia.

103

Iasi (1955-1972). Dintre multele realizari din perioada rectoratului sau selectam: sa finalizat extinderea cl
adirii universitatii, s-a nfiintat noua Gradina Botanic
a, au
reaparut Analele Stiintifice ale Universitatii, s-a organizat sarb
atorirea centenarului
Universitatii (1860-1960).
Dac
a n prima parte a activitatii sale stiintifice n cercetarile profesorului Ion
Creang
a sunt abordate subiecte de geometrie diferentiala (n contextul scolii de
geometrie iesene), n partea a doua aduce contributii n algebra si teoria numerelor.
In paralel, a publicat un numar mare de c
arti de referinta pentru domeniile respective.
Face parte din colectivul de profesori de la Seminarul Matematic Al. Myller care a
elaborat tratatul Curs de geometrie analitic
a (1951), lucrare distins
a cu premiul de
stat. A scris, n colaborare cu mai tineri colegi de catedr
a, c
arti utilizate si citate si
ast
azi: Algebra liniar
a (1962), Introducere n calculul tensorial (1963), Introducere n
teoria numerelor (1965) s.a.
Profesorul Ion Creang
a acorda o mare importanta actului didactic n toate componentele sale. Generatiile de studenti care l-au avut ca profesor si amintesc de lectiile
clare, accesibile, dar si riguroase ale acestuia. Odata intrat n sala de curs, profesorul
se d
aruia n ntregime auditoriului; expunerile sale nu erau niciodata tulburate de
griji personale sau provenite din functiile avute n cadrul facultatii sau rectoratului. Avea o figur
a nobil
a si o prezenta fizica agreabila, o voce armonioas
a si o privire
blanda, preda cu pasiune si cu bucuria de a transmite studentilor din cunostintele sale.
G
andirea studentilor era repede captat
a, acestia erau repede captivati de notiunile
expuse si teoriile explicate. Un z
ambet discret de multumire i lumina figura n momentele de bucurie spiritual
a care nsoteau ncheieri de demonstratii sau finaluri de
teorii matematice; era victoria spiritului n lupt
a cu umbrele necunoasterii.
Preocuparea pentru ndrumarea si formarea studentilor ca viitori profesori si specialisti nu se restrangea numai la orele de curs. Profesorul Ion Creang
a oferea
studentilor consultatii la materiile predate, ce aveau loc cu regularitate; obisnuia ca
la sfarsitul fiec
arui curs predat sa cheme la aceste consultatii si c
ativa studenti, prin
rotatie, pentru a discuta cu ei si a-i cunoaste. Asista la orele de seminar, la care
participa activ cu ntrebari lamuritoare si ncurajari. Era cunoscut ca profesorul care
la examinare ncuraja n asa masura studentii nc
at acestia prindeau aripi n expunerea subiectelor, iar reusita lor era o mare bucurie si pentru examinator. Ca si multi
altii, am simtit si eu c
aldura sufleteasca si ncurajarile sale la orele de curs si seminar
si, mai ales, la cele c
ateva examene sustinute la disciplinile pe care le-a predat.
Pentru ntrega activitate desf
asurat
a si aportul avut n prop
asirea nvatam
antului
si stiintei n tara noastra, profesorul Ion Creang
a a fost recompensat cu titlurile
de profesor emerit si om de stiint
a emerit.
S-a stins din viat
a la 21 mai 1987 si a fost nmorm
antat n cimitirul Eternitatea.
Anul trecut, 2010, la mplinirea a 150 de ani de la nfiintarea Universitatii Al. I.
Cuza, o delegatie a Senatului universitatii a depus o coroan
a de flori la mormantul
celui care a fost Ion Creang
a, omagiu adus omului si profesorului si semn de recunostint
a pentru ntreaga sa opera.

Prof.dr. Temistocle B
IRSAN
104

SPIRU HARET (1851-1912) - contributia stiintific


a
Spiru Haret a fost un savant multilateral: astronom, matematician, sociolog, organizator al nv
at
am
antului, promotor al culturii, patriot luminat.
Contemporanii l-au elogiat cu titluri precum:
mare patriot, p
arinte al t
ar
animii, omul scoalei,
marele ministru sau marele Haret.
S-a n
ascut la Iasi pe 15 februarie 1851 si
are naintasi n zona Panciu, unde o localitate se
numeste chiar Haretu. In unele materiale se d
a
ca loc de nastere Hanu Conachi, judetul Putna
(cu capitala la Focsani). A nceput scoala primara la Dorohoi, a continuat la Iasi si a terminat la Bucuresti. Tatal sau a fost notar si apoi
judec
ator si a trebuit sa se mute de mai multe ori,
n functie de cerintele serviciului. Urmeaza liceul
la Bucuresti, la Sf. Sava. Din anii liceului, au
ramas doua c
arti de matematici redactate de viitorul savant: o algebr
a si o trigonometrie. Dupa
cizel
ari minutioase, public
a, n 1873, cartea Elemente de trigonometrie, care a servit ca manual
de liceu p
ana n anul 1928. Dup
a terminarea liceului, n 1869, Haret s-a nscris la
Facultatea de Stiinte a Universitatii din Bucuresti, unde obtine licenta n matematici.
In 1874, obtine o bursa prin concurs si pleaca la Paris spre a se specializa n matematici. Mentionez c
a, din cele patru burse acordate n acel an de Titu Maiorescu, trei
erau pentru astronomie (Spiru Haret, Constantin Gogu, Nicolae Coculescu). In fapt,
mecanica cereasca era foarte la mod
a n acel moment! Se cuvine sa amintesc c
a n
acea perioada (1846) Le Verrier descoperise prin calcul (din varful condeiului cum le
placea francezilor sa se laude!) planeta Neptun, pe baza perturb
arilor orbitei planetei
Uranus. La Paris, n 1875, Haret si trece din nou examenul de licenta n matematica
si, n 1876, obtine si licenta n fizica. In ianuarie 1878 obtine titlul de doctor la
Sorbona, devenind primul roman doctor n matematica la Paris. Teza sa purta titlul
Asupra invariabilit
atii axelor mari ale orbitelor planetare si a fost elaborata
sub conducerea lui V.A. Puiseux. Se aborda o problem
a deosebit de interesant
a legat
a
de stabilitatea sistemului solar. Dup
a cum se stie, miscarile planetelor n sistemul
nostru solar sunt influentate, n principal, de masa Soarelui care face ca miscarea
sa se dezvolte pe cunoscutele elipse. Miscarile sunt descrise destul de bine de legile
lui Kepler. Printre altele, rezulta c
a axele mari ale elipselor ram
an neschimbate n
timp. Este totusi posibil ca misc
arile sa fie influentate si de masele fiecareia dintre
planetele existente n sistemul solar si atunci formele elipselor (n principal, valorile
axelor acestor elipse, dar si asezarea lor n spatiu) s-ar putea modifica. Matematicienii
Pierre Laplace (1773) si, apoi, Louis Lagrange (1776) luasera deja n consideratie,
cu mult naintea lui Haret, influenta maselor unor planete, care intervin n ecuatii
prin dezvoltarea p
ana la puterea nt
ai a fortelor perturbatoare. Rezultatul a fost
c
a axele mari ale elipselor pe care se misca panetele sunt invariabile. Chiar daca
105

se realizeaz
a dezvoltarea p
ana la puterea a doua pentru fortele perturbatoare, teoria
ram
ane valabila, asa cum aratase Simeon Denis Poisson n 1808.
Rezultatul obtinut de Haret n teza sa spune c
a, daca se iau n studiu dezvolt
arile
p
ana la puterea a treia ale fortelor perturbatoare, apar niste variatii seculare care
contrazic rezultatele obtinute p
ana atunci, c
and se luau n consideratie doar dezvolt
arile p
ana la puterea cel mult doi. La sustinerea tezei lui Haret a participat si
Henri Poincare, proasp
at absolvent, care a apreciat cu mare uimire rezultatele din
tez
a. Cercet
arile lui Haret au fost citate de c
atre Felix Tisserand n al sau Traite de
mecanique celeste care recomanda extinderea metodei la cazul planetei Saturn pentru verificarea rezultatelor lui Le Verrier relativ la perturbatiile acestei planete. Mai
t
arziu, 1955, cercetarile au fost reluate de J. Meffroy prin utilizarea unor concepte si
tehnici noi de calcul.
Problema stabilitatii axelor mari ale traiectoriilor planetelor a constituit unul din
testele relativitatii generale, alaturi de curbarea semnalului luminos n vecinatatea
unui corp masiv si de deplasarea spre rosu n spectrul unui semnal luminos provenit de
la o stea masiv
a. Concret, este vorba de cunoscuta problem
a a avansului periheliului
unei planete. Se stia deja (Le Verrier, 1859) c
a elipsele pe care se misca planetele
descriu ele nsele o miscare de avansare. Acest fapt se pune n evidenta cu ajutorul
punctului cel mai apropiat de Soare, periheliul, care nu e fix n spatiu (asa cum ar
trebui!), ci are o tendint
a de avansare c
atre sensul din care vine planeta. Acest lucru
este perceput cel mai bine n cazul planetei Mercur care are o perioada destul de mica
de revolutie (88 zile). Asa c
a diversele avansuri ce apar la fiecare rotatie se sumeaza
cel mai adesea. In 100 de ani, periheliul lui Mercur a avansat cu circa 574 de secunde.
Influenta maselor celorlalte planete a putut justifica doar 532 secunde din acest avans,
restul de 42,9 secunde ram
anand ca o enigm
a pentru mecanica cereasc
a. Pentru o
perioada, s-a avansat ideea existentei unei planete nedescoperite, care a fost chiar
si botezat
a Vulcan si care ar fi fost responsabila de avansul ce nu putea fi explicat.
In urma elabor
arii teoriei relativitatii generale de c
atre A. Einstein, n 1915, si a
g
asirii solutiei Schwarzschild, n 1916 (sub forma unei metrici pseudoriemanniene n
varietatea spatiu-timp), s-a putut explica cealalt
a parte din avans pornind de la ideea
c
a traiectoriile planetelor sunt geodezice n metrica Schwarzschild.
Mai t
arziu, unele rezultate au fost extinse si aplicate pentru calculul orbitelor
satelitilor artificiali ai P
am
antului. S-a constatat c
a, chiar forma P
am
antului influenteaz
a traiectoriile satelitilor sai artificiali. In leg
atura cu aceasta mentionez contributia
lui Eugen Grebenikov, membru n echipa sovietic
a ce a efectuat calculele pentru traiectoriile satelitilor n anii 50 si 60. Au existat dificultati de calcul si de conceptie
nt
ampinate n timpul desf
asur
arii operatiilor de lansare. Asa cum povestea Grebenikov
ntr-o conferint
a tinuta n 2010 n aula Academiei din Iasi, una din dificultati provine
din faptul c
a P
am
antul este turtit la poli. Diferenta ntre raza de la ecuator si raza
de la poli este de circa 18 km. Grebenikov mentiona c
a lucrurile ar fi fost mai simple dac
a P
am
antul ar fi fost alungit (sub forma unui pepene si nu ca un dovleac!).
Binenteles, multe lucruri ar fi fost mai simple daca s-ar fi dispus de tehnica de calcul
existent
a n momentul actual. In conferinta sa, Grebenikov se plangea de acest lucru
si vorbea de embargoul pus de lumea occidental
a n privinta tehnicii de calcul.
Mentionez c
a numele lui Spiru Haret a fost dat unui crater de pe partea nev
azut
a
106

a Lunii, iar numele lui Grebenikov a fost dat unui asteroid din puzderia de asteroizi
ce roiesc undeva ntre Jupiter si Saturn. Exista vreo 13 nume de romani purtate de
asteroizi, comete sau de cratere de pe diverse planete: Br
ancusi, Eminescu, Enescu,
Elena V
ac
arescu, Victor Daimaca, Nicolae Sanduleak, Constantin P
arvulescu, Hermann Oberth, E. Grebenikov, Nicolae Donici, Mirel Brlan, Jean Dragesco etc.
Desi ocupat cu activitati de interes obstesc (a fost ministru al educatiei de trei ori,
n 1897-1899, n 1901-1904 si n 1907-1910, realizand o reforma substantiala a nvatamantului, prin care s-a constituit sistemul de educatie modern), Haret a mai publicat
c
ateva articole de astronomie: despre acceleratia secular
a a Lunii (1880) si despre
pata rosie de pe Jupiter (1912) si meteorul luminos de la 29 noiembrie 1911 (1912).
Una din lucrarile sale cele mai interesante este monografia Mecanica social
a
(1910, Paris-Bucuresti), n care ncearc
a s
a aplice principiile mecanicii la viata social
a
- fiind astfel un precursor al utiliz
arii metodelor matematice n studiul fenomenelor
sociale. Rezultatele din aceasta monografie au starnit controverse aprige, dovedind c
a,
prin unele concepte si prin unele metode de cercetare, Spiru Haret era cu mult naintea
timpului sau. Prin aceasta carte, Haret a ncercat sa puna la punct metoda modelarii
si conceptia prin care s-ar putea descrie si evalua fortele ce guverneaz
a n mod obiectiv
fenomenele sociale si economice. A extins la viata societatii principiul minimei actiuni
si a folosit probabilitatile si, inevitabil, statistica matematica n analiza sociologica.
A elaborat si unele lucrari cu aplicatii practice: Despre m
asura capacitatii butilor
(1878), Consideratii relative la studiul experimental al misc
arii apei n canale descoperite si la constitutia intern
a a fluidelor (1882), Teorema ariilor n miscarea sistemelor materiale (1894), Not
a asupra poporatiunii Rom
aniei (1903) etc.
In 1892 a devenit membru al Academiei (membru corespondent devenise nc
a
din 1879). Imbin
and munca de cercetare stiintific
a cu cea de organizare si de popularizare a stiintei, Haret a lasat n urma sa si numeroase manuale de scoal
a.
Mai mention
am si eforturile intense depuse de Spiru Haret pentru nfiintarea
Observatorului Astronomic din Bucuresti, realizata n 1908.
Bibliografie
1. G. S
t. Andonie Istoria Matematicii n Rom
ania, vol. I, Ed. Stiintific
a, Bucuresti,
1965; Spiru Haret, 213-223.
2. M. Stavinschi Un nume rom
anesc pe harta Lunii: Spiru Haret (15 februarie 1851
- 17 decembrie 1912), Astronomical Institute of the Romanian Academy.
3. M. Stavinski, V. Mioc Astronomical Researches in Poincare s and Romanian
Works, Astronomical Institute of the Romanian Academy, 2004. (http://syrte.obspm.
fr/ journees2004/PDF/Stavinschi.pdf)
4. Spiru Haret Biography at Mac Tutor History of Mathematics archive, St. Andrews
University.
5. Spiru Haret Wikipedia, the free encyclopedia.
6. L. Modan Spiru Haret - reper al spiritualit
atii rom
anesti, Gazeta Matematica (A),
anul XIX (XCVII), 2001, 113-118.

Prof.dr. Vasile OPROIU


107

Alte propriet
ati caracteristice triunghiului
echilateral
Titu ZVONARU1 , Neculai STANCIU

Abstract. This article is a continuation of the paper [3]. We obtain several other characterizations of the equilateral triangle.
Keywords: orthocenter, centroid, incenter, Nagel point, Gergonne point, Spiecker point.
MSC 2000: 51M04.

Scopul lucrarii de fat


a este g
asirea unor propriet
ati caracteristice triunghiului echilateral, pe linia celor stabilite n [3]. Vom folosi aceleasi notatii: astfel, punctului H
(ortocentru) i corespunde AH BH CH (triunghiul pedal determinat de punctul H)
avand punctele importante, cu semnificatiile evidente, OH , GH , HH , IH etc. Ca si n
[3], avem n vedere numai triunghiuri ascutitunghice. Vom rezolva c
ateva dintre problemele de tipul: dac
a dou
a puncte importante (luate din diferite triunghiuri pedale)
coincid, atunci ABC este echilateral. O atentie deosebit
a va fi acordata punctului
lui Spiecker al triunghiului ABC centrul cercului nscris n triunghiul median, adica,
n notatiile convenite, punctul IG .
Procedeul utilizat decurge din observatia: punctele X si Y coincid daca si numai
dac
a picioarele cevienelor duse prin aceste puncte coincid (este suficient
a coincidenta
picioarelor numai pe doua laturi ale triunghiului). Vom pune acum la punct instrumentul de lucru; mai nt
ai, reproducem un rezultat din [4]:

Lem
a. In triunghiul ABC consider
am ceviana AD, cu D (BC). Dac
a o secant
a
intersecteaz
a laturile AB, AC si ceviana AD n punctele M, N , respectiv P , atunci
sunt adev
arate urm
atoarele dou
a relatii:
AM AN

M
B NC
. (R2 )
(R1 )
AN
AM
+ DC
BD
MB
NC
Problem
a. Fie ABC un triunghi si punctele M, N, P pe laturile BC, CA respectiv
BM
CN
AP
triunghiul M N P consider
AB astfel nc
at
= k1 ,
= k2 ,
= k3 . In
am
M
C
N
A
P
B

cevienele M M , N N , P P concurente n punctul Q (M (N P ), N (P M ), P


M N
N P
P M
(M N )), cu
=
p
,
=
p
,
= p3 . Dac
a {A } = AQ BC, s
a se
1
2
M
N
M P
N
P

BA
calculeze raportul .
AC
Solutie. Conform teoremei lui Ceva avem p1 p2 p3 = 1. Not
am {S} = AA P N ,

SP
BA
{T } = AA M N si
= t, = x (dac
a AQ intersecteaz
a [P M ], calculele sunt
SN
AC

A B BM
k1 BC x + 1
x k1
AM
=x

=
.
similare). Rezult
a usor c
a =
AC
A C
k1 + 1
BC
k1 + 1
AM BD AC P N

= 1;
AB DC AN P M

AP
=
PD

1 Com
anesti,
2 Profesor,

BC

e-mail: tzvonaru@yahoo.com
S
coala general
a George Emil Palade, Buz
au, e-mail: stanciuneculai@yahoo.com

108

AP A B AC
Conform relatiei (R1 ), avem

AB A C AN
k3
1
SN
=1
x (k2 + 1) = 1, adica
SP
k3 + 1
t
(1)

k3 (k2 + 1)
t
=
.
x
k3 + 1

S
P

M N
P
T

Cu relatia (R2 ), deducem c


a
(2)
M
SN T N
1 TN B
P
M

QN
SP T M
t TM
=
=
,
p
SN
T
N
1
1
TN
QN
2
PN
+ N M
+

SP
TM
p2 + 1 t
p2 + 1 T M

NM
NP
1
QN
=
= p1 + p1 p2 =
+
= p1 +

QN
MP
P M
p3
TN
1
p2
1 TN
+

p1 (p2 + 1) si atunci (2) se scrie succesiv: p1 (p2 + 1)


=
p2 + 1 t
TM
t TM
p1 p2
1 TN
N
=
+ p1 TT M
, deci
t
t TM
iar cu relatia lui Van Aubel avem

1 p1 t
TM
.
=
TN
p1 p2

(3)

Cu teorema lui Menelaus n M N C si transversala A T A obtinem


A C
= 1 sau
A M

T M AN

T N NC

1 p1 t
1
k1 + 1

= 1.

p1 p2
k2 + 1 x k1

(4)

Inlocuind n (4) pe t dat de (1), se obtine o ecuatie de gradul I cu necunoscuta x,


avand solutia
()

x=

k3 + 1 k1 p1 p2 (k2 + 1) + (k1 + 1)

.
k2 + 1 p1 p2 (k3 + 1) + k3 p1 (k1 + 1)

Dac
a X este un punct important n AY BY CY , vom nota cu xXY raportul n
care dreapta AX mparte latura BC a triunghiului ABC. In cazul punctului lui
a
b
c
Spiecker, IG , avem k1 = k2 = k3 = 1 si p1 = , p2 = , p3 = ; deci, utilizand relatia
b
c
a
a+b
(), obtinem xIG =
.
a+c
Propozitia 1. Dac
a IG GI (i.e. punctul lui Spiecker coincide cu centrul de
greutate al triunghiului pedal al punctului I), atunci ABC este echilateral.
109

c
a
b
, k2 = , k3 =
si p1 =
b
c
a
c(a + b)(a + b + 2c)
. Dac
a IG GI ,
p2 = p3 = 1; cu relatia (), obtinem xGI =
b(a + c)(a + 2b + c)
c(a + b)(a + b + 2c)
a+b
=
ab + 2b2 + bc =
n mod necesar xIG = xGI , i.e.
a+c
b(a + c)(a + 2b + c)
ac + bc + 2c2 (b c)(a + 2b + 2c) = 0, deci b = c. Analog deducem a = b, deci
a = b = c si ABC este echilateral.
Demonstratie. Relativ la punctul GI avem k1 =

Propozitia 2. Dac
a IG GH (i.e. punctul lui Spiecker coincide cu centrul de
greutate al triunghiului ortic al ABC), atunci ABC este echilateral.
a cos C
b cos A
c cos B
, k2 =
, k3 =
Demonstratie. Relativ la GH avem k1 =
b cos C
c cos A
a cos A
si p1 = p2 = p3 = 1; cu relatia (), dupa calcule de rutina, obtinem xGH =
c2 (a cos A + b cos B)
a+b
c2 (a cos A + b cos B)
. Avem: xIG = xGH
= 2
[a2 (b2
2
b (a cos A + c cos C)
a+c
b (a cos A + c cos C)
bc 2
c2 ) + a(b3 c3 )] cos A + bc(b2 c2 ) +
[a (b c) + b3 c3 + bc(b c)] = 0 (s-au
2a
nlocuit cos B si cos C prin expresiile date de teorema cosinusului) si rezulta b = c.
Analog deducem a = b, deci ABC este echilateral.
Propozitia 3. Fie punctul lui Gergonne al ABC. Dac
a IG G (i.e.
punctul lui Spiecker coincide cu centrul de greutate al triunghiului pedal al punctului
), atunci ABC este echilateral.
pb
pc
pa
Demonstratie. Pentru avem k1 =
, k2 =
, k3 =
, p1 = p2 =
pc
pa
pb
c a(p a) + b(p b)
p3 = 1 si obtinem xG =
. Avem succesiv:
b a(p a) + c(p c)
a+b
2abc + ac2 + bc2 a2 c b2 c

=
a+c
2abc + ab2 + b2 c a2 b bc2
(b c)(4abc a3 + ab2 + abc + ac2 + b2 c + bc2 ) = 0.

xI xG

Deoarece 2abc + ab2 + ac2 = a(b + c)2 > a3 (din inegalitatea triunghiului), rezulta
b = c. Analog deducem a = b si concluzia dorit
a.
Propozitia 4. Fie N punctul lui Nagel. Dac
a IG GN (i.e. punctul lui Spiecker
coincide cu centrul de greutate al triunghiului pedal al punctului N ), atunci ABC
este echilateral.
pa
pb
pc
, k2 =
, k3 =
,
Demonstratie. In cazul punctului N , k1 =
pb
pc
pa
c(p c)(a + b)
p1 = p2 = p3 = 1 si, ca urmare, xGN =
. Atunci xIG xGN
b(p b)(a + c)
c(p c)(a + b)
a+b
=
b(p b) = c(p c) (b c)(b + c a) = 0, de unde b = c.
a+c
b(p b)(a + c)
Analog deducem c
a a = b si, deci, concluzia dorit
a.
110

Propozitia 5. Dac
a HG GH (i.e. ortocentrul triunghiului median coincide cu
centrul de greutate al triunghiului ortic), atunci ABC este echilateral.
Demonstratie. Valoarea raportului xGH a fost stabilita n Propozitia 2. Pentru
a cos C
b cos A
c cos B
, p2 =
, p3 =
si
punctul HG avem k1 = k2 = k3 = 1, p1 =
b cos C
c cos A
a cos B
c cos C
obtinem xGH =
. Atunci,
b cos B
xHG = xGH

c(a cos A + b cos B)


cos C
=

cos B
b(a cos A + c cos C)

(ab cos C ac cos B) cos A + bc(cos2 C cos2 B) = 0

(a2 + b2 c2 a2 + b2 ) cos A + 2bc(cos C cos B)(cos C + cos B) = 0




2(b c ) cos A + 2bc

a2 + c2 b 2
a2 + b 2 c2

2ab
2ac

(cos C + cos B) = 0

1
2(b2 c2 )cosA + (b c)(b + c a)(b + c + a)(cos C + cos B) = 0,
a
deci b = c. Analog deducem a = b si rezulta c
a ABC este echilateral.
Observatii. 1) Formula (*) este utila n situatiile n care laturile triunghiului
pedal se calculeaza relativ usor sau nu este nevoie de ele (p1 = p2 = p3 = 1). Dac
a
p1 , p2 , p3 depind de laturile triunghiului si acestea sunt greu de calculat (cum ar fi,
de exemplu, cazul AO BO CO sau al AI BI CI ), calea descris
a mai sus nu poate fi
urmat
a.
2) Cu formula () se demonstreaza usor c
a ABC este echilateral n urmatoarele
situatii:
a) Punctul Nagel al triunghiului median este centrul cercului nscris n triunghiul
dat ([1]);
b) Punctul lui Lemoine al triunghiului de contact este punctul lui Gergonne al
triunghiului dat ([2]);
c) Izotomicul ortocentrului triunghiului median este punctul lui Lemoine al triunghiului dat ([3]).
Bibliografie
1. N. Altshiller-Court College Geometry, Barnes & Noble Books, New-York, 1968.
2. G. Mihalescu Geometria elementelor remarcabile, Editura Tehnica, Bucuresti,
1957.
3. F. Toma Propriet
ati caracteristice triunghiului echilateral, Recreatii Matematice,
1/2011, 17-19.
4. T. Zvonaru, B. Ionit
a Rapoarte determinate de o cevian
a si o secant
a ntr-un
triunghi, Recreatii Matematice, 1/2005, 15-17.

111

Inegalit
ati geometrice n poligoane convexe,
de tip Bergstr
om-Mitrinovi
c

Dumitru M. BATINET
U-GIURGIU1 , Neculai STANCIU2
Abstract. Some Mitrinovi
c type inequalities for general convex polygons are presented. The
main tool in the proofs is Bergstr
om inequality.
Keywords: Mitrinovi
c type inequalities, Bergstr
om inequality, convex polygon.
MSC 2000: 51Mxx, 26D15.

Inegalitatea lui Bergstr


om are urmatorul enunt: Dac
a n N {1}, xk R,
yk R+ , k = 1, n, Xn =
(B)

n
X

n
X

xk , Yn =

k=1

yk , atunci:

k=1
n
X
x2k
k=1

yk

Xn2
,
Yn

cu egalitate dac
a si numai dac
a exist
a t R astfel nc
at xk = tyk , k = 1, n.
Inegalitatea lui D.S. Mitrinovi
c are urmatorul enunt:
In orice triunghi de perimetru
2p, circumscris unui cerc C(I; r), are loc inegalitatea:

(M )
p 3r 3,
cu egalitatea dac
a si numai dac
a triunghiul este echilateral.
Scopul acestui articol este de a stabili unele inegalitati geometrice (altele decat
cele din [1]) de tipul (M) n poligoane convexe, folosind inegalitatea (B).
Pentru orice poligon convex A1 A2 . . . An , n 3, vom nota cu S aria poligonului,
cu 2p perimetrul poligonului, cu ak lungimea laturii [Ak Ak+1 ], k = 1, n, An+1 A1 ,
iar pentru orice punct M interior poligonului notam Tk = prAk Ak+1 M , dk = M Tk ,
uk = (Ak M Tk ), vk = (Tk M Ak+1 ), Sk = aria[Ak M Ak+1 ], k = 1, n.
Lem
a. Fie A, B, A 6= B dou
a puncte n plan si M
/ AB, cu T = prAB M [AB];
AB

atunci
T ), v = (T
M B) (n radiani), iar d este
= tg u + tg v, unde u = (AM
d
distanta de la M la dreapta AB.
Demonstratie. Avem urmatoarele situatii:
AT
BT
AB
i) T (AB). Atunci tg u =
si tg v =
, deci tg u + tg v =
.
MT
MT
MT
AB
BT
, deci tg u + tg v =
.
ii) T A (analog T B). Gasim tg u = 0 si tg v =
MT
MT
Teorema 1. Dac
a A1 A2 . . . An , n 3, este un poligon convex si M este un punct
interior lui astfel nc
at prAk Ak+1 M = Tk [Ak Ak+1 ], k = 1, n, An+1 A1 , atunci
(1)

n
X
ak
k=1

1 Profesor,
2 Profesor,

dk

2n tg

.
n

Colegiul National Matei Basarab, Bucuresti


S
coala General
a George Emil Palade, Buz
au

112

Demonstratie. Conform Lemei avem


n a
P
k

dk

n
P

ak
= tg uk + tg vk , k = 1, n, de unde
dk

(tg uk + tg vk ).

h 

R, f (x) = tg x este convex
a pe 0,
, rezulta
2
2
n a
n
P
P
k
(tg uk + tg vk )
c
a putem aplica inegalitatea lui Jensen si obtinem
=
d
k
k=1
k=1


n
n a
n
P
P
1 P
2
k
2n tg
(uk + vk ) . Din
(uk + vk ) = 2, deducem c
a
2n tg
=
2n k=1
d
2n
k=1
k=1 k

2n tg , ceea ce era de demonstrat.


n
Observatia 1.1. Dac
a poligonul A1 A2 . . . An este circumscris unui cerc C(I; r)
si M I, rezulta c
a dk = r, k = 1, n, iar (1) devine
k=1

k=1

Deoarece functia f : 0,

2p

1X
ak =
2n tg
p nr tg .
r
r
n
n
n

(1 )

k=1

Inegalitatea (1 ) este o generalizare a inegalitatii (M).


Observatia 1.2. In cazul n care poligonul este un triunghi ABC, relatia (1)
devine

b
c

a
+
+
6 tg = 6 3.
da
db
dc
3

(1 )

Pentru M I obtinem inegalitatea (M).


Teorema 2. Dac
a A1 A2 . . . An , n 3, este un poligon convex si M este un punct
interior poligonului, atunci
n
X
ak

(2)

k=1
n a
P
k

Demonstratie. Avem

si obtinem

n a
P
k
k=1

dk

n
P

k=1

dk

dk

n
P
k=1

2p2
.
S

n
P
a2k
a2k
=
. Aplic
am inegalitatea (B)
ak dk
k=1 2Sk

ak )

k=1
n
P

=
Sk

4p2
2p2
=
, ceea ce era de demonstrat.
2S
S

k=1

Observatia 2.1. Dac


a A1 A2 . . . An , n 3, este circumscris cercului C(I; r),
n a
P
2p2
2p
k

=
; apoi din (1 ) deducem c
a
atunci S = pr, iar (2) devine
pr
r
k=1 dk

(2 )

n
X
ak
k=1

dk

2n tg

,
n

M IntA1 A2 . . . An .
113

Prin urmare, M IntABC are loc inegalitatea


a
b
c
2p

+
+

6 tg .
da
db
dc
r
3

(2 )

Teorema 3. Dac
a A1 A2 . . . An , n 3, este un poligon convex si M este un punct
interior lui astfel nc
at prAk Ak+1 M = Tk [Ak Ak+1 ], k = 1, n, An+1 A1 , atunci:
n
X
ak

(3)

d2k

k=1
n a
P
k

Demonstratie. Avem
n
P

ak
2
d
k=1 k

n
P

k=1
n
P

k=1
ak 2
dk )

=
ak

1
2p

n a
P
k

k=1

ak

ak
dk

, unde aplic
am (B) si obtinem

2

, de unde, folosind (1), deducem (3).

dk

k=1

n 1
P

d2k

2n2 2
tg .
p
n

k=1

Observatia 3.1. Dac


a poligonul A1 A2 . . . An , n 3, este circumscris cercului
C(I; r) si M I, atunci (3) devine:

p2 n2 r2 tg2 .
n
n

2p2 2n2 r2 tg2

(3 )

Teorema 4. Dac
a A1 A2 . . . An , n 3, este un poligon convex si M este un punct
interior lui astfel nc
at prAk Ak+1 M = Tk [Ak Ak+1 ], k = 1, n, An+1 A1 , atunci:
n
X

(4)

n
X
1

a2k

k=1

Demonstratie. Avem


si obtinem

n 1
P
k=1

d2k

k=1

n 1
P
k=1

n a
P
k

d2k

k=1

4n2 tg2

d2k

n 1
P

2

k=1 dk
n
P
a2k
k=1

n
P

k=1

a2k

ak
dk



a2k

.
n

, unde aplic
am inegalitatea (B)

n 1
P
k=1

d2k

n a
P
k
k=1

dk

2

, apoi, tinand

cont de (1), deducem concluzia.


Observatia 4.1. Dac
a poligonul convex A1 A2 . . . An , n 3, este circumscris
cercului C(I; r), iar M I, atunci dk = r, k = 1, n, si relatia (4) devine
(4 )

n
r2

n
X
k=1

a2k

4n2 tg2

n
X

a2k 4nr2 tg2 .

n
n
k=1

Teorema 5. Dac
a un poliedru convex are n(n 4) fete poligoane convexe de arii
Sk (k = 1, n), iar M este un punct interior poliedrului cu distanta dk la fata de arie
114

Sk si V, S sunt volumul si respectiv aria total


a ale poliedrului, atunci:
n
X
Sk

(5)

k=1
n S
P
k

dk

S2
.
3V

n S2
P
Sk2
k
=
, unde Vk este volumul
d
d
S
3V
k
k
k
k
k=1
k=1
k=1
piramidei de varf M si baz
a poligonul fetei de arie Sk . Aplic
and inegalitatea (B),

Demonstratie. Avem


deducem c
a

n S
P
k
k=1

dk

n
P

n
P

2

Sk

k=1
n
P

S2
, ceea ce era de demonstrat.
3V

=
Vk

k=1

Observatia 5.1. Dac


a poliedrul este circumscris unei sfere S(I; r) de centru I si
raz
a r atunci relatia (5) devine:
n
X
Sk

(5 )

k=1

dk

S2
S2
S
=
= .
3V
Sr
r

Dac
a M I, atunci dk = r, k = 1, n, si relatia (5 ) devine o egalitate.
conditiile teoremei 5, are loc inegalitatea:
Teorema 6. In
n
X

(6)

Sk2

k=1

Demonstratie. Avem


deducem

n
P

1
2
k=1 dk

n
P

n 1
P

k=1
2
Sk

k=1 dk
n
P
Sk2
k=1

d2k

k=1

n
X
1

d2k

n
P
1
k=1

Sk2

Sk
dk

S4
.
9V 2
, unde aplic
am inegalitatea (B) si

. Dac
a tinem seama de

n S
P
k
k=1

si aplic
am din nou (B), atunci obtinem

n
P
k=1

ceea ce era de demonstrat.



Sk2

dk

n 1
P
k=1

d2k

n
P
k=1

n S2
P
Sk2
k
=
dk Sk
k=1 3Vk

n S
P
k
k=1

dk

2

S4
,
9V 2

Observatia 6.1. Dac


a poliedrul este circumscris unei sfere S(I; r) si M I,
n
n
P
n P
S4
r2 S 4
2
atunci relatia (6) devine 2
Sk2
S

, apoi, tinand seama c


a
k
r k=1
9V 2
9nV 2
k=1
rS = 3V , obtinem

(6 )

n
X
k=1

Sk2

S2
r2 S 4
=
.
nr2 S 2
n

Bibliografie
1. M. Dinc
a, M. Bencze Trip in world of geometrical inequalities (2), Octogon
Mathematical Magazine, vol 11, No. 1, april 2003, 45-76.
115

In leg
atur
a cu o problem
a de concurs
Dan S
tefan MARINESCU1 , Ioan S
ERDEAN

Abstract. Proposition 2, the main result of the paper, extends some problems which have
appeared in journals or contests of elementary mathematics.
Keywords: derivatives, convex functions.
MSC 2000: 26A51, 26A42.

1. Introducere. La Olimpiada National


a de Matematic
a din anul 2005 autorii
acestei note au propus urmatoarea problem
a:
P1. Fie f : [0, 1] R o functie care verific
a conditiile |f (x) f (y)| |x y|,
Z

f (x)dx = 0.

()x, y [0, 1] si

a) S
a se arate c
a|
Z

1
2

b) Dac
a

x
0

f (x)dx =

f (t)dt|

1
x(1 x), ()x [0, 1],
2

1
, s
a se determine f .
8

Enuntul acestei probleme a aparut n Lista scurt


a a Olimpiadei Nationale de
Matematic
a 2005 si n R.M.C. 2005. In anul 2007 la cunoscutul Concurs studentesc
de matematic
a Putnam din U.S.A. s-a propus problema
Z

P2. Fie f : [0, 1] R o functie derivabil


a cu derivata continu
a si
Z

f (x)dx = 0.
0

1
max |f (x)|.
8 0x1
0
In mod cert P2 se deduce din P1. Intr-adev
ar, fie M = max |f (x)|. Dac
a M = 0,

Atunci, pentru orice [0, 1] are loc inegalitatea |


Z

f (x)dx|
0x1

f (x)dx = 0 deducem c
a f este functia nula,

atunci f este constant


a pe [0, 1] si cum
0

iar inegalitatea este evident


a n acest caz. Dac
a M 6= 0, atunci functia g : [0, 1] R,
Z 1
1
1
1
g(x) =
g(x)dx| (1) ,
f (x) verific
a ipotezele problemei P1, de unde |
M
2
8
0
Z
1

f (x)dx|
() [0, 1] si, n concluzie, |
max |f (x)|.
8 0x1
0
Tot legat de problema P1, sa remarc
am faptul c
a n numarul din martie 2011 al
revistei americane The College Mathematics Journal apare sub semnatura lui Duong
Viet Thong urmatoarea problem
a:
Z

P3. Fie f : [a, b] R o functie derivabil


a cu derivata continu
a si
Z

Demonstrati c
a|
1 Profesor
2 Profesor,

f (t)dt| M

f (x)dx = 0.
a

(x a)(b x)
, ()x [a, b], unde M = max |f (x)|.
2
x[a,b]

dr., Colegiul National Iancu de Hunedoara, Hunedoara


Colegiul National Aurel Vlaicu, Or
astie

116

2. Rezultatul principal. Pentru scopul propus avem nevoie de c


ateva rezultate
cunoscute relativ la functiile convexe. O functie f : I R, I R interval nedegenerat, este convex
a dac
a f (ta+(1t)b) tf (a)+(1t)f (b), ()a, b I, ()t [0, 1].

Propozitia 1. (i) f : I R este convex


a dac
a si numai dac
a ()m, n R astfel
nc
at functia g : I R definit
a prin g(x) = f (x) + mx + n, ()x I, este convex
a.
(ii) Dac
a f este convex
a, a, b I, a < b, si f (a) = f (b) = 0, atunci f (x) 0,
()x [a, b].
(iii) Dac
a f convex
a si exist
a a, b I, a 6= b si t0 (0, 1) astfel nc
at f (t0 a + (1
t0 )b) = t0 f (a) + (1 t0 )f (b), atunci f (ta + (1 t)b) = tf (a) + (1 t)f (b), ()t [0, 1].
(iv) Dac
a f este derivabil
a, atunci f este convex
a dac
a si numai dac
a f este
cresc
atoare.
Demonstratie. (i) Se aplic
a faptul c
a orice functie afina (de gradul nt
ai) verific
a
definitia functiei convexe cu egalitate.
(ii) Fie x [a, b]. Atunci exista t [0, 1] astfel nc
at x = ta + (1 t)b si, cum f
este convex
a, deducem c
a f (x) = f (ta + (1 t)b) tf (a) + (1 t)f (b) = 0.
(iii) Din f convex
a si din faptul c
a orice functie afina verific
a definitia convexit
atii
cu egalitate deducem c
a functia : [0, 1] R, (t) = f (ta + (1 t)b) tf (a) (1
t)f (b), este convex
a, iar (0) = (t0 ) = (1) = 0 si (t) 0, ()t [0, 1]. Dac
a ar
exista t (0, 1)\{t0 } astfel nc
at (t ) < 0, din convexitate s-ar obtine o contradictie.
Intr-adev
ar, avem: (a) dac
a 0 < t < t0 , atunci exista s (0, 1) astfel ca t0 =

st + (1 s) 1, de unde (t0 ) s (t ) + (1 s)(1) = s (t ) < 0, ceea ce este fals.


(b) dac
a t0 < t < 1 atunci exista s1 (0, 1) astfel ca t0 = s1 0 + (1 s1 )t si se
obtine iarasi contradictie.
(iv) Fie x, y I cu x < y. Conform unui rezultat cunoscut ([1],[2]), pentru orice
f (v) f (u)
f (v) f (y)
f (u) f (x)

, de unde
u, v I cu x < u < v < y avem
ux
vu
vy

deducem c
a f (x) f (y). Reciproc, fie a, b I, cu a < b si t (0, 1). Aplic
and
teorema lui Lagrange pe [a, ta + (1 t)b] si pe [ta + (1 t)b, b], monotonia functiei f
ne conduce la concluzie.
Cititorii interesati de problematica functiilor convexe pot consulta excelentele
monografii [1] si [2]. Rezultatul principal este cuprins n
Propozitia 2. Fie a, b R, a < b si f : [a, b] R o functie derivabil
a cu
proprietatea c
a ()L 0 astfel ca |f (x) f (y)| L|x y|, ()x, y [a, b] si
f (a) = f (b) = 0. Atunci:
L
(i) |f (x)| (x a)(b x), ()x [a, b];
2
L

(ii) |f (x)| (b a), ()x [a, b].


2
Demonstratie. (i) Din |f (x) f (y)| L|x y|, ()x, y [a, b], deducem c
a
functiile f1 , f2 : [a, b] R definite prin f1 (x) = Lx f (x) si f2 (x) = Lx + f (x),
()x [a, b] sunt cresc
atoare, de unde, conform cu Propozitia 1, (i) si (iv), avem
L
c
a functiile F1 , F2 : [a, b] R definite prin F1 (x) = (x a)(x b) f (x) si
2
L
F2 (x) = (x a)(x b) + f (x) sunt convexe. Cum Fi (a) = Fi (b) = 0, i {1, 2},
2
117

din Propozitia 1, (ii) deducem c


a F1 (x) 0, F2 (x) 0, ()x [a, b], de unde
L
L
L
(xa)(bx) f (x) (xa)(bx), ()x [a, b], deci |f (x)| (xa)(bx),
2
2
2
()x [a, b].
L
(ii) Dac
a x = a sau x = b, atunci din punctul (i) deducem c
a |f (x)| (b a).
2
f (x)(t a)
, ()t
Dac
a x (a, b) definim functia h : [a, x] R prin h(t) = f (t)
xa

[a,
a si |h (t1 ) h (t2 )| =
x]. In mod evident, h(a) = h(x) = 0, h este derivabil


f (t1 ) f (x) f (t2 ) + f (x) = |f (t1 ) f (t2 )| L|t1 t2 | si, n consecint
a,

xa
x a
L
h verific
a ipotezele propozitiei. Conform cu punctul (i), |h(t)| (t a)(x t),
2


h(t) h(x)
L
L



(t a), ceea ce conduce la |h (x)| (x a).
()t [a, x], de unde
tx
2
2
|f (x)|
f
(x)
, deducem c
a ()x [a, b] avem |f (x)| |h (x)|+

Cum h (x) = f (x)


xa
xa
L (x a)(b x)
L
L
L
(x a) +
= (b a), adica |f (x)| (b a), ()x [a, b].
2
2
xa
2
2

Observatii. 1) Dac
a exista x0 (a, b) astfel nc
at n Propozitia 2 relatia (i)
are loc cu egalitate, atunci conform cu Propozitia 1, (iii) una dintre functiile F1 si
L
F2 este nula pe [a, b] si, n consecinta, f (x) = (x a)(b x), ()x [a, b] sau
2
L
f (x) = (x a)(b x), ()x [a, b].
2
2) Dac
a exista x0 (a, b) astfel nc
at n Propozitia 2 relatia (ii) are loc cu egalitate,
atunci analizand demonstratia g
asim c
a n acelasi x0 are loc egalitate si la (i) si
conform cu cele din observatia precedent
a g
asim forma functiei f . Dac
a |f (a)| =
L
L
(b a) sau |f (b)| = (b a), ajungem imediat la concluzia c
a F1 sau F2 din
2
2
demonstratia punctului (i) sunt functiile c
autate.
Observat
ie. In fiecare din problemele P1, P2 functia g : [0, 1] R definita prin
Z
x

g(x) =
0

f (t)dt, ()x [0, 1], verific


a conditiile din Propozitia 2 cu a = 0 si b = 1

si, n concluzie, P1 si P2 sunt consecinte ale Zacesteia. In acelasi mod se observa c


a
x
functia g : [a, b] R definita prin g(x) =
f (t)dt este functia din P3 si, cum
a

aceasta functie verific


a conditiile din Propozitia 2, deducem ca P3 este o consecinta
a acesteia.
Bibliografie

1. W.W. Breckner, T. Trif - Convex Functions and Related Functional Equations,


Cluj, 2008.
2. C.P. Niculescu, L.-E. Persson - Convex Functions. Basic Theory and Applications,
Editura Universitaria Craiova, 2003.
118

Aplicatii ale inegalit


atii mediilor
n rezolvarea unor probleme de minim
Lucian TUT
ESCU, Mihai DICU

Abstract. In this note some inequalities are presented; they are obtained by the inequality
of the arithmetical mean and geometrical mean. Moreover, every of the inequalities from 2) to 9)
represents a generalisation of the previous inequality.
Keywords: AM-GM inequality, extremum problems.
MSC 2000: 52A40.

In propozitia ce urmeaza, ultima inegalitate generalizeaz


a pe toate cele care o
preced. Am prezentat gradat acest tip de inegalitati din motive didactice lesne de
nteles.
Propozitie. Au loc urm
atoarele inegalit
ati:
(1)
(2)
(3)

n
n + 2, ()x (0, ), ()n N;
x2
n
(x + y)n + 2n1
2n1 (n + 2), ()x, y (0, ), ()n N;
xy
2xn +

n
Y

(xi + yi ) + 2n1

i=1

i=1

n
n + 3,
x3

(4)

3xn +

(5)

(x + y + z)n + 3n1

(6)

n
Y

n
X
1

n1

n
X
i=1

(7)
(8)

(x1 + x2 + + xk )n +

1
3n1 (n + 3),
xi yi zi

()x (0, ), ()k, n N;


nk n1
k n1 (n + k),
x1 x2 xk
()x1 , x2 , , xk (0, ), ()k, n N;

(xi1 + xi2 + + xik ) + k n1

i=1

()x, y, z (0, ), ()n N;

()xi , yi , zi (0, ), ()n N;

n
kx + k n + k,
x

n
Y

()x, y (0, ), ()n N;

()x (0, ), ()n N;

i=1

(9)

2n1 (n + 2),

n
3n1 (n + 3),
xyz

(xi + yi + zi ) + 3

xi yi

n
X
i=1

1
k n1 (n + k),
xi1 xi2 xik

()k, n N, ()xij (0, ), i = 1, n, j = 1, k.


Demonstratie. Pentru justificarea fiecareia dintre aceste inegalitati vom folosi
inegalitatea mediilor, n diverse moduri, grup
and corespunz
ator.
1 Profesori,

Colegiul National Fratii Buzesti, Craiova

119

1) Scriem membrul nt
ai desf
asurat si apoi utilizam inegalitatea mediilor:
r

1
1
1
1
n
1 1
n+2
2x + 2 = xn + xn + 2 + 2 + + 2 (n + 2)
xn xn 2 2 2 = n + 2,
x
x
x
x
x x
x
n

cu egalitate pentru x = 1 sau n = 0.


(x + y)n
n
2) Imp
artind cu 2n1 , inegalitatea se scrie sub forma
+
n + 2.
n1
2
xy
 x + y 2n 1
(x + y)n
(x + y)n
1
1
1
n+2
Apoi,
+
+

+
+

(n
+
2)
2n
2n
xy
xy
xy
2
(xy)n
r
2n 1
= n + 2. Egalitate avem pentru x = y = 1 sau n = 0.
(n + 2) n+2
xy
(xy)n
3) Proced
and n mod analog, vom avea

n
Y
xi + yi
i=1

n
Y
xi + yi
i=1

n
X
1
i=1

xi yi

(n + 2)

n+2

(n + 2)

n+2

n
Y
xi + yi
i=1
n
Y

i=1

!2

i=1

!2

xi yi

n
Y
1

xi yi

n
Y
1
i=1

xi yi

= n + 2,

cu egalitate pentru x1 = x2 = = xn = y1 = y2 = = yn = 1 sau n = 0.

Observatie. Pentru n = 3 se obtine Problema 26315 [1].


4) si 7) se dovedesc adaptand demonstratia data pentru 1), inegalitatile 5) si 8)
ca si 2), iar 6) ca si 3).
9) Dam n detaliu demonstratia acestei inegalitati, care cuprinde ca niste cazuri
particulare inegalitatile 1)-8), aplic
and n aceeasi maniera inegalitatea mediilor. Not
am
n
n
Q
P
1
n1
(xi1 + xi2 + + xik ) si S =
. S
a demonstram c
a P +k
S
P =
i=1
i=1 xi1 xi2 xik
k P
n
P
P
P
P
1
+

(n + k)k n1 sau n1 + S n + k. Avem n1 + S =


n
k
k
i=1 k
i=1 xi1 xi2 xik

n
1
P k Q
(n + k) n+k
.T
inand seama c
a
n
k
i=1 xi1 xi2 xik
x11 + x12 + + x1k
x21 + x22 + + x2k
xn1 + xn2 + + xnk
P
=

kn
k
k
k

obtinem c
a

P
k n1

k
x11 x12 x1k k x21 x22 x2k k xn1 xn2 xnk ,

+ S (n + k)

n+k

n
Q
i=1

xi1 xi2 xik .

litate avem pentru xij = 1, i = 1, n, j = 1, k.

n
Q

1
= n + k. Egai=1 xi1 xi2 xik

Bibliografie
1. D. S
avulescu, L. Tutescu Problema 26 315, G.M.-B, nr. 6/2010, pag. 324.
120

Asupra unei inegalit


ati aproape clasic
a
Dan D
an
ail
a1
Abstract. In this note we present two inequalities of Cauchy-Buniakovsky-Schwarz type and
some of their consequences.
Keywords: Cauchy-Buniakovski-Schwarz, cardinal number, matrix.
MSC 2000: 26D15.

Vom prezenta doua inegalitati de tip Cauchy-Buniakovski-Schwarz si c


ateva aplicatii ale lor.
Propozitie. Fie numerele m, n N, a1 , a2 , . . . , am R+ , b1 , b2 , . . . , bn R+ ,
x1 , x2 , . . . , xn R, y1 , y2 , . . . , ym R si multimile finite A1 , A2 , . . . , Am , B1 , B2 , . . . , Bn .
Au loc inegalit
atile:

m P
n
P

(1)

(2)

min(ai , bj )xi yj

i=1 j=1
m P
n

P
Ai Bj xi yj

i=1 j=1

m
P
Ai Aj |xi xj

min(bi , bj )yi yj

1i,jn

i,j=1

min(ai , aj )xi xj

1i,jm

n

P
Bi Bj yi yj .

i,j=1

Demonstratie. (1) Fie multimea {c1 , c2 , . . . , cr } = {a1 , a2 , . . . , am , b1 , b2 , . . . , bn },


cu 0 c1 < c2 < . .
. < c
si
r , l1 = c
1 , l2 = c2 c1 , l3 = c3 c2 , . . . , lr = cr cr1
matricea D = diag( l1 , l2 , . . . , lr). Fie vectorii di = (d
,
d
,
.
.
.
,
d
)

s
i
e
=
ir
i
i1 i2
1, ai cj
1, bi cj
(ei1 , ei2 , . . . , eir ), definiti prin dij =
si eij =
, si vectorii
0, ai < cj
0, bi < cj
di = xi di D, ei = yi ei D. Cu aceste notatii, inegalitatea (1) se rescrie astfel:

m P
n
P
i=1 j=1

di t ej

1i,jm

m
P

i=1

Dac
a not
am cu u =

di

m
P

i=1

di t dj

n
P
j=1

2
t
ej

di si v =

(u v) (u u)(v v)

1i,jn

n
P
i=1

P
m
P
i=1

ei t ej


di

m
P
i=1

t
di



n
P
i=1



ei

n
P
i=1


t
ei

ei , inegalitatea devine:

r
X

!2

ui vi

i=1

r
X
i=1

u2i

r
X

vi2

i=1

Observand c
a am ajuns la inegalitatea CBS aplicata numerelor u1 , u2 , . . . , ur si
v1 , v2 , . . . , vr , demonstratia se ncheie.
(2) Fie multimea C = A1 A2 . . .Am B1 B2 . . .Bn , cu C = {c1 , c2 , . . . , cr }.
Consider
am vectorii di = (di1 , di2 , . . . , dir ) si ei = (ei1 , ei2 , . . . , eir ), unde dik =
1 Elev,

cl. a XI-a, Colegiul National V. Alecsandri, Galati

121

1, ck Ai
1, ck Bi
si eik =
si di = xi di , ei = yi ei . Folosind aceste notatii,
0, ck
/ Ai
0, ck
/ Bi
inegalitatea se rescrie n forma
m X
n
X

!2

di t ej

i=1 j=1

di t dj

1i,jm

ei t ej

1i,jn

cu care ne-am nt
alnit mai sus. Continu
and n acelasi fel, deducem c
a (2) este
adevarata.

Consecinta 1. Fie a1 , a2 , . . . , an > 0 si xi , yi R, i = 1, n. Atunci


X

min(ai aj , 1)xi yj

1i,jn

min(ai , aj )xi xj

1i,jn

min(ai , aj )yi yj

1i,jn

Demonstratie. Consideram m = n si sirurile a1 , a2 , . . . , an R+ , b1 =

1
, b2 =
a1

1
1
, . . . , bn =
, x1 , x2 , . . . , xn R, y1 a1 , y2 a2 , . . . , yn an R. Aplic
and inegalitatea
a2
an
(1), obtinem:

n
P

n
P
1
min ai ,
xi yj aj
min(ai , aj )xi xj

aj
1,j=1
1,j=1
de unde rezulta imediat inegalitatea dorit
a.



P



Consecinta 2.
min(ai aj , 1)xi xj
1i,jn

n
P

min

1,j=1

1 1
,
ai aj

min(ai , aj )xi xj

y i y j ai aj

(ai > 0,

1i,jn

xi R, i = 1, n).

Consecinta 3. Matricele (min(ai , aj ))i,j=1,n (min(ai aj , 1))i,j=1,n ) sunt pozitiv


semidefinite.
Demonstratie. Se deduce din Consecinta 2. Acesta este un rezultat important
si nebanal!
n
P

Consecinta 4.

i,j=1

min(xi xj , yi yj )

n
P
i,j=1

min(xi yj , xj yi )(xi , yi R+ , i = 1, n).

(USAMO 2000, Gheorghe Zb


aganu).
Demonstratie. Far
a a restrange generalitatea putem considera xi , yi R+ ,
yi
, bi = xi yi , xi , yi R+
i = 1, n. Aplic
and inegalitatea (1) pentru m = n, ai =
xi
(i = 1, n), obtinem

n
P

n
n
P
P
yi xj
yi yj
xi xj
min
min
min
xi yj
xi xj
yi yj ,
,
,
,

x
y
x
x
yi yj
i
j
i
j
i,j=1
i,j=1
i,j=1
de unde inegalitatea de demonstrat. Astfel am obtinut o rezolvare elegant
a si rapida
a cunoscutei si dificilei inegalitati propus
a de Gheorghe Zb
aganu la USAMO 2000.

122

Consecint
a 5.

n
n P
P

min(ai , bj )

i=1 j=1

n
P

min(ai , aj )

i,j=1

n
P

min(bi , bj ) ,

i,j=1

a1 , a2 , . . . , am R+ , b1 , b2 , . . . , bn R+ (Don Zagier). C
and are loc egalitatea?

Demonstratie. Dac
a consider
am m = n, ai , bi R+ , xi = yi = 1(i = 1, n) si
aplic
am inegalitatea (1), obtinem inegalitatea lui Don Zagier.
Partea cea mai interesant
a a inegalitatii este cazul de egalitate, care se stabileste
usor apel
and la demonstratia dat
a pentru inegalitatea (1). Anume, folosind conditiile
u1
n care avem egalitate n inegalitatea CBS, deducem c
a egalitatea are loc c
and
=
v1

ur
u2
a c
a ui = vi , i = 1, n. Rezult
a
= ... =
; dar u1 = v1 = n l1 , ceea ce implic
v2
vr
c
a n fiecare din sirurile (ai )i=1,n si (bi )i=1,n , ck apare de acelasi numar de ori (ck de
uk
uk+1
ur
p
, k = 1, r 1 ori, iar cr de ori). Deci egalitatea are loc c
and sirul
lk+1
lk
lr
(bi )i=1,n este o permutare a sirului (ai )i=1,n .

Consecinta 6.

m P
n
P
i=1 j=1

|Ai Bj |

m
P

i,j=1

|Ai Aj |

n
P

i,j=1

|Bi Bj | , unde

n, m N si A1 , A2 , . . . , Am , B1 , B2 , . . . , Bn sunt multimi finite.

Demonstratie. Se aplic
a inegalitatea (2) pentru multimile A1 , A2 , . . . , Am ,
B1 , B2 , . . . , Bn si sirurile x1 = x2 = . . . = xn = 1, y1 = y2 = . . . = ym = 1 si se
obtine inegalitatea de mai sus.

Observatie. Toate aceste inegalitati se bazeaz


a pe aplicarea inegalitatii CBS
ntr-un mod ingenios si pe observarea faptului c
a sumele din dreapta sunt pozitive n
ciuda faptului c
a x1 , x2 , . . . , xn R si y1 , y2 , . . . , ym R.

Premiu pe anul 2011 acordat


de ASOCIAT
IA RECREAT
II MATEMATICE
Se acord
a un premiu n bani n valoare de 200 lei elevului

AIL
A
Dan
DAN

Colegiul National V. Alecsandri, Galati

pentru nota Asupra unei inegalit


ati aproape clasic
a aparuta n acest numar al
revistei Recreatii Matematice.

123

Autour du cardinal dun ensemble


de matrices binaires
Adrien REISNER 1
Abstract. We here study a couple of algebraic and analytic properties of certain binary matrices
in the spaces Mn (R). In particular, we study power series whose coefficients are functions of the
cardinal of these matrices, as well the rank of this family of matrices.
Keywords: binary bistochastic matrices, rank, power series.
MSC 2000: 15A03, 15A18.

On consid`ere l ensemble Un des matrices binaires de taille n comportant exactement deux 1 dans chaque ligne et exactement deux 1 dans chaque colonne (pour toute
1
matrice A Un la matrice A est bistochastique). On designe par un = Card Un et
2
on pose u0 = 1, u1 = 0. Jn est la matrice de Mn (R) dont tous les coefficients sont
egaux `
a 1.

1 1
.
1 1
Donc u2 =1. Pour n=3, U3 est forme des 6 matrices Aj , j : 1, . . . , 6 (suivant
la position

0 1 1
du seul element nul (a1i )i:1,2,3 de la premi`ere ligne): (a11 = 0) : A1 = 1 1 0 ,
1 0 1

0 1 1
1 0 1
1 0 1
A2 = 1 0 1 ; (a12 = 0) : A3 = 1 1 0 , A4 = 0 1 1 ; (a13 = 0) :
1 1 0
0 1 1
1 1 0

1 1 0
1 1 0
A5 = 1 0 1 , A6 = 0 1 1 . Donc u3 = 6.
0 1 1
1 0 1
Exemples. Pour n = 2, la seule matrice de U2 est la matrice J2 =

1. Etude
de un . On designe par Hn le sous-ensemble de Un comportant un 1 en
position (1, 1) et hn = Card Hn . En outre, Kn est le sous-ensemble de Hn comportant
un 1 en position (1, 2) et un 1 en position (2, 1) et kn = Card Kn . X0 = [1] designant
le vecteur de Rn dont tous les coefficients sont egaux `a 1 on a le
Th
eor`
eme 1. a) Pour toute matrice A Un , 2 est valeur
P propre de A et X0 est
A = h n Jn .
un vecteur propre associe `
a cette valeur propre. b) On a:
AUn

D
emonstration. a) Pour toute matrice A Un il est immediat que le vecteur X0
1
est vecteur propre associe `
a la valeur propre = 2 (la matrice A est bistochastique):
2
AX0 = 2X0 , A Un .
1 TELECOM

ParisTech; e-mail: Adrien.Reisner@telecom-paristech.fr

124

b) Pour (i, j) fixe soit (Un )ij l ensemble (Un )ij = {A Un ; aij = 1} et pour toute
matrice A de Un designons par A la matrice obtenue `a partir de la matrice A en
echangeant les lignes 1 et i puis les colonnes 1 et j. Il est immediat que cette matrice
A appartient `
a l ensemble Hn et que l application (Un )ij Hn , A 7 A est bijective.
Par suite pour tout i, j : 1, . . . , n : Card(Un )ij = Card Hn = hn , d o`
u l assertion b).
n
hn .
2
P
D
emonstration. Compte tenu des deux assertions precedentes il vient:
AX0

Corollaire. On a un =

AUn

= hn Jn X0 soit 2un X0 = nhn X0 et le corollaire est ainsi demontre, le vecteur X0 etant


vecteur propre de la matrice Jn pour la valeur propre n.
Th
eor`
eme 2. On a: a) hn = (n 1)2 kn , n 2; b) kn = un2 + hn1 pour n 4.

D
emonstration. a) Pour tout (i, j), 2 i, j n, soit Hnij l ensembles des
elements de Hn ayant un 1 dans la position (i, 1) et un 1 en position (1, j). Ces
ensembles Hnij , 2 i, j n, constituent une partition de Hn et il y a (n 1)2 de
tels ensembles. Remarquons que Hn22 = Kn . De plus, l application i,j : Hnij Kn ,
A 7 Aij , o`
u la matrice Aij est obtenue `a partir de la matrice A par echange des lignes
2 et i et des colonnes 2 et j (lorsque i = 2 ou j = 2 on ne fait pas d echange) est de
facon evidente bijective, soit pour tout (i, j), 2 i, j n, Card Hnij = Card Kn = kn .
On en deduit alors pour n 2 : hn = Card Hn =

n
P

i,j=2

Card Hnij =

n
P

Card Kn =

i,j=2

(n 1)2 kn .
b) Pour n 4, Kn est reunion disjointe des deux parties suivantes: 1) Kn1
ensemble des elements de Kn ayant un 1 dans la position (2, 2). L application de Kn1
dans Un2 qui `
a chaque matrice A fait associer la matrice obtenue `a partir de A en
supprimant les deux premi`eres lignes et les deux premi`eres colonnes est manifestement
bijective; donc: Card Kn1 = Card Un2 = un2 . 2) Kn2 ensemble des elements de
Kn ayant un 0 dans la position (2, 2). Dans ce cas on consid`ere l application de
Kn2 dans Hn1 qui `
a chaque matrice A fait associer la matrice obtenue de celle-ci
en remplacant le 0 de la position (2, 2) par 1 puis en supprimant la premi`ere ligne
et la premi`ere colonne. On definit ainsi une application bijective. Il vient alors:
Card Kn2 = Card Hn1 = hn1 . Finalement, pour n 4 on a: kn = Card Kn =
Card Kn1 + Card Kn2 = un2 + hn1 .
On pose pour tout n N: wn =

un
.
(n!)2

n1
1
wn2 +
wn1 ,
Th
eor`
eme 3. a) wn verifie la relation de recurrence wn =
2n
n
n 2, avec w0 = 1 et w1 = 0.
b) wn [0, 1] pour tout n N.
c) La serie de terme general wn diverge.
d) La serie

n=0

wn xn converge pour x ] 1, 1[.

D
emonstration. a) Pour n 4 il vient, compte tenu du theor`eme precedent et
125

du corollaire:
n
n
n
un = hn = (n 1)2 (un2 + hn1 ) = (n 1)2 un2 + n(n 1)un1 .
2
2
2
A l aide des conventions u0 = 1 et u1 = 0 cette relation est encore verifiee pour n = 2
un
1
n1
et n = 3. On a alors pour n 2: wn =
=
wn2 +
wn1 .
2
(n!)
2n
n
b) Pour tout n, un 0, donc wn 0. Montrons par recurrence que wn 1,
n N . D abord, w0 = 1 1 et w1 = 0 1. Soit n 2 et supposons que wn2 1 et
1
n1
wn1 1; alors, compte tenu de l assertion precedente: wn =
wn2 +
wn1
2n
n
n1
2n 1
1
+
=
1.
2n
n
2n
c) Il est clair que wn > 0, n 2. Compte tenu de l assertion a), il vient: wn
2
1
1
n1
wn1 et par suite: wn w2 =
u2 =
, pour n 3. Ceci montre que la
n P
n
2n
2n
serie
wn diverge.
d) 0 wn 1 (voir b)) implique |wn xn | |xn |. La serie geometrique xn etant
convergente pour 1 < x < 1, on deduit que

wn xn est absolument convergente,

n=0

donc convergente pour |x| < 1.

e 2 x
.
wn xn =
1x
n=0
D
emonstration. Soit x ] 1, 1[. D apr`es l assertion a) du Theor`eme 3,
Th
eor`
eme 4. Pour x ] 1, 1[ on a: W (x) =

2nwn = 2(n 1)wn1 + wn2 ,

n 2.

En multipliant les deux membres de cette egalite par xn1 , puis en sommant, on
obtient alors:
2

nwn xn1 = 2x

n=2

(n 1)wn1 xn2 + x

n=2

wn2 xn2 .

n=2

Il vient: 2(W (x) w1 ) = 2xW (x) + xW (x) soit puisque w1 = u1 = 0:


W (x) =

x
W (x),
2(1 x)

x ] 1, 1[.

D ici et du fait que W (0) = 1, on obtient alors


1

e 2 x
,
W (x) =
1x

x ] 1, 1[.

Remarque. Une etude plus approfondie permet de trouver, en utilisant la fonction ainsi que la formule de Stirling, un equivalent de un lorsque n , `a savoir:
 n 2n+ 12
, mais cette etude depasse le niveau de cet article.
un 2
e
126


2. Etude
de rang. On se propose dans cette partie de determiner le rang rn du
syst`eme constitue des un , matrices de Un considerees comme elements de Mn (R).
Proposition. Avec ces notations on a : r2 = 1 et r3 = 5.
D
emonstration. Pour n = 2, U2 contenant la seule matrice non nulle J2 : r2 = 1.
Pour n = 3, U3 contient les 6 matrices Ai , i : 1, . . . , 6, trouvees page 124. Designons
par J le sous - espace vectoriel de Mn (R) engendre par ces 6 matrices. D apr`es
l assertion b) du Theor`eme 1, la matrice J3 appartient J et par suite J est aussi
engendre par
J Ai
, i : 1, . . .
, 6. Ces 6 matrices
sont
dans l ordre:
les 6 matrices

1 0 0
1 0 0
0 1 0
J A1 = 0 0 1 , J A2 = 0 1 0 , J A3 = 0 0 1 , J A4 =
0 1 0
0 0 1
1 0 0

0 1 0
0 0 1
0 0 1
1 0 0 , J A5 = 0 1 0 , J A6 = 1 0 0 . Soient i , i : 1, . . . , 6,
0 0 1
1 0 0
0 1 0

1
2
3
4
5
6
6
P
4 + 6 2 + 5 1 + 3
des reels donnes. Alors
i (J Ai ) = 0
=
i=1
3 + 5 1 + 6 2 + 4

0 0 0
0 0 0 1 = 2 = 3 = 4 = 5 = 6 . On en deduit r3 = 5.
0 0 0
Soit Vn l espace vectoriel des matrices A Mn (R) telles que X0 = [1] soit `a
la fois vecteur propre de A et de sa transposee t A. [Vn est effectivement un sousespace vectoriel de Mn (R) car 0 Vn et si (, ) R2 et (A, B) (Vn )2 alors:
(A + B)X0 Vect(X0 ) et (t A + t B)X0 Vect(X0 ).]
Th
eor`
eme 5. Un Vn . Pour A Vn si AX0 = X0 , alors t AX0 = X0 .

D
emonstration. Si A Un , sa transposee t A appartient aussi `a Un . Comme X0
est vecteur propre de toute matrice de Un on a: Un Vn .
Soit alors A = (aij ) Vn , la valeur propre de A associee au vecteur propre X0 ,
i.e. AX0 = X0 et celle de la transposee t A associee `a X0 : tAX0 = X0 . Pour
tout i, j : 1, . . . , n, on a:
n P
n
P

n
P

k=1

aik = et

n
P

akj = . Par suite: n =

k=1

n
n P
P

aij ) =

i=1 j=1

aij ) = n soit = .

j=1 i=1

Th
eor`
eme 6. a) dim Vn = (n 1)2 + 1; b) rn (n 1)2 + 1.
D
emonstration. a) On munit Mn,1 (R) de sa structure euclidienne canonique
1
et on note B0 = {e1 , . . . , en } la base canonique de Mn,1 (R). Soit e1 = X0 . On
n
compl`ete la famille orthonormee (e1 ) en une base orthonormee B1 = {e1 , . . . , en }
de Mn,1 (R). On note P la matrice de passage de B0 `a B1 et pour A Mn (R)
soit f l endomorphisme de Mn,1 (R) canoniquement associe `a A. Compte tenu du
127

theor`eme 5:
A Vn P/Ae1 = tAe1 = e1

P, A Mn1 (R)/M at(f, B1 ) =

P, A Mn1 (R)/A = P

0 A

0
P 1 .
0 A

Il vient alors, l application M 7 P M P 1 etant un automorphisme de Mn (R),

dimVn = dim{

0
; R, A Mn1 (R)} = 1 + (n 1)2 .
A

b) On en deduit puisque Un Vn : rn = dim(Vect(Un )) dimVn = (n 1)2 + 1.


Pour n 3 soit A = (aij ) une matrice de Un comportant des 1 en positions (1, 1)
et (2, 2) et des 0 en positions (1, 2) et (2, 1). La matrice B = (bij ) definie par bij = aij
si i > 2 ou j > 2, bij = 1 aij si i 2 et j 2 est une matrice binaire ayant les
memes lignes et les memes colonnes que A `a partir de la troisi`eme ligne ou colonne.
La definition des elements (1, 1), (2, 2), (1, 2) et (2, 1) de cette matrice B montre alors
que B Un . Pour tout i, j : 1, . . . , n : aij bij = 0 si i > 2 ou j > 2; aij bij = 1 si
i = j, i 2 et j 2; aij bij = 1 si i 6= j, i 2 et j 2. La matrice A B ne
comporte donc que des elements nuls sauf en positions (i, j) avec i 2 et j 2.
Soit rn le rang du syst`eme constitue de toutes les matrices U U o`
u U, U Un .

Th
eor`
eme 7. a) rn (n 1)2 ; b) rn = n2 2n + 2.

D
emonstration. a) Pour tout i, j : 1, . . . , n notons Aij la matrice obtenue `a
partir de la matrice A par lechange des lignes i et 2 et les colonnes j et 2 et par Bij
la matrice obtenue `
a partir de la matrice B par les memes echanges. Ces matrices Aij
et Bij appartiennent `
a Un . De plus, Aij Bij est une matrice `a coefficients nuls sauf
en positions (1, 1), (i, j), (1, j) et (i, 1) et telle que lelement de la position (i, j) est
1. On remarque que par suppression de la premi`ere ligne et de la premi`ere colonne
de Aij Bij on obtient la matrice Ei1,j1 de la base canonique de Mn1 (R). La
famille {Aij Bij }i,j:2,...,n est donc libre, et par suite: rn (n 1)2 .
b) D apr`es l assertion b) du Theor`eme 6 et de l assertion a) precedente on a les
inegalites suivantes: (n1)2 rn rn (n1)2 +1. Montrons que l inegalite rn rn
est stricte. En effet, pour tout U, U Un on a (U U )X0 = 0. Or Jn X0 = nX0 et
par suite la matrice Jn ne peut etre combinaison lineaire des elements U U avec
U, U Un , mais elle est combinaison lineaire des elements de Un (voir Theor`eme 1
b)). Donc on a l inegalite stricte: rn < rn .
On en deduit finalement legalite: rn = (n 1)2 + 1.

128

Comentarii pe marginea unei probleme


Neculai ROMAN 1
Abstract. The author shows how a simple problem of elementary geometry may be a source of
new problems and results.
Keywords: equilateral triangle, orthocenter, incenter.
MSC 2000: 51M04.

La Concursul Al. Myller, ed. a VIII-a, 2010, elevii din cl. a VII-a au fost pusi
n fata problemei (v. Recreatii Matematice, 2/2010, p.134):
(P): Se consider
a triunghiul ABC n care AB = AC si fie M mijlocul segmentului
[BC]. Punctele D si E sunt picioarele perpendicularelor din M pe dreptele AC,
respectiv AB, iar H este mijlocul segmentului [DE]. Fie punctele P, Q BC astfel
nc
at M Q = M P = M D si P (BM ). Ar
atati c
a punctul H este ortocentrul
triunghiului AP Q.
Constat
am c
a problema (P) are trei elemente constitutive: 1) un triunghi ABC, 2)
un punct M BC si 3) un triunghi AP Q (construit astfel: D, E fiind proiectiile lui M
pe AC, respectiv AB, atunci punctele P, Q BC cu P (BM ) sunt determinate prin
M P = M E si M Q = M D), iar rezultatul problemei (P) are structura: n anumite
ipoteze relativ la triunghiul ABC si punctul M , urmeaza o anumita proprietate pentru
triunghiul AP Q.
Astfel privit
a, (P) devine o surs
a de noi probleme si rezultate. Intr-adev
ar, punctul
M , care n (P) este mijlocul laturii [BC], poate fi luat picior de naltime, bisectoare
etc. Relativ la triunghiul AP Q putem avea n vedere propriet
ati ca: este isoscel, are
otrocentrul sau alt punct important pe DE, AM DE etc. In aceasta configuratie
exista suficient loc pentru curiozitatea si imaginatia oricui, iar pentru descoperirea
propriet
atilor ei este nevoie de curaj, munca, rabdare.
Pregatim rezolvarea problemei (P) cu urmatoarea
Lem
a. Fie ABC un triunghi oarecare si M piciorul n
altimii din A pe BC.
Atunci, un punct H AM este ortocentrul triunghiului ABC dac
a si numai dac
a
M B M C = M H M A.

Demonstratie. Dac
a H este ortocentru, avem HM B CM A si, deci,
MB
HM
A
=
, de unde rezulta relatia dorit
a.
MC
MA
Fie, acum, B BH AC. Din AM BC si M B M C =
M H M A rezulta c
a HM B CM A, deci unghiurile marB
H
cate sunt congruente si BB AC, i.e. H este ortocentru.
1 Profesor,

S
coala V. Alecsandri, Mircesti (Iasi)

129

Demonstratia afirmatiei problemei (P). Din simetria figurii rezulta c


a DEkBC
si H AM. Cu teorema catetei obtinem M D2 = M A M H, de unde M P M Q =
M A M H. Conform Lemei, deducem c
a H este ortocentrul triunghiului AP Q.
Observatie. Rezolvarea data problemei (P) difera de cea oficiala (din barem).
In continuare, vom prezenta c
ateva reciproce si o generalizare a problemei (P).
A

Reciproca 1. Fie M piciorul n


altimii din v
arful A al triunghiului ABC, D si E proiectiile lui M pe AC, respectiv AB
H P
D
si punctele P, Q BC astfel nc
at M P = M E, M Q = M D
E
si P (BM ). Dac
a AP Q este isoscel de v
arf A, atunci si
B P
ABC este isoscel.
QC
M
Demonstratie. In conditiile enuntului, AM BC implic
a faptul c
a [M P ]
Cum aceasta
[M Q]. Deci [M E] [M D] si [AM ] este bisectoarea unghiului BAC.
este si naltime n ABC, rezulta c
a triunghiul este isoscel.
Reciproca 2. Consider
am un punct M (BC) si fie punctele D, E, P, Q construite ca n Reciproca 1. Dac
a AP Q este isoscel de v
arf A si AM DE, atunci
si ABC este isoscel.
Demonstratie. In cercul circumscris patrulaterului AEM D diametrul [AM ] si
coarda [DE] sunt perpendiculare, deci [M E] [M D]. Ca urmare, [M Q] [M P ],
deci AM BC. Conform Reciprocei 1, rezulta c
a ABC este isoscel.
Reciproca 3. Consider
am un punct M (BC) si fie punctele D, E, P, Q construite ca n Reciproca 1. Dac
a punctul H = AM DE este ortocentrul triunghiului
AP Q, atunci ABC este isoscel.

Demonstratie. Evident, [AM ] este naltime n ABC. Ca urmare, DE este


antiparalel
a cu BC (fapt cunoscut si usor de dovedit!). Fie P = P H AQ. In
QP tg P
P Q = 1. Dar

triunghiul dreptunghic P P C avem: tg P


QP =

tg P

AM
AM
1
=
=
si
MQ
MD
cos C

P Q =

tg P

EH
HM
cos C
HM
sin M
cos C
=
=
=
=
.

MP
ME
sin(90 B + C)
cos(B C)

sin M
HE

Ca urmare,

1
cos C

= 1, de unde cos(B C) = 1 si, deci, B = C.


cos C cos(B C)

In rezultatele precedente punctul M (BC) aparea (de la bun nceput sau pe


parcurs!) ca fiind piciorul naltimii din A. La nceputul notei au fost sugerate c
ateva
directii de investigare a configuratiei n discutie. Cititorul atras si dornic de aventuri
n c
ampul geometriei este ndemnat sa-si croiasc
a propriul traseu. Noi vom ncheia
cu un rezultat n care punctului M i se atribuie o alt
a pozitie pe latura [BC].
b Se
Fie ABC un triunghi oarecare si M BC piciorul bisectoarei unghiului A.
consider
a punctele D, E, P, Q ca n reciprocele de mai sus si punctul H ca intersectie
130

a dreptei DE cu perpendiculara din A pe BC. Atunci H este ortocentrul triunghiului


AP Q.
si C
sunt ascutite (alte cazuri se
Demonstratie. Presupunem c
a unghiurile B
trateaza similar sau sunt triviale). Fie A1 proiectia punctului A pe BC si N mijlocul
segmentului [DE].
A
Din faptul c
a [AM ] este bisectoare, deducem c
a AED este
isoscel. Cum [AN ] este mediana n AED isoscel, rezulta c
a
[AN ] este bisectoare si, deci, N AM .
E
Avem cu teorema catetei,
N H
D
(1)

M P 2 = M D2 = M A M N ;

B P

M A1

QC

de asemenea, scriind n doua moduri puterea punctului A1 fata de cercul circumscris


patrulaterului P EDQ, are loc egalitatea
A1 P A1 Q = M P 2 M A21 .

(2)
Din (1) si (2) rezulta c
a
(3)

A1 P A1 Q = M A M N M A21 .

Dar AN AM = AA1 AH (puterea lui A fata de cercul circumscris patrulaterului


A1 HN M ) sau
AM (AM M N ) = AA1 (AA1 A1 H) AM 2 M A M N = AA21 A1 A A1 H
(4)

AA21 + M A21 M A M N = AA21 A1 A A1 H

M A21 = M A M N A1 A A1 H.

Din (3) si (4) rezulta c


a A1 P A1 Q = A1 A A1 H, deci, conform Lemei, H este
ortocentrul triunghiului AP Q.
Observatie. Acest rezultat reprezint
a o generalizare a problemei (P).

Vizitati pagina web a revistei Recreatii Matematice:

http://www.recreatiimatematice.ro

131

Propriet
ati ale triunghiurilor n care 3a = b + c

Ion PATRAS
CU1
Abstract. The aim of this paper is to point out some properties of the triangles for which the
relation 3a = b + c holds.
Keywords: incenter, centroid, Nagel point.
MSC 2000: 51M04.

Vom pune n evident


a c
ateva propriet
ati ale triunghiurilor n care suma a doua
laturi este triplul celei de-a treia. Vom vedea c
a aceasta particularitate impune anumitor puncte importante ale triunghiului (cum ar fi Gcentrul de greutate, N punctul
lui Nagel) sa aib
a pozitii speciale fata de elementele triunghiului.
C
ateva propriet
ati imediate sunt date de
Propozitia 1.
Intr-un triunghi ABC urm
atoarele afirmatii sunt echivalente:
ha
, 5) S = ara , 6) ra = 2r,
1) 3a = b + c, 2) p = 2a, 3) S = 2ar, 4) r =
4
B
C
1
7) 3 sin A = sin B + sin C, 8) tg tg = .
2
2
2
aha
Demonstratie. Vom utiliza formule cunoscute, ca: S =
= pr = (p a)ra ,
2

(p b)(p c)
A
etc.
tg =
2
p(p a)
ha
aha
= 2ar
= r.
Evident, 1)2)3). Pentru 3)4) scriem: S = 2ar
2
4
Din S = (p a)ra , deducem usor c
a 2)5). Pentru a dovedi c
a 2)6), observam mai
nt
ai c
a pr = (p a)ra (= S); atunci 2) p = 2(p a) pr = 2(p a)r (p a)ra =
2(p a)r ra = 2r 6). Evident, 1)7) (teorema sinusurilor). In sfarsit, 2)8),
B
C
pa
A
B
c
aci tg tg =
n orice triunghi (cu formulele pentru tg si tg ) si atunci
2
2
p
2
2
pa
1
B
C
1
2)
= tg tg = b).
p
2
2
2
2
Propozitia 2. Fie ABC un triunghi oarecare si E, F punctele de tangent
a a
cercului nscris cu laturile AC, respectiv AB. Atunci, relatia 3a = b + c are loc dac
a
si numai dac
a G (EF ).

Demonstratie. Fie 3a = b + c. Not


am cu T intersectia dreptelor EF si BC
(figura corespunde cu cazul b > c). Mention
am faptul c
a EF kBC AB = AC si c
a
excludem cazul triunghiului isoscel ca banal. Fie M mijlocul segmentului [BC] si G
intersectia dreptelor AM si EF .
1 Profesor,

Colegiul National Fratii Buzesti, Craiova

132

Se stie c
a avem relatiile AE = AF =
A
p a, care, n ipoteza noastra, se scriu
AE = AF = a. Cu teorema lui Menelaus
aplicata ABC si transversalei T F E
G E
pb
TB
F
=
. Ca urmare,
obtinem:
.
TC
pc
I
pb
TB
TB
2(p b)
=
sau a =
sau,
T
B
M C
a
bc
bc
2
TB
2(p b)
nc
a,
=
. Pe de alt
a parte, aplic
and teorema lui Menelaus la ABM
TM
a

TB
a
G A
GA
=

, de unde
= 2, adica
si transversala T F G , obtinem
GM
TM p b
GM
G G.
TB
2(p b)
Reciproc, dac
a G (EF ), obtinem ca mai sus
=
. Cum GA =
TM
a
2GM , teorema lui Menelaus aplicat
a la ABM si transversala T F G ne d
a
2(p b)
TB
TB
=
. Egaland valorile obtinute pentru raportul
, avem a = p a,
TM
pa
TM
adica 3a = b + c. Propozitia este complet demonstrat
a.
Not
am cu Da , Eb , Fc punctele de tangenta a cercurilor A-exnscris, B-exnscris si
C-exnscris cu laturile BC, CA si, respectiv, AB. Se stie c
a BFc = CEb = p a,
CDa = AFc = p b si AEb = BDa = p c. Cevienele ADa , BEb , CFc se numesc
ceviene Nagel si sunt concurente ntr-un punct N punctul lui Nagel (concurenta se
dovedeste cu teorema lui Ceva, pe baza egalitatilor precedente).
Propozitia 3. Un triunghi are suma a dou
a laturi egal
a cu triplul celei de-a treia
dac
a si numai dac
a punctul N apartine cercului s
au nscris.
Demonstratie. Este cunoscuta formula
IN 2 = p2 16Rr + 5r2 .
Faptul c
a N apartine cercului nscris se exprim
a prin IN = r. T
inand seama de
S
abc
2
formulele R =
, r = si S = p(p a)(p b)(p c) si fortand aparitia factorilor
4S
p
p 2a, p 2b, avem (cu calcule de rutina!):
IN = r p2 16Rr + 5r2 = r2 p3 4abc + 4(p a)(p b)(p c) = 0
[(p 2a) + 2a]3 4abc + 4[(p 2a) + a](p b)(p c) = 0

(p 2a)[p2 2ap + 4(p b)(p c)] = 0

(p 2a){[(p 2b) + 2b]2 2a[(p 2b) + 2b] + 4[(p 2b) + b](p c)} = 0

(p 2a)(p 2b)[(p 2b) 2a + 4(p c)] = 0

(p 2a)(p 2b)(p 2c) = 0


(b + c 3a)(c + a 3b)(a + b 3c) = 0.

Prin urmare, N este pe cercul nscris daca si numai daca b + c = 3a sau c + a = 3b


sau a + b = 3c, ceea ce ncheie demonstratia.
133

Colegiul National Gheorghe Rosca Codreanu


din B
arlad

Comisul Gheorghe Rosca Codreanu (Codrianu) s-a n


ascut pe 10 martie 1805
si s-a stins din viat
a la Viena, foarte tnar, pe 19 noiembrie 1837 (mult
a vreme s-a
crezut c
a de tuberculoz
a; cercetari mai recente par sa indice un cancer). Din motive
financiare, nimeni nu s-a nvrednicit sa-l aduc
a n tara, asa c
a a ramas p
ana azi n
groapa comun
a.
Testamentul scris cu o zi naintea mortii face dovada unei luciditati extreme (prin
indicatiile pe care le d
a pentru administrarea averii, plata datoriilor si asa mai departe)
precum si pe cea a unei marinimii pasoptiste (de sorginte iluminista): cu limb
a de
moarte, Gheorghe Rosca Codreanu cerea nfiintarea unei scoli pentru educarea a
o sut
a de copii de pe una din mosiile sale. Totul urma a se realiza doar din averea sa,
dar administratorul acestei averi a t
araganat lucrurile, astfel c
a dorinta comisului avea
sa se mplineasca abia la 1846, c
and s-a nfiintat la Barlad Clasul Real al fundatiei
Codrianului ce avea sa functioneze n aceasta forma p
ana n 1858. Aici se studiau
gramatica, aritmetica, geografia, catehismul si, dupa dorinta expresa a fondatorului,
limba latin
a.
Conditiile materiale de care dispunea scoala erau precare, dar pasul nainte facut
prin nfiintarea ei este unul de necontestat. Trebuie sa mention
am c
a Neculai Rosca
Codreanu, fratele lui Gheorghe, a lasat si el, tot prin testament (la 30 ianuarie 1854)
o suma important
a de bani pentru cresterea nvatam
antului b
arladean: sase mii de
134

galbeni pentru nfiintarea unei scoli de fete (dupa acumularea unui capital din dobnzi
- remarc
am din nou luciditatea si chibzuinta acestor oameni luminati) si doua mii de
galbeni din al c
arora venit s
a tie doi profesori cu dou
a catedre, una de limba francez
a
si una de limba italian
a la Clasul Codrian fondat de c
atre Gh. Rosca Codreanu.
Binefacerea si dorinta de a vedea patria mea pus
a pe o cale de fericire mai statornic
a
au fost, din frageda mea copil
arie si va fi, p
an
a n cel din urm
a minut al vietii mele,
singura ideie ce m
a va ndeletnici scrie N. Rosca Codreanu n testamentul sau. Caci:
averea, ranguri si m
ariri, toate se sterg si pier ca sclipirea fulgerului n nouri.
Imperativul economic (n primul rand) cerea dezvoltare pe toate planurile; Barladul
era la acea vreme un puternic centru comercial, cu oameni care ntelegeau c
a scoli
mai bune nseamn
a, finalmente, o viata mai buna.
La trecerea sa prin Barlad, n 1849, dupa ce asculta o alocutiune a profesorului Clasului Codrian, Iosif Popescu-Patriciu, domnitorul Grigore Ghica a promis
nfiintarea unui gimnaziu la Barlad. Promisiunea se va materializa destul de t
arziu,
gimnaziul a fost pornit abia la 1858 si a durat p
ana n 1864. In 1859 avea sa ia fiinta
si la Botosani un asez
am
ant asem
anator, care va deveni Colegiul National A. T.
Laurian. Exista referinte din acea vreme care arata c
a elevii acestor doua gimnazii
erau silitori si primeau pentru aceasta burse din partea Ministerului Instructiunii si
Cultelor. Merit
a sa observ
am c
a gimnaziul (ca si clasul, ca si liceul si colegiul care au
urmat) a fost unul din primele din tara.
In 1860, prin ordonant
a a domnitorului Alexandru Ioan Cuza se ncuviinteaz
a
ca Gimnaziul sa poarte numele Codreanu, al p
arintilor sai fondatori. Ca sa ne facem
o idee, iat
a ce se studia la matematica n cei patru ani ai gimnaziului: aritmetica n
clasa I, algebra n clasa a II-a, algebra si geometria plan
a ntr-a III-a, iar n clasa a IV-a
stereometria (geometria n spatiu) si trigonometria (inclusiv trigonometria sferic
a ).
Legea instructiunii publice din 1864 transforma gimnaziul n liceu (sapte
clase, spre deosebire de patru). Materia la matematica se continua n clasa a V-a:
repetitii generale cu aplicarea algebrei n geometrie, a VI-a: teoria functiunilor,
geometria analitic
a cu dou
a dimensiuni si a VII-a: geometria practic
a.
Din nou necesitatea reformarii si nnoirii se facea simtita: se dorea ca profesorii
sa nu mai predea mai multe discipline si ca elevii sa nu mai stea nghesuiti n clase,
n conditii precare. Doctorul Constantin Codrescu (membru corespondent al Societatii de Medicina din Paris, de al c
arui nume se leaga si construirea a ceea ce
avea sa fie mult
a vreme Spitalul de adulti din Barlad) a fost cel care a prezentat, n
1881, Camerei Deputatilor memoriul care solicita o cladire noua pentru liceu. Memoriul nu a fost aprobat initial, dar n cele din urma s-a trecut la constructia actualei
cladiri, n 1885. La 27 aprilie 1886 aceasta a fost inaugurat
a, n prezenta ministrului
nvatam
antului D. A. Sturza si a secretarului general Spiru Haret. Constructia a
costat 230000 de lei.
Dumitru Bagdasar, Nicolae Bagdasar, Constantin Hamangiu, Garabet Ibr
aileanu,
Vasile P
arvan, Alexandru Philippide, S
tefan Procopiu, Theodor Anghelut
a, Ioan Barb
alat
a, Vasile Cruceanu, Mihai Botez, Cezar Cosnit
a, Anton Davidoglu, Nicolae Donciu, Bogdan Ionescu, Emanoil Gaiu, George Teiler, Valeriu Alaci, Virgil Claudian,
Radu Miron, Gheorghe Luca, Dorian Spulber - iat
a o nsiruire a unei (foarte) mici p
arti
a multimii marilor oameni care au trecut prin Liceul Gheorghe Rosca Codreanu n
135

aceasta vreme (ca elevi sau profesori). (M-am rezumat, din pricina lipsei de spatiu,
doar la mentionarea numelor c
atorva academicieni si/sau profesori universitari din
domeniul matematicilor. Scriitorul Alexandru Vlahut
a si pictorul Nicolae Tonitza au
fost, de asemenea, elevi ai liceului. Monografia [1] mentioneaz
a - n 1971 - 24 de
academicieni si membri corespondenti ai Academiei, si cam doua sute de profesori
universitari, scriitori, oameni de art
a, generali, alte personalit
ati.) Se vede dar c
a n
1946, cu ocazia centenarului, liceul si-a primit pe merit numele si titlul de Colegiul
National Gheorghe Rosca Codreanu.
Din p
acate nu pentru mult timp. Tav
alugul comunismului a maturat totul n
calea sa: nu numai numele, dar si statutul scolii a fost radical schimbat. Intre 1948
si 1989 scoala aceasta (cu o traditie de necontestat) a fost pe rand: Liceul de b
aieti
p
ana n 1952, S
coala medie de b
aieti nr. 2 ntre 1954-1956, S
coala medie mixt
a nr. 1
ntre 1956-1958, S
coala general
a de 7 ani nr. 1 din 1958 p
ana n 1962 (perioada n
care toate scolile medii din oras au fost unificate sub titulatura Complexul scolar,
probabil prin analogie cu complexele agrozootehnice). In fine, din 1962 redevine
S
coala medie nr. 2, n 1965 d
and iar promotie de liceu. Prin anii 80, c
and eram
elev al scolii, se numea Liceul de matematic
a-fizic
a Gheorghe Rosca Codreanu. Din
1996 s-a revenit la denumirea deplin meritata, dobandita n 1946, de Colegiul National
Gheorghe Rosca Codreanu.
[...] Singure numai virtutea si binefacerea sunt netrec
atoare, c
aci ele ne urmeaz
a
si dincolo de morm
ant. [...] Singura mea pl
acere era de a vorbi adeseaori cu oamenii
nv
atati si experienti; a le face mii de ntreb
ari asupra acelor lucruri pe care, din
nenorocirea mea si nt
ampl
arile venite asupra casei p
arintesti nu le cunosteam defel;
c
aci, amar omului care tr
aind n lume nu cunoaste ceea ce ar trebui s
a cunoasc
a
fieste care om, si vai de acei p
arinti ce nu se ngrijesc de buna crestere a copiilor
lor. [...] Adeseori c
and auzeam vorbindu-se de popoare si drepturile omenirii, despre
institutiile si civilizatia t
arilor str
aine, g
andeam la nenorocita stare a natiei mele,
dorind mbun
at
atirea soartei ei. Ideile acestea renviindu-mi n
adejdea c
a, poate, n
cur
and si Moldova s
a se bucure de o soart
a mai ferice, m
a f
acur
a a m
a ocupa cu
mai mult
a seriozitate despre planul ce de mult timp hr
aneam n inima mea, de-a-mi
regularisi averea n folosul obstesc al natiei mele.
Am citat iarasi din testamentul lui Neculai Rosca Codreanu, avand doar regretul de a nu putea cuprinde n acest colt de revist
a mai mult din ideile si de a nu putea
surprinde mai mult din sufletul celor doi frati atat de generos ntemeietori ai unei
scoli uimitoare prin vechime, traditie si calitatea celor ce s-au perindat n timp prin
spatiul sau cultural. Pe toti acestia am face bine sa nu-i uitam niciodata; ncepand,
desigur, cu Gheorghe si Neculai Rosca Codreanu.
Bibliografie
1. T. Nicola - Liceul Gheorghe Rosca Codreanu B
arlad. Monografie, Iasi, 1971.

Prof. Marian TETIVA


Colegiul National Gheorghe Rosca Codreanu
136

Concursul de matematic
a Al. Myller
Editia a IX-a, Iasi, 2 aprilie 2011
Clasa a VII-a
Problema 1. Fie p un num
ar natural prim.
arul perechilor
p Determinati num
(x, y) de numere naturale care verific
a egalitatea x2 + p4 = y.
Problema 2. Fie multimile

1
,
4

1
.
B = (x; y) | 0 < x y < 1; x + y 1; xy
4

A = (x; y) | 0 < x y < 1; x + y 1; xy

a) S
a se arate c
a multimea A are cel putin 2011 elemente.
b) S
a se determine multimea B.
Problema 3. a) Fie ABCD un trapez cu bazele [AB] si [CD], n care {O} =
AC BD si {Q} = AD BC. Demonstrati c
a dreapta OQ contine mijloacele bazelor
trapezului.
b) Se consider
a triunghiul dreptunghic ABC cu m(BAC) = 90 si AB < AC.
Punctele M si N sunt situate pe laturile [AB] si respectiv [AC] astfel nc
at AM AB =
AN AC. Paralela prin punctul M la dreapta BC intersecteaz
a [AC] n punctul P .
Dac
a {O} = BP CM , ar
atati c
a AOM N .
Problema 4. In interiorul unui p
atrat de latur
a 1 se afla un patrulater con1
vex de arie . Demonstrati c
a exista o dreapt
a d paralel
a cu una dintre laturile
2
p
atratului, care intersecteaz
a patrulaterul si determina cu laturile acestuia un seg1
ment cu lungimea mai mare sau egala cu .
2

Clasa a VIII-a
Problema 1. Determinati numerele reale a cu proprietatea c
a

a2 + 2a 3

+ a2 2a 15

= 8 a2 9

Problema 2. Se consider
a p
atratul ABCD de latur
a 1. Punctele M, N, P si
Q sunt situate pe laturile (AB), (BC), (CD) si respectiv (DA).
Demonstrati c
a

perimetrul patrulaterului M N P Q este mai mare sau egal cu 2 2.


Problema 3. Fie x un num
ar irational. Se stie c
a 36 este cel mai mic numar
natural nenul cu proprietatea c
a num
arul x36 este rational. Determinati numarul de
elemente rationale din multimea A = {xab | a + b = 36, a, b N\{0}}.
137

Problema 4. Fie ABCA B C o prisma triunghiulara dreapt


a. Aratati c
a, daca

dreptele A B,
B
C

s
i
C
A
sunt
perpendiculare
dou
a
c
a
te
dou
a
,
atunci
AB
=
BC =

CA = AA 2.

Clasa a IX-a
Problema 1. Fie K, L, M, N mijloacele laturilor (AB), (BC), (CD), respectiv
(DA) ale patrulaterului ABCD nscris ntr-un cerc de centru O. Not
a m HA , HB , HC ,
HD , ortocentrele triunghiurilor AKN , BLK, CM L, respectiv DN M .


a) Ar
atati c
a 2OHA = 2OA + OB + OD.
b) Ar
atati c
a HA HB HC HD este paralelogram.
Problema 2. Determinati numerele ntregi a, b pentru care sistemul

x2 2ax a 2 = 0
y 2 2by x = 0

are exact trei solutii n multimea numerelor reale.


Problema 3. Fie (an )nN un sir de numere reale definit prin a0 (0, 1) si
8
<

an+1 =

, daca an = 0

an

, daca an 6= 0

nn + 1o

Ar
atati c
a a0 este irational daca si numai daca sirul nu are termeni nuli.
Problema 4. Determinati toate functiile f : R R care au proprietatea c
a
(x2 + xy + y 2 )(f (x) f (y)) = f (x3 ) f (y 3 ), pentru orice x, y R.

Clasa a X-a
Problema 1. S
a se determine numerele complexe z cu proprietatea c
a |z| + |z
25| + |z 18 24i| + |z + 7 24i| = 70.

Problema 2. Consider
am un punct M n interiorul paralelogramului ABCD. S
a
se arate c
a M A M C + M B M D > AB AD.
Problema 3. Un sir de numere reale (an )n>1 se numeste convex daca
2an 6 an1 + an+1 , oricare ar fi n N, n > 2.

a) S
a se dea un exemplu de sir convex neconstant.
b) Fie (bn )n>1 un sir de numere reale pozitive astfel nc
at, oricare ar fi a > 0, sirul
(an bn )n>1 sa fie convex. S
a se demonstreze c
a sirul (ln bn )n>1 este convex.
Problema 4. S
a se determine functiile f : N N care verific
a relatia f (f (n)) +
f (n) = 6n, oricare ar fi n N.

138

Clasa a XI-a
Problema 1. Fie n un num
ar natural nenul si fie A o matrice p
atratic
a de ordin
n cu elemente ntregi, avand proprietatea c
a In + A + A2 + + A10 = On .
i) S
a se arate c
a matricea In + A + A2 este inversabil
a.
ii) S
a se arate c
a det(In + A + A2 ) = 1.
Problema 2. Consider
am un sir (an )n1 de numere reale, strict descrescator si
an + an+2
convergent la 0, cu proprietatea c
a an+1 6
. S
a se arate c
a lim n(an+1
n
2
an ) = 0.
Problema 3. Fie n un num
ar natural nenul. S
a se arate c
a exista numerele reale
x1 , x2 , . . . , xn astfel nc
at matricea

An = (sin pxq )16p,q6n =

sin x1
sin x2
..
.

sin 2x1
sin 2x2
..
.

sin 3x1
sin 3x2
..
.

...
...
..
.

sin nx1
sin nx2
..
.

sin xn

sin 2xn

sin 3xn

. . . sin nxn

sa fie nesingular
a.
Problema 4. Fie f : [0, ) R o functie cresc
atoare si convex
a, cu proprietatea
c
a lim (f (x) x) = 0. S
a se demonstreze c
a pentru orice numar x [0, ) avem
x

2x 6 f (f (2x)) 6 2f (x).

Clasa a XII-a
Problema 1. Fie I R un interval si fie f : I Z
R o functie integrabil
a pe

orice interval [a, b] I. S


a se arate c
a multimea A =

f (t)dt | x, y I

este un

interval, eventual degenerat.


Problema 2. Fie p un num
ar prim, p > 2. S
a se arate c
a polinomul cu coeficienti
ntregi f = (X 1)(X 2) (X p) + X + p este ireductibil n Z[X].
Problema 3. Se consider
a functia continu
a f : [0, 1] R cu propriet
atile:
a) f (0) < 0 < f (1);
b) exista un unic num
ar c (0, 1) pentru care f (c) = 0;
Z

c)

f (x)dx >
0

xf (x)dx.
Z

S
a se demonstreze c
a pentru orice numar n natural nenul avem

xn f (x)dx > 0.

Problema 4. Fie A un inel finit cu propriet


atile:
i) orice divizor al lui zero este element nilpotent;
ii) num
arul elementelor inversabile din A si numarul automorfismelor inelului A
sunt relativ prime.
S
a se arate c
a:
1 Pentru orice element inversabil a A, functia fa : A A, fa (x) = axa1 este
un automorfism al inelului A;
2 Inelul A este comutativ.
139

Concursul de matematic
a Florica T. C
ampan
Editia a XI-a, Iasi, 2011
Etapa judetean
a, 20 februarie 2010
Clasa I
1. G
asiti regula si completati cu numerele care lipsesc:
10

a)
0;
2;
b) 4;
;
;

2;
4;
6;
;
;

4;
6;
8;
;
.

c) 1, 2, 4, 3; 5, 6, 8, 7; , , , .
2. Bogdan are cu 3 timbre mai mult decat Radu, iar Radu are cu 10 mai putine
dec
at Mihai, care are 20 de timbre. Cate timbre are Bogdan?
3. C
atalin se joac
a n scara blocului astfel: urc
a 3 trepte, coboar
a doua, urc
a apoi
4 trepte, coboar
a una, urc
a 5 trepte si coboar
a doua. Cati pasi a facut Catalin si c
ate
trepte a urcat?

Clasa a II-a
1. a) Completati n toate modurile posibile dreptunghiul alaturat cu numere
diferite de 0, astfel nc
at suma numerelor pe fiecare linie si pe fiecare
3
coloan
a sa fie egala cu num
arul indicat n dreptul liniei, respectiv coloanei.
5
b) Intr-o scoal
a, elevii intra pe baza unui cod format din trei cifre
6
nenule distincte, f
ar
a a conta ordinea n care sunt introduse cifrele. Intruna din zile, Claudiu uit
a cele trei cifre care i trebuie pentru a intra n
4 10
scoal
a, ns
a tine minte c
a suma celor trei cifre este 12. Aflati toate valorile posibile
pe care le poate avea codul de acces al lui Claudiu n scoal
a.
2. a) Dup
a ce Diana a mancat 6 mere, iar Elena a mancat 7 mere, le-au mai
ramas fiec
areia atatea mere c
ate au mancat la un loc. Cate mere a avut fiecare?
b) Rares a desenat 24 de figuri geometrice, p
atrate si cercuri, astfel nc
at fiecare
p
atrat sa contina trei cercuri. Stiind c
a n fiecare cerc deseneaza c
ate un trandafir si
n fiecare p
atrat c
ate doua garoafe, aflati c
ate flori a desenat Rares.
3. Vl
adut arunca sase sageti la tinta din figura alaturat
a si nimereste de fiecare
140

data ntr-unul din cercurile ei, c


astig
and atatea puncte cate sunt
scrise n fiecare cerc.
a) Care este cel mai mic scor pe care l poate obtine Vladut?
b) Care este cel mai mare scor pe care l poate obtine
Vladut?
c) Indicati toate modurile posibile prin care poate obtine
scorul 10.

Clasa a III-a
1. a) Aflati num
arul de doua cifre c
aruia i apartine replica: Diferenta dintre
prima si a doua mea cifra este 1 iar suma lor este
9. Justificati!
b) La petrecerile din Imp
ar
atia Numerelor iau
parte doar numerele majore, adica cele mai mari
decat 50. Cifra 7 doreste insistent sa participe la
petrecere; Imp
aratul Zero i d
a o sansa si o supune
unei probe: In circuitul din figura alaturat
a trebuie sa pornesti de la START, sa efectuezi c
ateva
dintre operatii, n ordinea ar
atata de sageti si sa
obtii un rezultat mai mare de 50. Doar astfel te
voi primi la petrecere! Astfel, cifra 7 intra n circuit, alege un drum si obtine la
STOP rezultatul 55. A c
astigat! Puteti voi g
asi la ce operatii a fost supusa cifra 7
c
and a parcurs circuitul?
2. a) La o masa p
atrat
a pot sta exact 4 copii, c
ate unul pe fiecare latur
a. La o
serbare s-au asezat sapte mese, lipite una langa alta, astfel nc
at sa formeze o masa
lung
a, dreptunghiular
a. C
ati copii se pot aseza la masa formata?
b) Copiii din familiile Apostol si Balan s-au nt
alnit ntr-o cofetarie, pentru a
sarb
atori o aniversare. Este posibil ca fiecare b
aiat sa aib
a exact trei frati si fiecare
fata sa aib
a exact o sora n acea cofetarie? Cati copii ar fi atunci, de toti? (Fiecare
familie are drept copii si b
aieti si fete.)
3. Bancomatul jucarie din scoala mea mi permite sa extrag sume doar n
jetoane de 2 euro si de 5 euro. Pot scoate cel mult 10 jetoane de 2 euro si cel mult 5
jetoane de 5 euro.
a) Care este suma cea mai mare pe care o pot extrage din bancomat?
b) Descrieti un mod de a extrage exact 17 euro.
c) Este posibil sa scot din bancomat, dintr-o singura extragere, exact 42 euro?

Clasa a IV-a
1. Mos Martin avea n toamna anului 2010 o greutate de 350 de kilograme. Stiind
c
a vara se ngrasa cu 20 kg, iar iarna sl
abeste cu 10 kg, aflati ce greutate avea Mos
Martin n toamna anului 2007.
Doina Nechifor
2. Monograma unei persoane este reprezentat
a de trei litere: initiala numelui
de familie, initiala primului prenume si initiala celui de al doilea prenume (literele
141

alfabetului se consider
a a fi A, B, C, D, E, F, G, H, I, J, K, L, M, N, O, P, Q, R, S, T, U,
V, W, X, Y, Z). Domnul si doamna Campan doreau sa puna un nume bebelusului
C
ampan astfel nc
at literele din monograma lui sa fie diferite si n ordine alfabetica.
C
ate astfel de monograme exista?
Alexandru Negrescu
3. Pe planeta Nintendo, pokemonii sunt de patru feluri: de apa, de p
am
ant, de
foc si de noapte. Ei pot avea de la 5 la 7 aripi si de la 4 la 21 antene. Imparatul
lor, Lucian-Georges, vrea sa porneasca un razboi mpotriva lui Katalin, cumplitul
sau inamic. Care este numarul minim de pokemoni din armata ce ataca mparatia
lui Katalin, dac
a Lucian-Georges vrea sa aib
a certitudinea c
a va putea selecta un
comando format din 21 de pokemoni identici? (Doi pokemoni se consider
a identici
dac
a sunt de acelasi fel, au acelasi numar de aripi si acelasi numar de antene.)

Clasa a V-a
1. C
atalin si Doru au fost n vacanta n Egipt. In ziua n care au sosit napoi
ei s-au mboln
avit de gripa noua (AH1N1), contamin
and multe dintre persoanele cu
care au intrat n contact. In fiecare zi, numarul oamenilor care s-au mboln
avit este
de trei ori mai mare dec
at n ziua precedent
a.
a) C
ati oameni se vor mboln
avi n primele 100 de zile?
b) Ar
atati c
a num
arul total al bolnavilor din primele 100 de zile este mai mare ca
2150 .
Ciprian Baghiu
2. La o lucrare de control, cei 29 de elevi ai unei clase primesc spre rezolvare un
set de trei ntrebari. Dac
a raspund corect la ntrebarea I primesc 1 punct, pentru a
II-a ntrebare primesc 2 puncte, pentru a III-a ntrebare primesc 3 puncte, iar daca
nu dau niciun raspuns corect primesc 0 puncte.
a) Ar
atati c
a cel putin cinci elevi au obtinut acelasi punctaj.
b) Se p
astreaz
a concluzia daca n acea clasa sunt 28 de elevi?
C
at
alin Budeanu
3. Suma a 30 de numere naturale pare si nenule este 328.
a) Dati un exemplu de numere ce ndeplinesc conditiile de mai sus, printre care sa
existe exact o grup
a cu patru termeni egali.
b) Ar
atati c
a oricum am alege numere care satisfac toate conditiile problemei, vom
avea cel putin patru numere egale ntre ele.
Ciprian Baghiu

Clasa a VI-a
1. P
acala si T
andala au primit bonuri valorice de 11 lei bucata. Intr-un supermarket P
acala a cumparat doua p
aini, a mancat 7 c
arnciori si a b
aut un pahar cu must
iar T
andala a cumparat 3 p
aini, a mancat 5 c
arn
aciori si a b
aut 7 pahare cu must.
P
acala a platit cu un num
ar ntreg de bonuri valorice, fara a primi rest. Aratati c
a si
T
andala poate achita plata total
a la fel.

142

2. Ana deseneaz
a pe tabla figura alaturat
a. Maria alege zece numere din
multimea {0; 1; 2; . . . ; 14} si le scrie n cercurile din desen. Luiza
scrie pe fiecare segment din desen diferenta numerelor din cercurile
pe care acesta le uneste (diferentele sunt numere naturale). Este
posibil ca numerele de pe segmente sa fie distincte? Justificati!
Andrei Nedelcu
3. Elevii Andrei, Bogdan si Costel joac
a ping-pong. Cel care
pierde un set lasa loc la masa celui care s-a odihnit. In final Andrei
a jucat 13 seturi iar Bogdan 27. C
ate seturi a jucat Costel?
Gheorghe Iurea

Clasa a VII-a
1. La un club sportiv sunt nscrisi mai putin de 70 de elevi. O treime din numarul
fetelor reprezint
a un sfert din num
arul b
aietilor. Unii copii joac
a volei, ceilalti joac
a
handbal. Un sfert din num
arul celor care joac
a volei reprezinta o cincime din numarul
celor care joac
a handbal. S
tiind c
a 17 fete joac
a handbal, aflati c
ati b
aieti joac
a volei.
Gazeta Matematic
a 11/2010
2. Despre un num
ar ntreg a vom spune c
a are valenta n daca exista exact n
triplete de numere ntregi (x, y, z) astfel nc
at 5 x 10, 5 y 10, 5 z 10
si x 2y + 3z = a. Determinati valentele numerelor 50, 50 si 0.
Claudiu S
tefan Popa si Gabriel Popa
3. Un copil de clasa a VII-a g
aseste un document ng
albenit de vreme care arata
locul unde a fost ngropat
a o comoar
a pe o insul
a: Caut
a turnul bisericii T , cascada
C, stejarul b
atr
an S si st
anca diavolului D. Infige un tarus M la mijlocul drumului
drept dintre T si D si nc
a unul N la mijlocul drumului drept dintre C si S. Uneste
prin linii drepte M cu N si T cu S, marc
and locul n care aceste linii se nt
alnesc (E).
Mergi de la M la E, num
ar
andu-ti pasii, apoi numara tot atatia pasi n prelungirea
liniei M N , ncepand din N , si nfige un tarus n locul P n care ai ajuns. Comoara
se afla acolo unde linia dreapt
a prin C si D nt
alneste linia dreapt
a prin T si P .
Ajuns pe insul
a, copilul afla de la b
astinasi c
a stejarul b
atr
an a fost doborat de un
trasnet, cu multi ani n urma. G
aseste ns
a o harta veche pe care se vede c
a TCSD
era un trapez cu baza mare T C si DS = SO, unde {O} = ST CD.
a) Ar
atati c
a ST = DS + T C.
b) Demonstrati c
a unghiul SP T este drept.
c) Ajutati-l pe elev sa g
aseasca locul comorii, chiar n absenta stejarului b
atr
an!
Claudiu S
tefan Popa si Gabriel Popa

Clasa a VIII-a
1. Ali-Baba si ai lui 40 de hoti au strans de-a lungul timpului o suma imensa de
a Ali-Baba se temea de hoti, a pus suma ntr-un seif la
bani: abcde256 euro. Pentru c
o banc
a n Elvetia. Un agent sub acoperire, infiltrat de foarte mult
a vreme n banda
lui Ali-Baba, era cunoscut ca rapper de succes cu numele M-One. Acesta a aflat n
scurt timp combinatia de la seif si a scris urmatorul e-mail c
atre colegii din politie:
143

cde < 256


In fiecare c
asut
a este un p
atrat perfect si numarul din fiecare c
asuta este
suma numerelor peste care se afla

La varf este codul:


Sunt p
atrat de p
atrat
Natural, adevarat
Cel mai mare din o mie
Toat
a lumea ma stie.
Cei din politie nu s-au descurcat cu mesajul si l-au trimis la Concursul Florica
T. C
ampan, sper
and ca unul dintre elevi sa dezlege enigma. Aflati pentru ei codul
seifului si suma din seif.
Marian Pantiruc

2. In varfurile unui cub se aseaza numere naturale si pe fiecare muchie se aseaza


media aritmetica a numerelor din capete, de asemenea numar natural.
a) Pot fi cele 20 de numere pare si distincte doua c
ate doua?
b) Pot fi cele 20 de numere impare si distincte doua c
ate doua?
c) Demonstrati c
a nu putem aseza n acest fel 20 de numere consecutive.
Julieta Grigoras
3. Un chiosc de ziare are forma unei prisme patrulatere regulate DIFUZARE,
cu latura bazei de 2m si naltimea de 3m. Chioscul este
alimentat cu electricitate printr-un fir ce uneste varful
V al unui st
alp V O nalt de 4m cu mijlocul laturii DU
(evident, f
ar
a a strapunge chioscul). Stiind c
a punctele
D, I, O sunt coliniare si IO = 4m, aflati lungimea celui
mai scurt fir care poate alimenta chioscul cu energie electric
a.
Marian Pantiruc

Etapa interjudetean
a, 26 martie 2011
Clasa a IV-a
1. In timpul vacantei, Claudiu si ajuta p
arintii lucrand la magazinul familiei.
Intr-una din zile, la magazin se aduc cutii de compot care au naltimea de 10 centimetri. Claudiu trebuie sa le aranjeze pe o masa unele peste altele: pe primul rand
de jos pune 12 cutii, pe randul al doilea 11 cutii, pe urmatorul rand 10 cutii si asa
mai departe.
144

a) De c
ate cutii de compot are nevoie Claudiu pentru aranjamentul de pe masa?
Ce naltime va avea acest aranjament?
b) Pentru o alt
a aranjare a cutiilor, Claudiu se hotaraste sa puna 12 cutii pe
primul rand de jos, 10 cutii pe al doilea rand si asa mai departe, p
ana ce pune doua
cutii pe ultimul rand de sus. De c
ate cutii are nevoie Claudiu? Ce naltime va avea
acest nou aranjament?
2. In p
atratul al
aturat, produsul numerelor de pe fiecare linie, de pe fiecare
coloan
a si de pe fiecare dintre cele doua diagonale este acelasi si nenul.
2 a b
a) Aflati b.
9 6 c
b) Dac
a, n plus, 9f + 2d = 144, aflati si celelalte numere.
d e f
3. Un num
ar care nu se mparte exact la niciuna din cifrele sale se numeste num
ar
civilizat (preciz
am c
a niciun num
ar nu se mparte la 0).
2 3
a) Ar
atati c
a numerele 52 si 354 nu sunt civilizate.
9
b) Claudiu si Diana au g
asit doua numere civilizate care

nmultite dau ca rezultat tot un numar civilizat.
Reconstituiti nmultirea g
asit
a de cei doi copii (stelutele nlocuiesc cifre).

Clasa a V-a
1. O carte ciudat
a are paginile numerotate astfel: 16, 23, 30, 37, 44, . . ., 2011.
a) Aflati c
ate foi are cartea ciudat
a.
b) Determinati c
ate cifre s-au folosit pentru numerotarea paginilor c
artii ciudate.
c) Calculati suma numerelor nscrise pe foaia din mijlocul c
artii.
2. Monica este o nvat
atoare foarte serioasa si iubita de copiii pe care i nvata. In
clasa la care pred
a are 10 copii pregatiti pentru concursuri, dintre care 7 la matematica
si 6 la limba romana. La sfarsitul unei saptam
ani au loc trei concursuri: samb
ata
unul de matematica si unul de limba romana (la aceeasi ora), iar duminica unul de
matematica. La fiecare concurs ea trimite c
ate un singur copil. In c
ate moduri poate
alege participantii la cele trei concursuri?
Ciprian Baghiu
3. a) Ar
atati c
a num
arul 1006009 este p
atrat perfect.
b) Demonstrati c
a exista macar 2011 p
atrate perfecte care nu au ultima cifra egala
cu 0 si care contin un num
ar impar de cifre de 0.
atrat
. . . 00 }b nu este p
c) Ar
atati c
a, oricare ar fi cifra b nenula, numarul b| 000 {z
2011 cifre de 0

perfect.

Cristian Laz
ar

Clasa a VI-a

1. Construim succesiv dreptunghiuri vecine,


alternativ spre dreapta si n jos, ca n figura
alaturat
a. (Doua dreptunghiuri se numesc vecine
daca au o latur
a comun
a.) Laturile dreptunghi1 1 1 1 1
urilor au consecutiv lungimile , , , , , . . ..
1 2 3 4 5
145

1/3

1 /2
1/5
1/4
1/6 ...

Ar
atati c
a exista o grup
a de doua sau mai multe dreptunghiuri vecine doua c
ate
1
doua astfel nc
at suma ariilor acestora sa fie .
3
2. Pe o tabla de sah 8 8 se aseaza 8 turnuri astfel nc
at niciunul sa nu le atace
pe celelalte.
a) Ar
atati c
a un astfel de aranjament este posibil.
b) Pentru un astfel de aranjament, aratati c
a n orice p
atrat 5 5 exista cel putin
doua turnuri.
3. Demonstrati c
a pot fi alese cel mult 671 numere din multimea A = {1, 2, . . . , 2011}
astfel nc
at diferenta oric
aror doua numere alese sa nu divid
a suma acestora.

Clasa a VII-a
1. Ioana deseneaz
a pe monitorul calculatorului, cu ajutorul unui program de
grafic
a pe computer, un triunghi isoscel ABC si ia punctele D pe baza (BC) si E pe
latura (AB) astfel nc
at ADE ACB. Programul folosit i permite Ioanei sa selecteze orice triunghi care apare n desen, sa-l mareasca sau sa-l micsoreze (pastrandu-i
forma) cu functia zoom, sa-i schimbe pozitia sau sa-l roteasc
a, dupa dorinta. Numim
transformare o succesiune oarecare de astfel de operatii.
a) Demonstrati c
a Ioana poate g
asi o transformare n urma c
areia triunghiul DBE
sa se suprapuna exact peste triunghiul ACD.
b) Observand alte transformari urmate de suprapuneri de triunghiuri n configuratia
desenata, Ioana redescopera relatia lui Stewart pentru triunghiul isoscel: AB 2 =
AD2 + BD CD. Demonstrati si voi aceasta relatie!
Claudiu S
tefan Popa
c
a
2. Se consider
a numerele rationale strict pozitive a, b, c, d astfel nc
at = .
b
d

a) Ar
atati c
a ac Q daca si numai daca bd Q.

b) Demonstrati c
a (a + b)(c + d) Q daca si numai daca ac Q.

c) Este adevarata implicatia (a + b)(c + d) N ac N? Dar reciproca?


Claudiu S
tefan Popa si Gabriel Popa
3. Consider
am multimea M = {1, ; 2; 3; . . . ; 2011}. Pentru fiecare submultime nevid
a A a lui M, calculam produsul PA al tuturor elementelor sale; daca
A = {a}, atunci PA = a.
a) G
asiti trei submultimi distincte A, B si C ale lui M, pentru care PA = PB = PC .
b) Care este valoarea cea mai mare pe care o poate lua un astfel de produs? Dar
cea mai mica?
c) Calculati suma tuturor produselor care se obtin. (Daca valoarea unui produs
se repeta pentru mai multe submultimi ale lui M, respectiva valoare se va repeta de
acelasi num
ar de ori si n suma.)

Clasa a VIII-a
1. Baronul de M
unchhausen spune o obisnuit
a poveste gogonata despre cum a
zburat el pe Luna c
alare pe o ghiulea trasa dintr-un tun. Oricine si-ar putea da seama
146

c
a este o uriasa minciun
a, ns
a putini puteau si demonstra acest lucru. In cele din
urma, un copil istet de vreo 15 ani a reusit sa afle, studiind diverse specificatii tehnice,
c
a viteza unei ghiulele ar fi de 200m/s si, conform legilor fizicii, dupa t > 0 secunde
de miscare, ghiuleaua se afla la naltimea h(t) = 5t2 + 200t. Aflati si voi naltimea
maxim
a la care ar fi putut ajunge baronul M
unchhausen si timpul total al c
alatoriei
sale c
alare pe ghiulea.
Marian Pantiruc
2. Fie prisma triunghiular
a regulata A1 A2 A3 B1 B2 B3 avand toate muchiile, precum si diagonalele fetelor laterale, colorate cu rosu sau cu albastru. In fiecare triunghi
care se formeaz
a, exceptand bazele, exista atat o latur
a rosie c
at si o latur
a albastr
a.
Aratati c
a cele sase muchii ale bazelor sunt colorate toate cu aceeasi culoare.
Gheorghe Iurea
3. Lucian si C
atalin au vandut mpreuna, la piata, n kilograme carne de curcan,
cu valoarea de n euro kilogramul. Ei actioneaz
a succesiv: primul si laud
a marfa
Lucian (pana vinde carne n valoare de 10 euro), apoi Catalin (pana vinde si el n
valoare de 10 euro), din nou Lucian etc. Catalin c
astiga la ultima strigare o suma
ntreag
a, mai mica de 10 euro. Determinati acest
a suma.
Doru Buzac si Gabriel Mrsanu

Nume de rom
ani date unor corpuri ceresti
sau unor forme de relief pe acestea
Denumirea corpurilor ceresti (stele, planete, asteroizi, comete etc.) si a oricarei
forme de relief pe acestea este decis
a de Uniunea Astronomic
a International
a
(UAI), cu sediul la Paris. In timp ce cometele poarta numele descoperitorilor lor,
corpurile si formele de relief poarta numele unor personalit
ati din lumea artelor si
stiintei.
1. Nume rom
anesti pe planete
Craterul Haret (Spiru Haret, 1851-1912) pe fata invizibila a Lunii, cu
diametrul de 29,77 km;
Craterul Eminescu pe Mercur, cu diametrul de 125 km;
Craterul V
ac
arescu (Elena V
ac
arescu, 1866-1947) pe Venus, cu diametrul de 315,5 km;
Patera Darcl
ee (Hariclea Darclee, 1860-1939) pe Venus, formatiune vulcanic
a numit
a patera cu diametrul de 15 km;

(continuare la pag. 75)


147

Solutiile problemelor propuse n nr. 2/2010


Clasele primare
P.196. Mioara aranjeaz
a patru m
argele, dou
a albe si dou
a galbene, una l
ang
a alta,
pe o at
a.
In c
ate feluri poate aranja Mioara m
argelele?
(Clasa I)
Inst. Maria Racu, Iasi
Solutie. Mioara poate aranja margelele n sase moduri: AAGG, GGAA, GAGA,
AGAG, AGGA, GAAG.
P.197. Dan mparte o ciocolat
a astfel: o linie ntreag
a de 4 p
atr
atele pentru
fratele s
au, o coloan
a ntreag
a de 4 p
atr
atele pentru sora sa, iar restul pentru sine.
C
ate p
atr
atele de ciocolat
a i-au revenit lui Dan?
(Clasa I)
Maria Ursu, elev
a, Iasi
Solutie. Initial ciocolata avea 4 coloane si 5 linii. Dan a ramas cu 3+3+3+3 = 12
p
atr
atele de ciocolat
a.
P.198. Num
arati figurile geometrice din desenul al
aturat si scrieti:
a) num
arul triunghiurilor;
b) num
arul figurilor geometrice care sunt p
atrate sau dreptunghiuri.
(Clasa a II-a)
Andreea Amarandei, student
a, Iasi
Solutie. a) 5 triunghiuri; b) 5 p
atrate si 4 dreptunghiuri formeaza 9 figuri geometrice care sunt p
atrate sau dreptunghiuri.
P.199. Dac
a a 15 = 15 b, poate fi diferenta a b un num
ar impar?
(Clasa a II-a)
Andreea Bzdg
a, elev
a, Iasi
Solutie. Din a 15 = 15 b se obtine a + b = 30, deci a b este numar par, c
aci
suma si diferenta a doua numere naturale au aceeasi paritate.
P.200. Dintr-un num
ar de 12 bile, 11 au mase egale, iar una are o mas
a mai
mare dec
at celelalte. Care este cel mai mic num
ar de c
ant
ariri prin care se poate
depista bila cu masa mai mare, dac
a avem la ndem
an
a o balant
a cu dou
a talere?
(Clasa a III-a)
Ana Cojocaru, Iasi
Solutie. Formam trei grupe: 5b, 5b, 2b. Asez
am c
ate 5 bile pe fiecare taler.
Dup
a prima c
ant
arire determinam grupa care contine bila mai grea. In continuare
mai avem nevoie de maximum doua c
ant
ariri pentru a determina bila cea mai grea.
In concluzie, num
arul cel mai mic de c
ant
ariri este 3.
P.201. Diferenta dintre suma v
arstelor a dou
a persoane si diferenta lor este 20
de ani. Triplul sumei v
arstelor este egal cu de 7 ori diferenta v
arstelor. Care este
v
arsta fiec
arei persoane?
(Clasa a III-a)
Valeria Avaslcei, Iasi
Solutie. Din (a + b) (a b) = 20 se obtine b + b = 20, deci b = 10. Utilizand
a doua informatie g
asim 3(a + 10) = 7(a 10), de unde 3a + 30 = 7a 70, asadar
30 = 4a 70 si a = 25. V
arstele sunt: 25 ani, 10 ani.
148

P.202. Aflati numerele ab si c, cu b 6= 9 si c 6= 0, astfel nc


at, dac
a m
arim cifrele
a si b cu c
ate o unitate, atunci produsul ab c se dubleaz
a.
(Clasa a III-a)
Amalia Munteanu, elev
a, Iasi
Solutie. Dac
a cifrele a si b se maresc cu c
ate o unitate, atunci numarul ab devine
ab + 11 si conditia problemei ne d
a (ab + 11)c = 2 ab c, de unde 11 c = ab c si
obtinem ab = 11, c = 1, 2, 3, , 9.
P.203. Cei doi tigri de la Zoo au suficient
a hran
a pentru 3 s
apt
am
ani. Au mai
fost adusi, ns
a, nc
a 5 tigri. Dac
a fiecare m
an
anc
a aceeasi cantitate de hran
a pe zi,
c
ate zile le va mai ajunge acum hrana?
(Clasa a IV-a)
Inst. Laura Chiril
a, Iasi
Solutie. Dac
a 2 tigri au suficient
a hran
a 3 saptam
ani, atunci un singur tigru are
suficient
a hran
a 6 sapt
am
ani, adica 42 zile. 7 tigri vor avea suficient
a hran
a numai
42 : 7 = 6 zile.
P.204. Aflati toate numerele naturale n astfel nc
at triplul predecesorului lui n nu
dep
aseste dublul succesorului lui n.
(Clasa a IV-a)
Andreea Simion, elev
a, Iasi
Solutie. Trebuie sa avem 3(n1) 2(n+1), adica 3n31 2n+21,
ceea ce este echivalent cu n 5 si obtinem n = 1, 2, 3, 4, 5.
P.205. Fie S suma a zece numere naturale nenule.
a) Dac
a S = 54, ar
atati c
a exist
a cel putin dou
a numere egale.
b) Aflati zece numere naturale nenule distincte pentru care S = 57. C
ate posibilit
ati
sunt? Justificati!
(Clasa a IV-a)
Constanta Tudorache si Nelu Tudorache
Solutie. a) Suma celor mai mici zece numere naturale nenule distincte este 1 +
2 + 3 + + 9 + 10 = 55. Micsorand cu o unitate unul dintre termeni, vom obtine
doua numere egale.
b) In suma 1 + 2 + 3 + + 9 + 10 = 55 putem sa nlocuim pe 9 cu 11 sau pe 10
cu 12 si obtinem 10 numere naturale nenule si distincte care au suma 57. Avem doua
posibilitati.

Clasa a V-a
V.123. Un spiridus i sopteste Ioanei: Pentru a salva lumea de r
au, prepar
ao
potiune din praf de aur si praf de stele.
In camera albastr
a este un dulap urias, cu
numeroase rafturi, fiecare contin
and c
ate 20 de sticlute numerotate: de la 1 la 20 pe
primul raft, de la 21 la 40 pe al doilea etc. Num
arul raftului cu sticluta cu praf de aur
este egal cu num
arul sticlutei cu praf de stele. Suma dintre num
arul sticlutei cu praf
de aur si num
arul sticlutei cu praf de stele este 243. Care sunt numerele sticlutelor
pe care trebuie s
a le aleag
a Ioana?
Cristina Timofte, Iasi
Solutie. Fie x num
arul sticlutei cu praf de stele; numarul sticlutei cu praf de aur
va fi 20(x 1) + r unde r poate fi 1, 2, , 20. Avem:
x + 20(x 1) + r = 243 21x + r = 263, r 20.
Singura solutie convenabil
a este x = 12, r = 11. Sticlutele c
autate sunt numerotate
cu 12 (cea cu praf de stele), respectiv 231 (cea cu praf de aur).
149

V.124. Ar
atati c
a num
arul A = 100
1
1
+ +
35
587 589

7
7
7
1
+
+ +
+
+589
12 23
99 100
13

este cel mai mare num


ar natural de trei cifre distincte.

Anca Chiritescu, T
ig
anasi (Iasi)
Solutie. Calcul
and sumele (telescopice) din paranteze, obtinem c
a A = 100
693
294
+ 589
= 987, care este cel mai mare numar natural de trei cifre distincte.
100
589
V.125. Se consider
a num
arul a = 1 + 2 + 3 + + 2010 si multimea M = {n
N|na = k 2 , k N}. Determinati cele mai mici cinci elemente ale lui M .
Nicolae Iv
aschescu, Craiova
Solutie. Calcul
and suma, obtinem c
a a = 2010 2011 : 2 = 3 5 67 2011, unde
factorii ultimului produs sunt numere prime. Cum na este p
atrat perfect, rezulta c
a
n = 3 5 67 2011 m2 = am2 , cu m N. Cele mai mici cinci elemente ale lui M
sunt 0, a, 4a, 9a si 16a.
V.126. Se consider
a multimea A = {1, 2, 3, , 2010}. Scrieti multimea A ca
reuniune a trei multimi disjuncte dou
a c
ate dou
a, av
and acelasi cardinal si aceeasi
sum
a a elementelor.
Mirela Marin, Iasi
Solutie. Avem c
a A = B C D, unde B = {6k + 1|0 k 334} {6k + 6|0
k 334}, C = {6k + 2|0 k 334} {6k + 5|0 k 334} si D = {6k + 3|0 k
334} {6k + 4|0 k 334}. Fiecare dintre multimile B, C si D are cardinalul 670 si
suma elementelor egala cu 673 685, deci sunt ndeplinite cerintele problemei.
V.127. Determinati numerele naturale A pentru care A + S(A) = 2010. (Am
notat cu S(A) suma cifrelor num
arului A.)
C
at
alin Budeanu, Iasi
Solutie. Num
arul A este de cel mult patru cifre, cu cifra miilor cel mult egala
cu 2, deci S(A) 2 + 9 + 9 + 9 = 29. Rezult
a c
a 1981 A 2010. Verific
and cele
treizeci de posibilit
ati, obtinem c
a A {1986, 2004}.
Evident, num
arul verific
arilor poate fi micsorat. De exemplu, cum A si S(A) dau
.
acelasi rest la mpartirea prin 3, iar suma 2010 se divide cu 3, rezulta c
a A..3 si astfel
ram
an de f
acut doar zece verific
ari.
V.128. Reconstituiti o mp
artire, stiind c
a mp
artitorul, c
atul si restul sunt cifre
ale demp
artitului.
Ioan S
ac
aleanu, H
arl
au
Solutie. Fie D = I C + R, cu R < I. Cum I, C, R sunt cifre, rezulta c
a D 89,
iar din R < I, urmeaza c
a D nu poate fi numar de o cifra. Astfel, D = ab si
I, C, R {a, b}; deducem c
a I = C > R sau I > C = R. In cazul n care a > b,
obtinem c
a I = C = a, R = b, respectiv I = a, C = R = b; prima situatie conduce la
10a + b = a2 + b a = 10, imposibil, iar a doua conduce la 100 + b = ab + b b = 10,
din nou impisibil. R
am
ane c
a a < b, deci I = C = b, R = a sau I = b, C = R = a.
In primul caz, 10a + b = b2 + a 9a = b(b 1), cu unica solutie a = 8, b = 9, iar n
cel de-al doilea, 10a + b = ab + a 9a = a(b 1) b = 10, imposibil. In concluzie,
mpartirea c
autata este 89 = 9 9 + 8.
150

a+b

a+c

b+c

V.129. Fie a, b, c trei numere impare, iar A = 2 2 3 2 5 2 . S


tiind c
a 30A nu
este p
atrat perfect, ar
atati c
a m
acar unul dintre numerele a, b, c nu este p
atrat perfect.
Andrei Nedelcu, Iasi
a+c
b+c
a+b

,y=
si z =
Solutie. Intrucat a, b, c sunt impare, numerele x =
2
2
2
sunt naturale, avand suma egala cu a + b + c, deci impara. Rezult
a c
a x, y, z sunt
fie toate impare, fie unul impar si doua pare. Prima situatie nu convine, deoarece
numarul 30A = 2x+1 3y+1 5z+1 ar avea toti exponentii pari, deci ar fi p
atrat perfect.
R
am
ane c
a exista un num
ar par printre numerele x, y, z (de fapt, chiar doua) si atunci
doua dintre numerele impare a, b, c dau resturi diferite (1 si 3) la mpartirea prin 4.
Acela dintre numerele a, b, c care este de forma M4 + 3 nu poate fi p
atrat perfect.

Clasa a VI-a
VI.123. Stabiliti n c
ate moduri l putem scrie pe 2010 ca sum
a de trei numere
naturale nenule, direct proportionale cu trei numere naturale consecutive.
Mirela Obreja si Ioan Lungu, Vaslui
Solutie. Fie a, b, c, n N astfel nc
at a + b + c = 2010 si (a, b, c)D.P.(n, n + 1, n +
2); folosind proprietatea fundamental
a a sirului de rapoarte egale, avem c
a
a
b
c
a+b+c
2010
670
=
=
=
=
=
.
n
n+1
n+2
n + (n + 1) + (n + 2)
3(n + 1)
n+1
n+2
n
, iar c = 670
. Cum (n, n+1) = 1 si (n+1, n+2) =
n+1
n+1
1, rezulta c
a n + 1 este un divizor al lui 670, cel putin egal cu 2 (deoarece n 6= 0).
Otinem c
a n {1, 4, 9, 66, 133, 334, 669}, prin urmare 2010 poate fi descompus n
sapte moduri cu respectarea cerintelor problemei.
VI.124.
Intr-o duminic
a, bunica face cl
atite pentru nepoti; 40% dintre cl
atite
sunt cu gem, iar restul cu ciocolat
a.
In duminica urm
atoare, bunica face cu 10% mai
multe cl
atite cu gem si cu 5% mai putine cl
atite cu ciocolat
a.
In care dintre duminici
a f
acut bunica mai multe cl
atite?
Doru Turbatu, Iasi
Solutie. Fie n num
arul cl
atitelor facute de bunica n prima duminica. Dintre
2n
3n
acestea, 40% n =
sunt cu gem, iar
sunt cu ciocolat
a. In a doua duminica,
5
5
2n 10 2n
11n
3n
5 3n
57n
avem
+

=
cl
atite cu gem si

=
clatite cu ciocolat
a,
5
100 5
25
5
100 5
100
101n
n total
cl
atite. Rezult
a c
a n a doua duminica bunica a facut mai multe clatite
100
(cu 1% mai mult dec
at n prima duminica).
Atunci b = 670, a = 670

VI.125. Determinati restul mp


artirii prin 2010 a num
arului A = 12011 + 22011 +
2011
+ 2011
.
Andrei Pasa, elev, Iasi
n
n
Solutie. Pentru exponeti impari, a +b = M (a+b), oricare ar fi a, b N. In cazul
nostru, 12011 + 20092011 = M2010 , 22011 + 20082011 = M2010 , , 10042011 + 10062011 =
M2010 . Evident c
a 20102011 = M2010 , iar 20112011 = (2010 + 1)2011 = M2010 + 1.
151

Num
arul 10052011 este impar si se divide cu 1005, prin urmare 10052011 = M2010 +
1005. In concluzie, A = M2010 + 1006.
VI.126. Pe tabl
a sunt scrise numerele 1, 2, 3, , 2010. Andrei sterge de pe tabl
a
dou
a numere, nlocuindu-le cu media lor aritmetic
a si procedeaz
a astfel n mod repetat,
p
an
a c
and pe tabl
a r
am
ane un singur num
ar. Este posibil ca acest ultim num
ar s
a fie
2009, 25?
Gabriel Popa, Iasi
Solutie. Media aritmetica a doua numere distincte este strict mai mica decat
num
arul mai mare. Pentru ca n final sa ram
ana pe tabla numarul 2009, 25, la ultimul
pas trebuie sa faca media numerelor 2010 si 2008, 5. Pentru a obtine 2008, 5, la
penultimul pas Andrei va face media numerelor 2009 si 2008. Astfel, n primii 2007
pasi, ar trebui ca din numerele 1, 2, , 2008, sa ram
ana pe tabla 2008, fapt imposibil
conform observatiei initiale.
VI.127. Consider
am punctele coliniare distincte A, B, C, D si E astfel nc
at
AB = b, AC = a, BD = b a (2a < b < 3a), E este simetricul lui B fat
a de
D, iar mijlocul segmentului [AC] este punctul E. Aflati numerele a si b, stiind c
a
BD = 6.
Matei H
av
arneanu, elev, Iasi
Solutie. Consider
and pe dreapta AB sensul de parcurs dinspre A c
atre B, primele
a a
patru puncte se pot afla ntr-una dintre or2
a
b-a
2
dinile A B C D, C A B D sau
D
B
C E A
C A D B. In primele doua situatii, mijlocul segmentului AC nu poate coincide
cu simetricul lui B fat
a de D. R
am
ane de studiat doar cazul C A D B.
a
Cum E este mijlocul lui [AC], obtinem c
a AE = . Insa D este mijlocul lui [BE],
2
a
5a
prin urmare a + = b a, deci b =
. Conditia BD = 6 conduce la a = 4, b = 10.
2
2
b = 120. Perpendiculara n
VI.128. Se consider
a triunghiul isoscel ABC cu m(A)

A pe AC intersecteaz
a bisectoarea unghiului C n F si latura BC n E. Paralela prin
E la AB taie CF n M si not
am {P } = AM BC. Determinati m
asura unghiului

AP
B.
Gabriela Popa, Iasi

= 120
Solutie. Observam c
a AB = AC, m(B) = m(C) = 30 , iar m(BAE)

A
90
= 30 .
Cum ABkEM, deducem c
a
) = 30 . Ins
= 180 90
a m(AEC)
m(AEM
F
30 = 60 , prin urmare EM este bisectoarea
M

unghiului AEC. Rezult


a c
a M este centrul cerC
) = 1 90 = B
cului nscris n AEC, deci m(EAP
P
E
2

B) =
45 . Din triunghiul AEP obtinem c
a m(AP
180 60 45 = 75 .
VI.129. Se consider
a triunghiul ABC. Determinati un punct M pe latura [AC],
aflat la egal
a distant
a de v
arful A si de dreapta BC.
Mihaela Cianga, Iasi
152

A
Solutie. Fie O punctul de intersectie dintre BC si perpendiculara n A pe dreapta AC.
Din ipoteza problemei, rezulta c
a M este egal
departat de dreptele OA si OC, prin urmare se
afla situat la intersectia dintre AC si bisectoarea

unghiului AOC.

M
O

Clasa a VII-a
Determinati numerele abc, scrise n baza 10, pentru care num
arul A =
VII.123.
p
abc

abc 28 este natural.

Vasile Chiriac, Bac


au
Solutia 1. Observam c
a abc 28 = abc A N, deci numarul abc 28 este
p
atrat perfect. Rezult
a c
a abc = k 2 + 28, unde k {9, 10, , 31}. Verific
and cele 22
de cazuri posibile, obtinem c
a A este numar natural doar c
and abc {109, 812}.
p

ia 2.
Solut

Mai general, determinam numerele naturale x pentru care A =


x x 28 este num
ar natural. Vom avea x = k 2 +28, k N, iar k 2 k+28 = A2 ,
2
2
adica (2k 1) 4A = 111. Deducem c
a (2k 2A 1)(2k + 2A 1) = 111 si,
studiind cazurile posibile, reg
asim exact solutiile de mai sus. Remarcam astfel faptul
c
a nu este esential ca num
arul natural x sa fie de trei cifre.

Demonstrati c
a nu exist
a numere naturale n pentru care num
arul A =

VII.124.
n + n + 2010 s
a fie p
atrat perfect.
Neculai Stanciu, Buz
au

Solutie. Dac
a n, m sunt numere naturale astfel nc
at n + m Q , atunci n

nm
Q,
si m sunt p
atrate perfecte. Intr-adev
ar, am obtine c
a n m =
n+ m

1
deci n = [( n + m) + ( n m)] Q si m = ( n + m) n Q. Cum
2

n, m N, deducem c
a n, m N, prin urmare n si m sunt p
atrate perfecte.
S
a presupunem c
a ar exista n N pentru care A N; atunci n = a2 , n+2010 = b2 ,
cu a, b N. Rezult
a c
a b2 a2 = 2010, deci (b a)(b + a) = 2010. Cum A = a + b
este p
atrat perfect, ar trebui ca a + b = 1, b a = 2010, situatie care nu convine. In
concluzie, nu exista n N pentru care A sa fie p
atrat perfect.

VII.125. Demonstrati c
a num
arul A = 2009 20112010 2010 20112009 + 1 este
2
divizibil cu 2010 .
Tamara Culac, Iasi
Solutie. Intrucat (a + b)n = ma2 + nMa + bn , avem c
a 20112010 = M20102 +
2010 M2010 + 1 = M20102 + 1, iar 20112009 = M20102 + 2009 M2010 + 1. Atunci
A = 2009(M20102 + 1) 2010 (M20102 + 2009 M2010 + 1) + 1 = M20102 + 2009
M20102 2010 + 1 = M20102 .
VII.126. Dac
a a, b, c sunt lungimile laturilor unui triunghi, iar p este semiperimeb
c
a
+
+
6.
trul acestuia, demonstrati c
a
pa pb pc
Ionel Nechifor, Iasi
153

p
p
p
a
b
c
+
+
=
1 +
1 +
1 =
p

a
p

b
p

c
p

a
p

b
p

1
1
1
1
1
1
3 = ((pa)+(pb)+(pc))

+
+
+
+
p
pa pb pc
pa pb pc
3 9 3 = 6, cu egalitate n cazul triunghiului echilateral.
Solutie. Avem:

Not
a. S-au primit alte trei solutii ale acestei probleme din partea d-lui Titu
Zvonaru, Com
anesti.

A
VII.127. Se consider
a triunghiul ABC, punctul
D pe latura (BC) si romburile BDEF si CDGH cu
E
E, G (AD), astfel nc
at A, F si H sunt de aceeasi parte F
a dreptei BC, iar AD separ
a F si H. Demonstrati c
a
G
H
dreptele AD, BH si CF sunt concurente.
O
Dan Popescu, Suceava
Solutie. Fie {O} = BH CF ; cum BF kCH, rezulta
C
B
D
HO
HC
CD
c
a
=
=
. Cu reciproca teoremei lui Thales n BCH, obtinem c
a
OB
BF
DB
DOkCH si atunci O AD, deci dreptele AD, BH si CF sunt concurente.

VII.128. Pe latura (AB) a triunghiului ascutitunghic ABC se consider


a punctul
M , iar pe segmentul (CM ) punctul N , astfel nc
at BN M CAM. Demonstrati
c
a AN BC.
Constantin Apostol, Rm. S
arat

A
Solutie. Avem c
a N
M B CM
A si, cum cele
doua unghiuri sunt suplementare, ele vor fi drepte, deci

BM
CM AB. Fie {P } = BN AC; deoarece N
P

si m(ACM
) + m(A)
b = 90 , rezult
)+
ACM
a c
a m(ABP
b = 90 , prin urmare BP AC. Astfel, N se afl
m(A)
a M
N
la intersectia a doua naltimi ale triunghiului, adica este
C
ortocentrul acestuia. Deducem c
a AN este tot naltime, B
deci AN BC.
VII.129. Se consider
a triunghiul ABC cu BC > AC > AB si punctele D, E pe
latura (BC) si F pe (AC), astfel nc
at AB = BD = AF, iar AC = CE. Dreapta
BF intersecteaz
a AD si AE n G, respectiv H. Ar
atati c
a punctele D, E, G si H sunt
conciclice.
Daniela Brum
a, Deleni (Iasi)
Solutie. Observam c
a, pe BC, ordinea punctelor este B E D C,
A
deoarece altfel ar rezulta c
a BC > AB + AC, imposibil. Din triunghiurile isoscele ABF si EAC, obtinem c
a
1
1

m(ABF ) = 90 m(A), iar m(EAC) = 90 m(C),


2
2
1

b
deci m(BAH) = m(A) + m(C) 90 . Deducem c
a
F
2
G
H
= 180 m(ABF
) m(BAH)
= 180 m(A),
b
m(AHB)
D
C
B
E
= 180 m(A)
b = 180 m(D)

prin urmare m(EHG)

=
D)
(dat fiind faptul c
a triunghiul BAD este isoscel). Rezult
a astfel c
a m(EHG)+m(

154

. .
.

180 , de unde concluzia problemei.

Clasa a VIII-a
VIII.123. Rezolvati n R ecuatia 2x2 + 2y 2 2xy + 2x 10y + 14 = 0.
Ionica Marcovschi, Pascani
Solutie. Ecuatia dat
a se poate scrie sub forma (xy +2)2 +(x1)2 +(y 3)2 = 0.
Cum p
atratele numerelor reale sunt nenegative, obtinem c
a xy +2 = x1 = y 3 =
0, de unde x = 1, y = 3.
VIII.124. Demonstrati ca num
arul
A=

(24 + 22 + 1)(34 + 32 + 1) (20104 + 20102 + 1)


3(20102 + 2010 + 1)

este p
atrat perfect.
Bianca Maria Filip, elev
a, Iasi
Solutie. Folosind descompunerea n4 + n2 + 1 = (n2 + 1)2 n2 = [n(n 1) + 1]
[n(n + 1) + 1], obtinem c
a A = (22 + 2 + 1)2 (32 + 3 + 1)2 (20092 + 2009 + 1)2 , deci
A este p
atrat perfect.
VIII.125. Determinati numerele ntregi n pentru care
p

n2 + 3n + 9 =

n2 + 3n + 9.

Ionut Stroe, student, Iasi


Solutie. Partea ntreag
a fiind numar ntreg, rezulta c
a n2 + 3n + 9 = k 2 , unde
2
2
k N. Obtinem c
a (2n + 3) + 27 = 4k , deci (2k + 2n + 3)(2k 2n 3) = 27,
cu ambele paranteze numere ntregi. Analizand situatiile posibile, g
asim solutiile
n {8, 3, 0, 5}.

VIII.126. Fie a, b, c, p N cu a, b, c distincte si p R\Q, iar A = {a p, b p,

c p}. Functia f : A N este definit


a prin f (x) = x2 + (2c a b)x b 1. Dac
a
x, y A, x < y, demonstrati c
a f (x) < f (y).
Cosmin Manea si Dragos Petric
a, Pitesti

a (a2 p b 1) + a(2c a b) p N, de
Solutie. Cum f (a p) N, obtinem c
unde a(2c a b) = 0. S
tim c
a a 6= 0, deci 2c a b = 0 si atunci f (x) = x2 (b + 1).
Dac
a x, y A, x < y, numerele x si y vor fi pozitive, prin urmare x2 < y 2 si de aici
rezulta c
a f (x) < f (y).
VIII.127. Determinati numerele reale x, y si z, stiind c
a
4(x 1)y 2 z 2 + 4(y 1)z 2 x2 + 4(z 1)x2 y 2 = 3x2 y 2 z 2 .
Lucian Tutescu si Mariana M
arculescu, Craiova
Solutie. Dac
a unul dintre numerele x, y sau z este zero, vom avea si un al
doilea num
ar nul, prin urmare ecuatia data are solutiile (, 0, 0); (0, , 0) si (0, 0, ),
x1
+
cu , , R. Dac
a xyz 6= 0, mpartim prin 4x2 y 2 z 2 si obtinem c
a
x2
z1
3
y1
+
= . Fiecare dintre fractiile din stanga este ns
a cel mult egala cu
y2
z2
4
155

1
1 x1
( 2
(x 2)2 0, evident adevarat, cu egalitate doar daca x = 2).
4 x
4
Deducem c
a ecuatia din enunt, mai admite, n plus, solutia (2, 2, 2).
VIII.128. Dac
a cercurile nscrise n trei fete ale unui tetraedru sunt tangente
dou
a c
ate dou
a, atunci cercul nscris n a patra fat
a este tangent primelor trei.
Mih
aly Bencze, Brasov
Solutie. Consider
am cercurile nscrise triunghiurilor ABC, ACD si ABD
A
tangente muchiilor n T1 , T2 , , T6 . Avem c
a AT1 =
AT2 = AT3 = a (deoarece sunt tangente duse din A la
T3
cercurile nscrise n triunghiurile ABC, ACD). La fel,
T
BT1 = BT4 = BT6 = b; CT4 = CT2 = CT5 = c si
1
T2
DT5 = DT3 = DT6 = d. Dac
a M, N, P sunt punctele de
D
T
6
tangent
a ale cercului nscris triunghiului BCD cu laturile
1
T5
BC, CD respectiv BD, cum BM = PBCD CD = B
2
T4
C
1
(2b + 2c + 2d) (c + d) = b = BT4 , rezulta c
a M = T4 . La fel, N = T5 si P = T6 .
2
VIII.129. S
a se arate c
a din fetele unui cub de muchie l, confectionat din carton,
putem construi, f
ar
a resturi, fetele a sase cuburi de muchii l1 , l2 , , l6 astfel nc
at
l2 = l12 + l22 + + l62 .
Petru Asaftei, Iasi
2
Solutie. Imp
artim fiecare fata a cubului dat n sase p
atrate de laturi l1 = l si
3
1
4
1
l2 = l3 = = l6 = l. Evident, l12 + l22 + + l62 = l2 + 5 l2 = l2 .
3
9
9

. . .
. .
.

Clasa a IX-a
a2 + 1 b 2 + 1 c2 + 1
+
+
3.
b+c
c+a a+b
Pedro H.O. Pantoja, Brazil
x
y
z
3
Solutia 1. Conform inegalitatii lui Nesbitt, avem c
a
+
+
.
y+z z+x x+y
2
X a2 + 1
3
2
2
2
. Pe de
Luand x = a + 1, y = b + 1, z = c + 1, obtinem c
a
b 2 + c2 + 2
2
2
2
2
2
alt
a parte, cum b + 1 2b, c + 1 2c, rezulta c
a b + c + 2 2(b + c), deci
X a2 + 1
X a2 + 1
2
si de aici urmeaza inegalitatea ceruta. Egalitatea se
b+c
b 2 + c2 + 2
atinge pentru a = b = c = 1.
P a2
(a + b + c)2
a+b+c P 1
9
Solutia 2. Avem

=
si

,
b+c
2(a + b + c) r 2
b+c
2(a + b + c)
P a2 + 1
a+b+c
9
9
a+b+c

+
2

= 3.
deci
b+c
2
2(a + b + c)
2
2(a + b + c)
IX.111. Let a, b, c be positive real numbers. Prove that

IX.112. Ar
atati c
a pentru fiecare num
ar natural n 2, exist
a numerele naturale
1
1
1
1
x1 < x2 < < xn astfel nc
at
+
+ +
=
.
x1
x2
xn
2010
Radu Sava, Iasi
156

Solutie. Vom demonstra afirmatia prin inductie matematica. Pentru n = 2, putem considera x1 =2011 si x2 = 2010 2011. Presupunem c
a exista x1 , x2 , , xn ca n
1
1
1
enunt si sa ar
atam c
a exista y1 < y2 < < yn+1 astfel nc
at
+ ++
=
y1 y2
yn+1
1
1
1
1
=
. Cum
+
, putem lua y1 = x1 , , yn1 = xn1 ,
2010
xn
xn + 1
xn (xn + 1)
yn = xn + 1, yn+1 = xn (xn + 1) si toate cerintele sunt ndeplinite.
IX.113. Coardele AB si CD ale unui cerc de centru O sunt perpendiculare si
se intersecteaz
a n P . Dac
a E si F sunt mijloacele segmentelor AC, respectiv BD,
ar
atati c
a P E = OF .
Petru Asaftei, Iasi
Solutia 1. Dac
a R, S sunt mijloacele segmentelor AB, respectiv CD, atunci

D
2P O = 2P R + 2P S = (P A + AR) + (P B + BR) + (P C +




CS)+(P D+ DS) = (P A+P B+ P C+ P D)+(AR+ BR)+


(CS + DS) = (P A + P C) + (P B + P D) = 2P E + 2P F =

F
2P E + 2P O + 2OF . Deducem c
a 2P E + 2OF = 0 , prin
O

S
urmare P E = F O; n particular, P E = OF.

Solutia 2. Fie {M } = P F AC. Cum AP


M

, rezult

F
PB F
BP ACP
a c
a m(AP
M) +
P
A
B
R

AM ) = 90 , deci m(AM
P ) = 90 . Astfel, F P
m(P
E
C
AC si, cum OE AC, deducem c
a P F kOE. Analog se
arata c
a OF kP E si atunci P EOF este paralelogram, de unde concluzia problemei.

IX.114. a) Dac
a O este punctul de intersectie a diagonalelor neperpendiculare
ale patrulaterului convex ABCD, atunci ABkCD dac
a si numai dac
a AO2 + DO2 +
2
2
2
2
BC = BO + CO + AD .

BD) 6= 90 si not
am cu r1 , r2 razele cercurilor
b) Presupunem c
a ABkCD, m(AC,
nscrise n triunghiurile AOD, respectiv BOC si cu R1 , R2 razele cercurilor circumscrise acestor triunghiuri. Ar
atati c
a AD = BC r1 = r2 R1 = R2 .
Claudiu S
tefan Popa, Iasi

deci
Solutie. a) Cum AOD BOC (opuse la varf), atunci cos AOD = cos BOC,
2
2
2
2
2
2
BO + CO BC
AO + DO AD
D
C
=
(). Deducem
2AO DO
2BO CO
BO
AO
=
AO DO = BO CO
c
a ABkCD
O
OC
OD
()
2
2
2
2
2
2
2
2
AO + DO AD = BO + CO BC AO + DO +
BC 2 = BO2 + CO2 + AD2 .
B
b) Cum ABkCD, are loc egalitatea de la punctul a). A
In plus, AO DO = BO CO si SAOD = SBOC (= S). Atunci R1 = R2
AD DO AO
S
BO CO BC
S
=
p1 =
=
AD = BC, iar r1 = r2
4S
4S
p1
p2
p2 (= p) p21 = p22 AO2 + DO2 + BC 2 + 2(AO DO + AO BC + DO BC) =
BO2 + CO2 + AD2 + 2(BO CO + BO AD + CO AD) BC(AO + DO) =
AD(BO + CO) BC(p AD) = AD(p BC) BC = AD.
157

IX.115. Trei ceviene concurente mpart un triunghi ABC n sase triunghiuri mai
mici, av
and razele cercurilor circumscrise R1 , R2 , , R6 . Dac
a R = R1 +R2 + +R6 ,
1
demonstrati c
a m
acar dou
a dintre numerele R1 , R2 , , R6 sunt cel mult egale cu R.
6
Marius Dr
agan, Bucuresti
BO
AO
= 2R1 ,
= 2R2 ,
Solutie. Cu notatiile din figur
a, avem c
a
sin AP O
sin BP O
AO
BO R5
CO
R3
R1
A
=
=
=
. Analog,
,
, prin
deci
R2
BO
R4
CO R6
AO
AO BO CO
R1 R3 R5

= 1 si atunci
urmare
R2 R4 R6
BO CO AO
R1 R
N
6
P
R1 R3 R5 = R2 R4 R6 . Vom arata c
a cel putin unul dintre
R5
1
R2 O
numerele R1 , R3 si R5 este cel mult egal cu R. IntrR3 R4
6
1
1
adevar, n caz contrar am avea c
a R1 > R, R3 > R, B
C
M
6
6
1
1
1
R5 > R, deci R1 + R3 + R5 > R, iar R2 + R4 + R6 = R (R1 + R3 + R5 ) < R.
6
2
2
3

3
R
R2 + R4 + R6
R
Cum R1 R3 R5 >
, iar R2 R4 R6
, ajungem la o
<
6
3
6
contradictie. In aceeasi maniera se arata c
a cel putin unul dintre numerele R2 , R4 si
1
a.
R6 este cel mult egal cu R si cu aceasta, rezolvarea problemei este complet
6

Clasa a X-a

X.111. Rezolvati ecuatia [log1+|x| ( 1 + x2 + |x|)] [log1+x2 +|x| (1 + |x|)] = a,


unde a este un parametru real, iar [t] este partea ntreag
a a lui t.
S
tefan Gavril, Piatra Neamt
Solutie. Pentru ca baza logaritmilor sa nu fie 1,
se impune conditia x R .

Observam c
a pentru orice x R , numerele 1 + |x| si 1 + x2 + |x| sunt strict mai

1
mari dec
at 1, prin urmare log1+|x| ( 1 + x2 + |x|) =
= t > 0.

log 1+x2 +|x| (1 + |x|)


1
Nu putem avea t = 1, ntrucat x 6= 0 si atunci unul dintre numerele t sau se afla
t
n intervalul (0, 1), deci partea sa ntreag
a va fi 0. In concluzie, ecuatia data nu are
solutii dac
a a R , iar pentru a = 0 multimea solutiilor este R .
X.112. Se consider
a multimile finite X, Y si Z astfel nc
at Z Y, |X| = 4,
|Y | 5 si |Z| = 3. Determinati |Y |, stiind c
a num
arul functiilor f : X Y a c
aror
imagine include Z este 108.
Mihai Haivas si Constantin Chiril
a, Iasi
Solutie. Dac
a Z = {y1 , y2 , y3 } Y, notam cu Ai = {f : X Y |yi
/ f (X)},
i = 1, 2, 3. Num
arul functiilor care verific
a cerinta problemei este m4 |A1 A2 A3 |,
unde m = |Y |. Conform principiului includerii si excluderii, |A1 A2 A3 | = |A1 | +
|A2 | + |A3 | |A1 A2 | |A1 A3 | |A2 A3 | + |A1 A2 A3 | = 3(m 1)4 3(m
2)4 + (m 3)4 = m4 24m + 36. Deducem c
a 24m 36 = 108, de unde m = 6.
X.113. Fie a, b, x1 , x2 , , xn numere reale strict pozitive (n N, n 2) si o
158

permutare a multimii {1, 2, , n}. Demonstrati c


a


ax1 +

b
x(1)

axn +

b
x(n)

ax1 +

b
b
axn +
x1
xn

Dan Nedeianu, Drobeta Tr. Severin


Solutie. Elimin
and numitorii, avem de demonstrat c
a
(ax1 x(1) + b)(ax2 x(2) + b) (axn x(n) + b) (ax21 + b)(ax22 + b) (ax2n + b).
Observam c
a (ax1 x(1) + b)2 (ax21 + b)(ax2(1) + b), ntrucat aceasta inegalitate
este echivalent
a cu ab(x1 x(1) )2 0. Scriind si celelalte inegalitati similare si
nmultindu-le membru cu membru, obtinem ceea ce dorim.
X.114. Fie z1 , z2 , z3 C astfel nc
at |z1 | = |z2 | = |z3 | 6= 1 si z1 z2 z3 , z2 z3 z1
si z3 z1 z2 sunt numere reale. Demonstrati c
a (z1 z2 )(z2 z3 )(z3 z1 ) = 0.
Ionut Iv
anescu, Craiova
Solutie. Fie |z1 | = |z2 | = |z3 | = r. Cum z1 z2 z3 = z 1 z 2 z 3 si z 2 z 1 z 3 =
z2 z1 z3 , prin nmultire membru cu membru, obtinem (dupa reduceri) c
a (1r)(z1 z 2
z2 z 1 ) = 0. Intrucat r 6= 1, deducem c
a z1 z 2 = a R . Analog se arata c
a z2 z 3 = b
R si z3 z 1 = c R . Din z1 z 2 = a, z1 z 3 = c = c obtinem, prin mpartire membru cu
a
|a|
z2
= , unde = R, cu || =
= 1, asadar z2 = z3 . Similar,
membru, c
a
z3
c
|c|
z2 = z1 si atunci macar doua dintre cele trei numere vor fi egale, de unde cerinta
problemei.
X.115. Fie ABCD un patrulater convex si punctele P, Q AC, R, S BD astfel
PA
QA
AB
RB
SB
nc
at
=
=
=
=
. Dac
a M si N sunt mijloacele segmentelor
PC
QC
CD
RD
SD
P Q, respectiv RS, demonstrati c
a 2M N < P Q + RS.
Titu Zvonaru, Com
anesti
AB
Solutie. Not
am AC = , BD = ,
= k si putem presupune k > 1 (n cazul n
CD
y
care k = 1, unul dintre punctele S, P sau Q
nu exista).
Alegem un reper cartezian n raport
Q
cu care A(0, 0), B(a, 0), C(b, c), D(d, e), cu b >
C
S
D
d.
T
inand cont de formula care d
a coordonatele
P
punctului care mparte
un
segment

ntr-un
raport

ER
ck
bk
bk ck
, Q
,
,
,
dat, obtinem c
a P
x
1 +k 1 + k

1 k 1 k
a + dk ek
a dk ek
B
A
R
si S
, prin urmare
,
,
1+k 1+k
1k 1k
2k
2k
, iar RS =
.
Coordonatele punctelor M si
PQ =
21
21
k
k

 2

2
2
ck
bk
dk a 2k 2
, respectiv N
, deci M N 2 =
N sunt M
,
,
k2 1 k2 1
k2 1 k2 1

2

2


dk 2 a
ek 2
k4
bk 2
ck 2

+
=
(b d)2 + (c e)2 +
2
2
2
2
2
k 1
k 1
k 1 k 1
(k 1)2

159

.
.
..

a2
2ak 2 (b d)
+
. Cum CD2 = (b d)2 + (c e)2 si b d CD, rezulta
(k 2 1)2
(k 2 1)2

k2
k2
2
2
2 1
k

CD
+
AB
=
+
2AB

CD
(AB 2 + CD2 +
c
a MN2 2
(k 1)2
k2
(k 2 1)2
k(AB + CD)
.
2AB CD), de unde M N
k2 1
2k(AB + CD)
<
Pentru a ncheia rezolvarea, ar fi destul sa demonstram c
a
k2 1
2k(AC + BD)
, adica AB + CD < AC + BD. Aceasta inegalitate rezulta imediat
k2 1
din inegalitatea triunghiului: daca {E} = AC BD, atunci AB + CD < AE + EB +
CE + ED = (AE + EC) + (BE + ED) = AC + BD.

Clasa a XI-a
XI.111. Demonstrati c
a 4(x3 + y 3 ) 9xy|x y|, x, y [0, ).
Lucian Tutescu, Craiova
Solutie. Dac
a y = 0, avem de aratat c
a 4x3 0, adevarat. Fie y > 0; daca
x = y, inegalitatea este evident
a, iar c
and x 6= y putem presupune c
a x > y (datorit
a
x
3

simetriei). Imp
artind prin y , cu notatia t = > 1, avem de aratat c
a f (t) 0, unde
y
f : [1, ) R, f (t) = 4t3 9t2 + 9t + 4. Avem c
a f (t) = 3(4t2 6t + 3) > 0, t R
si, cum f (1) = 8 > 0, rezulta c
a f (t) > 0, t [1, ). Egalitatea n inegalitatea din
enunt se atinge c
and x = y = 0.

1
, n
2n + 1
pn

2wn ln 2wn ln nn
N (sirul lui Wallis), iar pn este al n-lea num
ar prim. Calculati lim
.
n

Gabriel Mrsanu, Iasi

pn
Solutie. Se stie c
a lim wn = , iar lim
= 1. In aceste conditii, avem
n
2 n n ln n
de-a face cu o nedeterminare de tip 1 . Folosind procedeul uzual, valoarea limitei
2wn
2wn
1

= 1.
cerute este el , unde l = lim
=
lim
2w
n
n ln
n ln 1 + 2wn 1

XI.112. Fie sirurile (wn )n1 si (pn )n1 , unde wn =

(2n)!!
(2n 1)!!

XI.113. Calculati limita sirului (xn ) definit prin: x1 (0, ), (0, 1)(1, ),
xn+1 = 1 + (1 + xn )1/ , n 1.
Gheorghe Costovici, Iasi
Solutie. Pentru nceput, fie (1, ); vom demonstra c
a lim xn = 0. Prin
n

inductie, obtinem c
a xn > 0, n N . Apoi, (1 + x) > 1 + x, x (0, ) (ntrucat
functia f : [0, ) R, f (x) = (1 + x) x 1 are derivata pozitiv
a si f (0) = 0).
Rezult
a c
a 1 + xn = (1 + xn+1 ) > 1 + xn+1 , de unde xn+1 < xn , n N . Astfel,
exista lim xn = l [0, ) si (1 + l) = 1 + l. Insa (1 + l) > 1 + l, daca l (0, ),
n
dupa cum am demonstrat anterior si ram
ane c
a l = 0.

Dac
a (0, 1), se arata similar c
a (1 + x) < 1 + x, x (0, ). Deducem c
a
sirul este strict cresc
ator si, daca ar fi marginit superior, ar avea o limita L (0, )
160

astfel nc
at (1+L) = 1+L, ceea ce ar intra n contradictie cu inegalitatea anuntata.
R
am
ane c
a sirul este nem
arginit superior, deci lim xn = +.
n
XI.114. Fiind date a, b, , R, determinati functiile f : R R cu proprietatea:

x

f (x)

y
= af (x) + bf (y), x, y R.
f (y)

Marius Tiba, elev, Bucuresti


Solutie. Observam c
a schimb
and x si y ntre ele, semnul determinantului se
schimb
a, ceea ce conduce la af (x) + bf (y) = af (y) bf (x), x, y R, adica
(a + b)(f (x) + f (y)) = 0, x, y R.
Dac
a a + b 6= 0, atunci f (x) + f (y) = 0, x, y R. Luand y = 0, obtinem c
a
f (x) = c, x R (unde c = f (0)). O astfel de functie verific
a egalitatea din enunt
daca si numai dac
a c = = 0. Atunci, daca 6= 0, ecuatia data nu are solutii, iar
daca = 0, singura solutie este functia nula.
Dac
a a + b = 0, definim g : R R prin
g(x) = f (x) , x R. Ecuatia
x a y
= a[g(x) g(y)], x, y R, de unde
functional
a din enunt devine
g(x)
g(y)
xg(y) yg(x) = (a )[g(x) g(y)], x, y R. Dac
a a = , pentru y = 1 obtinem
g(x) = xg(1), x R, deci f (x) = mx + , cu m R. Dac
a a 6= , luam y = 0 si
obtinem c
a g(x) = m(x + a ), x R, de unde f (x) = m(x + a ) + , x R.
Se constata usor c
a functiile afine obtinute n acest caz verific
a ecuatia din enunt.
Problema 26043 din G.M. 9/2008 se obtine din aceasta, n cazul particular =
= 0, a = b.
XI.115. Determinati matricele X, Y, Z M2 (Z), av
and determinantul 1, stiind
c
a X 4 + Y 4 + Z 4 = X 2 + Y 2 + Z 2 + 6I2 .
Florin St
anescu, G
aesti
Solutie. Dac
a not
am cu x urma matricei X, cum det X = 1, rezulta c
a X2 =
xX I2 . De aici, X 3 = xX X = (x2 1)X xI2 , iar X 4 = (x2 1)X 2 xX = (x3
2x)X(x2 1)I2 . Trec
and la urma, obtinem c
a T rX 2 = x2 2 si T r(X 4 ) = x4 4x2 +2
si relatii analoage au loc pentru matricele Y si Z. Trecem acum la urma n egalitatea
din enunt; rezulta c
a x4 4x2 +2+y 4 4y 2 +2+z 4 4z 2 +2 = x2 2+y 2 2+z 2 2+12,
de unde (x2 1)(x2 4) + (y 2 1)(y 2 4) + (z 2 1)(z 2 4) = 12 (). Cum x, y, z
sunt numere ntregi, produsele (x2 1)(x2 4), (y 2 1)(y 2 4) si (z 2 1)(z 2 4)
sunt sigur nenegative. Dac
a, de exemplu, |x| 3, atunci (x2 1)(x2 4) 40 si se
ajunge la o contradictie. Rezult
a c
a |x|, |y|, |z| {0, 1, 2}; pentru a putea avea loc
2
2
(*), va trebui ca |x| = |y| = |z| = 0, prin urmare
Z 2 = I2 . In concluzie,
X = Y =
a
b
, cu a, b Z si a2 + 1
X, Y si Z vor fi matrice arbitrare de forma 1 a2
a
b
divizibil cu b.

Clasa a XII-a
XII.111. Fie x, y, z numere reale nenule. Dac
a x + y + z = 0 si x5 + y 5 + z 5 =
7
7
2
x + y + z , calculati valoarea expresiei A = x + y 2 + z 2 .
Mihai Cr
aciun, Pascani
7

161

Solutie. Fie xy + yz + zx = a, xyz = b; atunci x, y, z sunt solutiile ecuatiei


t3 + at b = 0. Not
and Sn = xn + y n + z n , n N, obtinem c
a Sn+3 = aSn+1 + bSn .
Cum S0 = 3, S1 = 0, S2 = 2a, deducem: S3 = 3b; S4 = 2a2 ; S5 = 5ab; S7 = 7a2 b.
5
Din conditia S5 = S7 , rezulta c
a ab(7a + 5) = 0. Cum ab 6= 0, obtinem c
a a = si
7
10
A = S2 =
.
7
Z

XII.112. Calculati limita sirului (an )nN definit prin an =

xn + xn+2 dx,

n N.

Bogdan Victor Grigoriu, F


alticeni
Solutia 1. Vom aplica inegalitatea CBS, forma integrala:
Z

an =
0

xn 1 + x2 dx

x dx

(1 + x2 )dx =

Cum evident an > 0, n N, rezulta c


a lim an = 0.
n

XII.113. Calculati lim

1
x
1
x+1

3(n + 1)

x2

Solutia 2 (Moubinool Omarjee). Observam c


a 0 < an =

Z 1

n
2 2
x 2 2dx =
, n N, prin urmare lim an = 0.
n
n+2
0
Z

1 + x2 dx

ctg t2 dt.
Z

Solutie. Din formula de medie,

1
x
1
x+1

ctg t2 dt =

Silviu Boga, Iasi


1
1
1
ctg t2x =

x x+1
x(x + 1)

1
1
x2
t2x
. Cum
< tx < , atunci lim tx = 0 si lim (xtx ) = 1;
cos2 tx
2
x
x
(xtx )2
x+1
x
sin tx
obtinem c
a limita cerut
a este egala cu 1.
XII.114. Se consider
a functia continu
a f : [0, 1] (0, 1]. Demonstrati c
a pentru
Z

orice n N , exist
a un unic xn (0, 1) astfel nc
at

xn

f (t)dt.

f (t)dt = n
0

xn

1
1
f (t)dt.
n
f (1) 0
Florin St
anescu, G
aesti
Not
a. Problema a aparut si n G.M. 6/2010, cu numarul C.O: 5130. Solutia sa
poate fi g
asit
a n G.M. 1/2011.

Calculati limita sirului (xn ) si ar


atati c
a lim n(1 xn ) =

XII.115. Fie F multimea functiilor de dou


a ori derivabile pe [a, b] cu derivata
de ordin doi continu
a, pentru care f (a) = si f (b) = , unde si sunt constante
Z

fixate. Not
am I(f ) =
a

[f (x)]2 dx, f F si J(f ) =

b
a

[f (x)(1 + 2f (x))]2 dx,

f F . Determinati min{I(f )|f F } si min{J(f )|f F }.


Adrian Corduneanu, Iasi
162

Solutie. Fie f0 F astfel nc


at I(f ) I(f0 ), f F . Orice f F poate
fi scrisa sub forma f = f0 + , unde C 2 [a, b] si (a) = (b) = 0, iar este o
Z

constant
a arbitrara. Rezult
a c
a I(f ) = I(f0 +) =
Z

2
a

f0 (x) (x)dx + 2

[(f0 +) ]2 dx =

[f0 (x)]2 +

[ (x)]2 dx, expresie al c


arei minim se atinge pentru = 0

(ntrucat I(f ) I(f0 )). Rezult


a c
a

b
a

f0 (x) (x)dx

= 0, de unde
a

f0 (x)(x)dx =

0, C 2 [a, b] cu (a) = (b) = 0. De aici, f0 (x) = 0, x [a, b], deci f0 (x) =


x + , cu , constante care se determina din conditiile n capete. In final, g
asim

c
a f0 este unic determinat: f0 (x) =
(x a) +
(b x). Valoarea minima a
ba
ba
( )2
lui I(f ) este I(f0 ) =
.
ba
Pentru a doua parte, observ
am c
a f (1 + 2f ) = (f + f 2 ) , f F . Considerand

2
functia g = f + f , cu g(a) = + , g(b) = + 2 , avem de g
asit minimul integralei
Z b
(g(b) g(a))2
[g (x)]2 dx. T
inand seama de a), valoarea minima a lui J(f ) este
=
ba
a
( )2 (1 + + )2
.
ba

Parabola
Un preasf
ant p
arinte spune discipolului sau:
Fiule, sa nu uiti niciodat
a: y = ax2 + bx + c !
Da, preasfinte, o sa tin minte. . .
Peste ani de zile, credinciosul l ntreaba pe mentorul sau:
M
a chinuie o ntrebare, p
arinte, ce este y = ax2 + bx + c ?
La care preasf
antul p
arinte i raspunde:
Fiule, asta e o parabol
a!

163

Solutiile problemelor propuse n nr. 1/2011


Clasele primare
P.206. Dan a scris n ordine descresc
atoare numerele de la 75 la 23. Calculati
diferenta dintre al zecelea si al 32-lea num
ar.
(Clasa I)
Andreea Bzdg
a, elev
a, Iasi
Solutie. Al zecelea numar este 75 10 + 1 = 66, iar al 32-lea este 75 32 + 1 = 44.
Diferenta celor doua numere este 22.
P.207. Fiec
arei forme geometrice i corespunde un pret de cost: 

10 lei, 20 lei, 30 lei, 20 lei. C


at cost
a confectionarea
c
asutei?
(Clasa I)
Mariana Nastasia, elev
a, Iasi
Solutie. C
asuta costa 10 lei + 10 lei + 10 lei +20 lei +30 lei +20
lei = 110 lei.
P.208. Pe trei rafturi sunt 75 c
arti. Dac
a pe primul raft punem jum
atate din
c
artile de pe cel de-al doilea, atunci vom avea pe rafturi numere consecutive de c
arti.
C
ate c
arti erau la nceput pe fiecare raft?
(Clasa I)
Iulia Sticea, elev
a, Iasi
Solutie. La nceput, pe primul raft erau a c
arti, pe al doilea b + b c
arti, iar pe
al treilea c c
arti. In final, pe primul raft sunt a + b c
arti, pe al doilea b c
arti, iar pe
al treilea c c
arti. Deoarece a + b > b, inseamn
a c
a pe primul raft vor fi 26 c
arti, pe
al doilea 25 c
arti, iar pe al treilea 24 c
arti. La nceput, pe cele trei rafturi erau 1, 50,
respectiv 24 de c
arti.
P.209. Num
arul lalelelor dintr-o vaz
a este cu 7 mai mare dec
at num
arul trandafirilor, care reprezint
a jum
atate din num
arul lalelelor. C
ate flori sunt n vaz
a?
(Clasa a II-a)
Inst. Maria Racu, Iasi
Solutie. Num
arul florilor din vaza este (7 + 7) + 7 = 14 + 7 = 21.
P.210. Ce zi nu poate fi ast
azi, dac
a alalt
aieri nu a fost luni si poim
aine nu este
s
amb
at
a?
(Clasa a III-a)
Mihaela G
alc
a, elev
a, Iasi
Solutie. Ast
azi nu poate fi nici miercuri, nici joi.
P.211. Suma a dou
a numere este un num
ar de dou
a cifre al c
aror produs este 5.
Care sunt cele dou
a numere, dac
a diferenta lor este 7?
(Clasa a III-a)
Ana Cojocariu, Iasi
Solutie. Suma celor doua numere poate fi 15 sau 51. Deoarece diferenta lor este
7, atunci numerele sunt 4 si 11 sau 22 si 29.
P.212. Un elev a gresit la adunarea a dou
a numere: a scris cifra zero la sf
arsitul
primului num
ar n loc s-o scrie la sf
arsitul celui de-al doilea si astfel a obtinut suma
98 si nu 89 - suma corect
a. Aflati cele dou
a numere.
(Clasa a III-a)
Cristina Timofte, Iasi
Solutie. Dac
a numerele sunt a si b, avem a + b = 89, iar 10a + b : 10 = 98 sau
a + b = 89 si 100a + b = 980, de unde 99a + 89 = 980. Gasim a = 9, b = 80.
164

P.213. O vil
a turistic
a are apartamente cu 3 si 4 camere. S
tiind c
a n usa de
intrare a fiec
arui apartement se afl
a c
ate 2 chei, iar num
arul camerelor si al cheilor
la un loc este 39, aflati c
ate apartamente cu 3 camere sunt n vil
a.
(Clasa a IV-a)
Dorel Luchian, Iasi
Solutie. Not
am cu x num
arul apartamentelor cu 3 camere si cu y numarul celor
cu 4 camere. Avem (3x + 4y) + (2x + 2y) = 39 5x + 6y = 39, cu solutia unic
a
x = 3, y = 4.
P.214. La o mas
a rotund
a stau cinci copii, fiecare av
and c
ate un jeton pe care
este scris un num
ar. Toti copiii afirm
a c
a vecinii lor au jetoane cu numere de parit
ati
diferite. Ar
atati c
a m
acar un copil nu spune adev
arul.
(Clasa a IV-a)
Iuliana Moldovan, student
a, Iasi
Solutie. Fie A, B, C, D si E cei cinci copii si sa presupunem c
a niciunul dintre ei
nu minte. Spunem c
a A este par dac
a are un jeton cu numar par si spunem c
a A este
impar n caz contrar.
Dac
a A este par, avem succesiv c
a C este impar, E par, B impar, D par, A impar
contradictie. Analog ajungem la contradictie daca A este impar. In concluzie, cel
putin un copil minte.

P.215. Intr-un
rucsac sunt 12 sosete care pot forma sase perechi de culori diferite,
iar dou
a dintre ele sunt rupte. C
ate sosete trebuie scoase la nt
amplare din rucsac
pentru a avea o pereche bun
a?
(Clasa a IV-a)
Petru Asaftei, Iasi
Solutie. Avem de analizat doua cazuri. Primul, c
and cele doua sosete rupte au
aceeasi culoare. Cea mai dezavantajoas
a situatie este sa scoatem 7 sosete, dintre care
5 sosete bune de culori diferite si o pereche de aceeasi culoare, ns
a rupte. Dac
a mai
extragem o soset
a, atunci putem forma o pereche de sosete bune.
Al doilea, c
and cele doua sosete rupte au culori diferite. In aceasta situatie, cel
mai dezavantajos este sa extragem 8 sosete, 6 sosete bune de culori diferite si doua
sosete rupte, tot de culori diferite. Dac
a mai extragem o soset
a, aceasta este buna
si putem forma o pereche de sosete bune. In concluzie, pentru a fi siguri c
a avem o
pereche de sosete bune, trebuie sa extragem 9 sosete.

Clasa a V-a
V.130. Determinati num
arul a0bb(3) , dac
a a0bb(3) = bba(7) .
Nicolae Iv
aschescu, Craiova
Solutie. Trec
and n baza 10, egalitatea din enunt devine 27a+ 4b = 56b + a, adica
26a = 52b, prin urmare a = 2b. Cum a si b sunt cifre n baza 3, a 6= 0, convine doar
situatia a = 2, b = 1. In concluzie, a0bb(3) = 2011(3) .
V.131. Se consider
a num
arul N = 1 + 7 + 13 + 19 + a + b + 37, unde termenii
sumei sunt scrisi n ordine strict cresc
atoare. Determinati num
arul perechilor (a, b)
pentru care N este p
atrat perfect.
Anca Chiritescu, T
ig
anasi (Iasi)
Solutie. Observam c
a Nmin = 1 + 7 + 13 + 19 + 20 + 21 + 37 = 118, iar Nmax =
1 + 7 + 13 + 19 + 35 + 36 + 37 = 148, deci N este p
atrat perfect daca este egal cu
121 sau cu 144. Rezult
a a + b poate lua valorile 44 sau 67, prin urmare (a, b)
165

{(20, 24); (21, 23); (31, 36); (32, 35); (33, 34)}. In concluzie, exista cinci perechi (a, b)
pentru care N este p
atrat perfect.
V.132. Numerele 0, 1, . . . , 2011 sunt aranjate ntr-un tablou astfel:
0
1
2
3

7
6
5
4

8
9
10
11

15
14
13
12

...
...
...
...

2008
2009
2010
2011.

a) Stabiliti care sunt elementele coloanei 211 (de sus n jos).


b) Calculati suma elementelor de pe a treia linie.
Ioana Maria Popa, elev
a, Iasi
Solutie. a) Coloana n contine numerele 4n 4, 4n 3, 4n 2 si 4n 1, n ordine
cresc
atoare dac
a n este impar si n ordine descrescatoare daca n este par. Atunci
elementele coloanei 211 sunt (de sus n jos): 840, 841, 842, 843.
b) Suma elementelor de pe a treia linie este S = S1 + S2 , unde S1 = 2 + 10 +
(2 + 2010) 252
= 253 512, iar S2 = 5 + 13 + 21 + . . . + 2005 =
18 + . . . + 2010 =
2
(5 + 2005) 251
= 252 255. In concluzie, S = 505 767.
2
V.133. Calculati sumele:
a) S1 = 85 + 985 + 9985 + . . . + |99 {z
. . . 9} 85;
2011 de 9

b) S2 = 17 + 197 + 1997 + . . . + 1 |99 {z


. . . 9} 7.
2011 de 9

Eugeniu Bl
ajut, Bac
au
Solutie. a) Observam c
a S1 = (100 15) + (1000 15) + . . . + (1 00
.
.
.
0
15)
=
| {z }
2013 de 0

11
. . 11} 00 15 2012 = 11
. . 11} 080920.
| .{z
| .{z
2012de1

2008 de 1

b) Cum S1 = 5S2 , atunci S2 = S1 : 5 = 22


. . 22} 16184.
| .{z
2008 de 2

V.134. Lucian-Georges r
am
ane nesupravegheat la calculator si, din neatentie,
printeaz
a toate numerele naturale de 1 la 1000000. Drept pedeaps
a, el trebuie s
a
numere de c
ate ori a fost tip
arit
a cifra 5. Care este r
aspunsul corect pe care trebuie
s
a-l dea copilul?
Andrei Nedelcu si C
at
alin Budeanu, Iasi
Solutie. Num
arul aparitiilor cifrei 5 nu se modific
a daca n loc de numerele
1 . . . 1000000 consider
am numerele 0 . . . 999999, iar fiecare numar se scrie cu ajutorul
a sase cifre, adaug
and zerouri n stanga sa (de exemplu, n loc de 2011 vom scrie
002011). In acest fel, num
arul total de cifre va fi 6 106 si fiecare cifra va avea acelasi
num
ar de aparitii, deci cifra 5 va fi folosita de 6 106 : 10 = 600 000 ori.
V.135. Demonstrati c
a 2122 < 1037 .

Titu Zvonaru, Com


anesti
166

Solutie. Cum 210 = 1024 < 1029 = 3 343 = 3 73 , atunci 220 < 9 76 =
9 (72 )3 < 9 503 = 9 23 56 , deci 217 < 9 56 . Deducem c
a 223 < 9 106 , prin urmare
69
18
8
18
61
18
2 < 729 10 < 3 2 10 , de unde 2 < 3 10 . Obtinem c
a 2122 < 9 1036 < 1037 .
Not
a (T. Zvonaru). Pe parcurs, am obtinut c
a 223 < 9 106 , deci 217 < 107 .
Aceasta inegalitate poate fi folosit
a pentru a demonstra c
a 2100 < 261 , fara a calcula
33
2 (a se vedea solutia problemei V.111. din Rec.Mat. 2/2010). Intr-adev
ar, 223 <
63
10
107 2207 < 1063 2200 <
si, cum 128 > 102 , rezulta c
a 2200 < 1061 .
128
V.136. Se consider
a sirul de numere naturale 4, 19, 163, 1945, . . .. Determinati
ultimele 501 cifre ale celui de-al 2011-lea termen al sirului.
Mihai Cr
aciun, Pascani
Avem: a1 = 4 = 31 1! + 1, a2 = 19 = 32 2! + 1, a3 = 163 = 33 3! + 1,
a4 = 1945 = 34 4! + 1. Atunci a2011 = 32011 2011!+ 1. Num
cu
arul de
zerouri

care se
2011
2011
2011
termina scrierea zecimala a num
arului 2011! este
+
+
+... =
5
52
53
402 + 80 + 16 + 3 + 0 + . . . = 501, deci ultimele 501 cifre ale lui a2011 sunt 00
. . 00} 1.
| .{z
500de0

Clasa a VI-a
VI.130. Fie A1 , A2 , . . . , A2011 puncte echidistante pe o dreapt
a d. Not
am cu
B1 , B2 , . . . , B2010 mijloacele segmentelor A1 A2 , A2 A3 , . . . , A2010 A2011 ; apoi, fie C1 , C2 ,
. . . , C2009 mijloacele segmentelor B1 B2 , B2 B3 , . . . , B2009 B2010 s.a.m.d., p
an
a c
and
obtinem un singur punct M . Determinati pozitia punctului M .
Elena Iurea, Iasi
Solutie. La fiecare pas, num
arul punctelor scade cu 1. In etapa 1, avem punctele
A1 , A2 , . . . , A2011 . Dup
a un pas, n etapa 2, avem punctele B1 , B2 , . . . , B2010 . In etapa
3, punctele sunt C1 = A2 , C2 = A3 , . . . , C2009 = A2010 si n toate etapele cu numar
de ordine impar vom obtine puncte dintre cele initiale: A3 , A4 , . . . , A2009 n etapa 5;
A4 , . . . , A2008 n etapa 7; . . . ; A1005 , A1006 , A1007 n etapa 2009; A1006 n etapa 2011.
In concluzie, M = A1006 .
C
VI.131. Se consider
a ABC cu AC = BC si
punctele M, N cu B [AM ], N [AC]. Ar
atati c
a

BC CN
M.
M A = M N dac
a si numai dac
aM
N
Petru Asaftei, Iasi
CBA,
avem:
Solutie. Folosind faptul c
a CAB

M A = M N M AN M N A CBA M
NA

B
M
A
M BC CN M .
VI.132. Se consider
a triunghiurile isoscele ABC si ABD cu AB = AC = AD,
= 32 , punctele C si D fiind de o parte si de alta a dreptei
= 28 , m(BAD)
m(BAC)
AB. Dac
a E este mijlocul segmentului AC si {M } = DE BC, determinati m
asurile
unghiurilor triunghiului M AB.
Eugeniu Bl
ajut, Bac
au

Solutie. Triunghiul ACD are AC = AD si m(CAD) = 28 + 32 = 60 ,


deci este echilateral; cum DE este mediana, rezulta c
a DE AC. Atunci M E
167


este mediatoarea segmentului AC si triunghiul M AC va fi isoscel, cu m(M
AC)
1

m(M CA) = (180 28 ) = 76 si m(AM C) = 180 2 76 = 28 . Apoi,


2

) = 180 28 48 = 104 .
m(M
AB) = 76 28 = 48 , iar m(ABM

arul
VI.133. M
asura unui unghi este u = ab ab , unde 0 < ab < 60, iar num
natural nenul n are proprietatea c
a n u exprim
a un num
ar ntreg de grade. Aflati u
pentru care n este minim.
Marian Pantiruc, Iasi


ab
ab
61 ab
Solutie. Observam c
a u =
=
. Cum n u N ,
+
60 3600
3600
0 < ab < 60, 61 este relativ prim cu 3600 iar n este minim, rezulta c
a ab trebuie sa
fie multiplu al celui mai mare divizor al lui 3600 mai mic decat 60. Astfel, ab = 45,
caz n care n = 80.
VI.134. Ar
atati c
a num
arul A = 20102010 + 20122012 2 este divizibil cu 2011.
Daniela Munteanu, Iasi
2010
2010
Solutie. Cum 2010
= (2011 1)
= M2011 + (1)2010 = M2011 + 1 si
2012
2012
2012
= (2011 + 1)
= M2011 + 1, rezulta c
a A = (M2011 + 1) + (M2011 + 1) 2 =
M2011 .
VI.135. Determinati c
ate fractii ireductibile si subunitare au proprietatea c
a suma
dintre num
ar
ator si numitor este 1005.
Mirela Marin, Iasi
a
Solutie. Fractiile c
autate sunt de forma
, cu a < 1005 a, adica a
1005 a
{1, 2, . . . , 502}. O astfel de fractie este reductibil
a prin d daca d|a si d|1005 a, deci
c
a
nd
d|1005,
adic
a
d

{3,
5,
67,
3

5,
3

67,
5

67}.
arul fractiilor reductibile este






Num
502
502
502
502
502
502
+
+

= 167 + 100 + 7 33 2 1 = 238,


3
5
67
15
201
335
prin urmare num
arul de fractii ireductibile va fi 502 238 = 264.
VI.136. Dup
a ce fiecare echip
a a jucat cu fiecare dintre celelalte c
ate un meci,
clasamentul grupei A de la Campionatul mondial de fotbal 2010 ar
ata astfel:
1.
2.
3.
4.

Uruguay
Mexic
Africa de Sud
Franta

3
3
3
3

210
111
111
012

4-0
3-2
3-5
1-4

7
4
4
1

S
tiind c
a n meciul Uruguay-Franta nu s-a marcat niciun gol, aflati rezultatele fiec
aruia
dintre cele sase meciuri.
Titu Zvonaru, Com
anesti
Solutie. Deoarece rezultatul meciului Uruguay-Franta a fost 0 0, scorurile
posibile pentru fiecare echipa sunt:
168

Uruguay:
Franta:
Mexic:

(0-0,1-0,3-0) sau (0-0,2-0,2-0)


(0-0,1-2,0-2) sau (0-0,1-3,0-1)
(0-0,2-0,1-2) sau (0-0,3-0,0-2) sau (0-0,3-1,0-1)
sau (1-1,2-0,0-1)
Africa de Sud: (0-0,1-0,2-5) sau (0-0,2-0,1-5) sau (0-0,3-0,0-5)
sau (0-0,2-1,1-4) sau (0-0,3-1,0-4) sau (0-0,3-2,0-3)
sau (1-1,1-0,1-4) sau (1-1,2-0,0-4) sau (1-1,2-1,0-3).
Cum printre scorurile posibile ale echipelor Uruguay, Franta si Mexic nu se g
asesc
5 2, 5 1, 5 0, 4 1, 4 0, 3 2 singura posibilitate pentru Africa de Sud ram
ane
(1 1, 2 1, 0 3). Deoarece meciul egal al Africii de Sud a fost cu Mexicul, scorurile
acestei echipe vor fi (1 1, 2 0, 0 1). Acum obtinem usor scorurile echipelor
Uruguay: (0 0, 1 0, 3 0) si Frantei: (0 0, 1 2, 0 2). Rezultatele complete ale
grupei au fost:
Uruguay-Franta 0 0; Mexic-Africa de Sud 1 1;
Uruguay-Africa de Sud 3 0; Mexic-Franta 2 0;
Uruguay-Mexic 1 0; Africa de Sud-Franta 2 1.

Clasa a VII-a

VII.130. Fie C mijlocul laturii AB a triunghiului


ABC, iar P un punct pe segmentul CC . Dac
a AP
BC = {R}, ar
atati c
a AP CR = BC P R.
Claudiu S
tefan Popa, Iasi
Solutie. Cu teorema lui Menelaus n triunghiul ABR
AC BC RP

=
cu transversala C P C, obtinem c
a
C B CR P A
BC P R
1, deci
= 1, de unde cerinta problemei.
AP CR

A
C
P
B

a
b+c
VII.131.
Intr-un triunghi ABC, lungimile laturilor verific
a relatia
=
.
b
a
b = , determinati m
si C.

asurile unghiurilor B
Dac
a m(A)
Neculai Stanciu, Buz
au
Solutie. Consider
am punctul D astfel nc
at A (CD), AD = AB = c. Cum
CD
CB
este comun, rezult
=
si unghiul C
a c
a B
CA
CB
= m(CDB).

CAB CBD, de unde m(ABC)


Insa BAC
este unghi exterior triunghiului isosc
a
= 2m(D)
=
cel ABD, prin urmare m(BAC)

= , iar
2m(CBA).
Deducem c
a m(CBA)
c
2
D
C b A
= 180 3 . (Evident, se impune ca < 120, n caz contrar neexist
m(C)
and
2
triunghiuri ca n enunt.)
VII.132.
In cercul C(O, r) se consider
a coarda [AB]. Not
am cu C mijlocul arcului
cu O simetricul punctului O fat
Determinati
a de AB si fie x = m(ACB).
mare AB,
valorile lui x pentru care O Int C.
Geanina Hav
arneanu, Iasi

169

r
AO
<
=
sin(OAB)
2
2
1
< 30 m(AOD)
> 60 (unde D este mijlocul
C
m(OAB)
2
m(AD)
> 60 m(AB)
> 120 2x >
arcului mic AB)

120 x > 60 . Evident c


a x 90 (deoarece C se afla pe
O
prin urmare valorile c
arcul mare AB),
autate ale lui x sunt cele
B
A
din intervalul (60 , 90 ].

O
VII.133. Se consider
a p
atratul ABCD cu latura de 3 5cm.
D
Pe semidreptele (AB si (BC
a punctele
M , respectiv
se consider
asura unghiului
N , astfel nc
at AM = 7 5cm si BN = 10 5cm. Determinati m

M
N D.
Constantin Apostol,
Rm. S
arat

Solutie. Intrucat AM = CN = 7 5cm si AD = CD = 3 5cm, triunghiurile


D
dreptunghice AM D si CN D sunt congruente (C.C.), A

prin urmare DM = DN si M DA N DC. Insa


) + m(M

m(ADM
DC) = 90 , deci m(N
DC) + B
N

m(M DC) = 90 , adica m(M DN ) = 90 . Deducem c


a

N D) =
DM N este dreptunghic isoscel si astfel m(M
45 .

2+ M
Comparati numerele
a = 2 1+2
VII.134.

reale

2 3 + . . . + 2 64 si b = ( 0 + 2) + ( 1 + 3) + ( 2 + 4) + . . . + ( 63 + 65).
Cosmin Manea si Dragos Petric
a, Pitesti
Solutie. Elimin
and termenii comuni
ai
celor
dou
a
sume,
compararea
numerelor
a

si b revine la compararea numerelor 1 + 64 cu 65. Insa 1 + 64 = 9 = 81 >


65, prin urmare a > b.
VII.135. Rezolvati n numere ntregi ecuatia x3 + y 3 + 3xy = 9.
Cristina Ene, elev
a, Craiova

Solutie. Cum 3 x3 + y 3 , avem doua cazuri:
I. x = 3a, y = 3b, cu a, b Z; atunci 27(a3 + b3 + ab) = 9, imposibil.
II. x = 3a+ 1, y = 3b 1, cu a, b Z; atunci x3 + y 3 = 27a3 + 27a2 + 9a+ 1 + 27b3
.
.
.
27b2 + 9b 1, deci x3 + y 3 ..9. Rezult
a atunci c
a 3xy ..9, adica xy = (3a + 1)(3b 1)..3,
ceea ce este imposibil. In concluzie, ecuatia data nu are solutii ntregi.
Solutie. Avem O Int C OO < r d(O, AB) <

..
.

VII.136. Numerele ntregi x, y si z verific


a relatia 17x + 5y 2z = 0. Ar
atati c
a
num
arul
(3x + y)(z x)(2x + 2y + z)(3y + 3z x)
A=
210
este natural, p
atrat perfect.
Mihai Haivas, Iasi
x+y
17x + 5y
= 8x + 2y +
, rezulta c
a x + y = 2n, n Z.
Solutie. Cum z =
2
2
2 3 5 7 (x + n)2 (3n + 2x)2
Atunci y = 2n x si z = 5n + 6x, deci A =
=
210
[(x + n)(3n + 2x)]2 si concluzia se impune.
170

Clasa a VIII-a
VIII.130. Date fiind punctele A, B si C, determinati punctele P din spatiu cu
proprietatea c
a P A2 + P B 2 P C 2 = AB 2 AC 2 BC 2 .
Dan Popescu, Suceava
Solutie. Fie M mijlocul segmentului AB; folosind teorema medianei n triunghi1
urile P AB si CAB, obtinem c
a P A2 + P B 2 = 2P M 2 + AB 2 , iar AC 2 + BC 2 =
2
1
2CM 2 + AB 2 . Inlocuind n relatia din enunt, g
asim c
a 2(P M 2 + CM 2 ) = P C 2 .
2
Cu teorema medianei n CM P , avem c
a 2(P M 2 + CM 2 ) = 4M N 2 + P C 2 , unde
N este mijlocul segmentului CP. Atunci 4M N 2 = 0, de unde N = M. In concluzie,
singurul punct P cu proprietatea din problem
a este simetricul lui C fata de mijlocul
lui AB.
Not
a. Cazul triunghiului dreptunghic n C, situatie n care relatia din eunt devine
P A2 + P B 2 = P C 2 , constituie obiectul uneia dintre problemele de la Concursul
National de Matematic
a, R.D.G. 1982, problem
a ce apare si n G.M. 11/2000. A se
vedea si problema nrudita G5 din RecMat 2/2001.
VIII.131. Demonstrati c
a
x, y [1, 1].

(1 x)(1 + y) +

(1 + x)(1 y)

4 (x + y)2 ,

Rodica Pop si Ovidiu Pop, Satu Mare


Solutie. Inegalitatea din enunt se scrie echivalent:
(1 x)(1 + y) + (1 + x)(1 y) + 2
0 2 x2 y 2 2

(1 x2 )(1 y 2 ) 4 (x + y)2
p

(1 x2 )(1 y 2 ) 0 ( 1 x2

1 y 2 )2 ,

inegalitate adevarata. Egalitatea are loc daca si numai daca y = x.

VIII.132. Dac
a a, b, c R+ si a2 + ab + b2 + a2 + ac + c2 + b2 + bc + c2 =

3(a + b + c), ar
atati c
a a = b = c.
D.M.
atinetu-Giurgiu, Bucuresti
B

3
Solutie. Se arata usor c
a a2 + ab + b2
(a + b), egalitatea realizandu-se
2
P
c
and a = b. Scriind inegalitatile similare si sumand, deducem c
a
a2 + ab + b2

3(a + b + c). Cum se atinge egalitatea, rezulta c


a a = b = c.
(1 + x)(1 + y) (1 + y)(1 + z) (1 + z)(1 + x)

.
+
+

1 + xy
1 + yz
1 + zx
a) Ar
atati c
a E(x, y, z) x + y + z + 3, x, y, z (0, 1).
b) Ar
atati c
a E(x, y, z) x + y + z + 3, x, y, z (1, ).
Ion Nedelcu, Ploiesti si Liviu Smarandache, Craiova
(1 + x)(1 + y)
x+y
Solutie. a) Dac
a x, y (0, 1), atunci
1+
(dupa calcule,

1 + xy
2

2
a doua inegalitati
aceasta inegalitate revine la ( x y) ( xy 1) 0). Scriind nc
analoage si adun
and membru cu membru, obtinem c
a E(x, y, z) x + y + z + 3, cu
egalitate c
and x = y = z.
VIII.133. Fie E(x, y, z) =

171


(1 + x)(1 + y)

b) Pentru x, y (1, ), avem c


a ( x y)2 ( xy 1) 0, deci

1 + xy
x+y
etc.
1+
2
2
VIII.134. Dac
a a, b, c R+ sunt laturile unui triunghi, demonstrati c
a aa +2ac
b

b2 +2ab

c2 +2bc

<

a+b+c
2

(a+b+c)2

R
azvan Ceuc
a, elev, Iasi
Solutie. Imp
artind ambii membri prin cantitatea din dreapta, inegalitatea de
demonstrat devine

2a
a+b+c

a2 +2ac

2b
a+b+c

b2 +2ab

2c
a+b+c

c2 +2bc

< 1.

2a
< 1. Exponentul
a+b+c
2
a + 2ac fiind strict pozitiv, obtinem c
a primul factor al produsului precedent este
subunitar. Proced
and analog pentru ceilalti doi factori, ncheiem solutia problemei.
VIII.135. Fie n 2 un num
ar natural fixat si x1 , x2 , . . . , xn numere reale pozitive
astfel nc
at x1
+ x2 + . . . + x
asiti cea
mai mic
a si cea mai mare valoare ale
n = n. G
expresiei E = x1 + x1 x2 + x2 + x2 x3 + . . . xn + xn x1 .
Gheorghe Iurea, Iasi
Solutie. Folosind inegalitatea mediilor M G M A, obtinem:

Cum a < b + c (inegalitatea triunghiului), rezulta c


a

2 = 2x1 (x2 + 1) + 2x2 (x3 + 1) + . . . + 2xn (x1 + 1)


1
[(2x1 + x2 + 1) + (2x2 + x3 + 1) + . . . + (2xn + x1 + 1)] =
2

1
= [3(x1 + x2 + . . . + xn ) + n] = 2n E n 2.
2

Pentru x1 = x2 =. . . = xn = 1, avem c
a x1 + x2 + . . . + xn = n si E = n 2, prin
urmare Emax = n 2.

n, x2 = . . . =
Evident c
a E x1 + x2 + . . . + xn = nsi, cum pentru x1 =
xn = 0 avem c
a x1 + x2 + . . . + xn = n si E = n, rezulta c
a Emin = n.
VIII.136. Fie m, n Z, m < n si x1 , x2 , . . . , xnm numere ntregi dinstincte
dou
a c
ate dou
a, cuprinse ntre 2m si 2n. Demonstrati c
a printre numerele x1 , x2 , . . . ,
xnm ori exist
a unul egal cu m + n, ori exist
a dou
a av
and suma 2m + 2n.
Dan Nedeianu, Drobeta Tr. Severin
Solutie. Cum 2m < xk < 2n, k = 1, n m, atunci 2m < 2m + 2n xk < 2n,
k = 1, n m. Astfel, x1 , x2 , . . . , xnm , 2m+2nx1, 2m+2nx2, . . . , 2m+2nxnm
sunt 2n 2m numere ntregi cuprinse ntre 2m si 2n, primele n m si ultimele n m
fiind distincte. Insa ntre 2m si 2n exista exact 2m 2n 1 numere ntregi, prin
urmare doua dintre numerele de mai sus vor fi egale: exista i, j {1, 2, . . . , n m}
astfel nc
at xi = 2m + 2n xj . In cazul n care i = j, avem c
a xi = m + n. Dac
a
i 6= j, atunci xi + xj = 2m + 2n.

172

Clasa a IX-a
IX.116. Rezolvati n numere ntregi ecuatia 8x3 + y 3 + 12xy = 8.
Vasile Chiriac, Bac
au
Solutie. Cu notatiile s = 2x + y, p = 2xy, ecuatia data se scrie succesiv:
(2x + y)(4x2 2xy + y 2 ) + 12xy 8 = 0 s(s2 3p) + 6p 8 = 0

(s 2)(s2 + 2s + 4) 3p(s 2) = 0 (s 2)(s2 + 2s + 4 3p) = 0.

Dac
a s = 2, obtinem solutiile x = m, y = 2 2m, m Z. Dac
a s 6= 2, atunci
.
(s + 1)2 + 3 2p = 0, deci s + 1..3. Rezult
a c
a s = 3k 1, k Z, apoi p = 3k 2 + 1.
2
Numerele 2x si y vor fi solutiile ecuatiei t (3k1)t+(3k 2 +1) = 0, cu discriminantul
= 3(k + 1)2 < 0, prin urmare n acest caz nu obtinem solutii ntregi. In final,
ecuatia dat
a are solutiile (m, 2 2m), m Z.

IX.117. Determinati functiile f : N N cu proprietatea c


a f (x + y) = f (x) +
f (y) + 3xy(x + y), x, y N.
Mih
aly Bencze, Brasov
Solutie. O solutie evident
a este functia f0 (x) = x3 , x N. Dac
a f verific
a
ecuatia functional
a din enunt, functia g(x) = f (x) x3 , x N, verific
a ecuatia
Cauchy g(x + y) = g(x) + g(y), x, y N. Atunci g(x) = x, unde = g(1) N,
prin urmare f (x) = x3 + x, x N, unde N.
IX.118. Dac
a x, y, z R+ , demonstrati c
a are loc inegalitatea
X xy

cyclic

x+y

X
cyclic

z xy
.
(x + z)(y + z)

Marian Tetiva, B
arlad

z xy
yz

xz
+
4
este echivalent
a cu x(y +
Solutie. Relatia
x+z
y+z
(x + z)(y + z)

z) + y(x + z) 4 xyz si, cum y + z 2 yz si x + z 2 xz, are loc inegalitatea


anuntata. Scriem nc
a doua inegalitati analoage si, prin adunare, rezulta inegalitatea
din enunt.
IX.119. Fie P un punct n interiorul triunghiului echilateral ABC si P1 , P2 , P3
proiectiile lui pe laturile triunghiului. Dac
a G este centrul de greutate al triunghiului
ABC, demonstrati c
a mijlocul G1 al segmentului P G este centru de greutate pentru
triunghiul P1 P2 P3 .
Petru Asaftei, Iasi
Solutie. Vom folosi urmatoarele propriet
ati uzuale:

(1) G este centrul de greutate al ABC GA + GB + GC = 0 ;



(2) Dac
a P IntABC si G este centrul de greutate al ABC, atunci P A + P B +

P C = 3P G;
(3) Dac
a ABC este echilateral atunci, cu notatiile din enunt, are loc egalitatea


P A + P B + P C = 2(P P1 + P P2 + P P3 ).
173





Intrucat
G1 P1 + G1 P2 + G1 P3 = (G1 P + P P1 ) + (G1 P + P P2 ) + (G1 P + P P3 ) =

(3) 3 1 (2) 3 3

3G1 P + (P P1 + P P2 + P P3 ) = GP + (P A + P B + P C) = GP + P G = 0 ,
2
2
2
2
din (1) rezulta c
a G1 este centrul de greutate al triunghiului P1 P2 P3 .

IX.120. Fie P un punct n interiorul triunghiului ABC astfel nc


at P
AB

P
BC P
CA. Dac
a AB AP + BC BP + CA CP = 0 , ar
atati c
a ABC este
echilateral.
Claudiu M
andril
a, elev, T
argoviste

Solutie. Fie masura comun


a a unghiurilor din enunt. Vectorii AB AP si

AP AB au acelasi modul, deci se obtin unul din cel


alalt printr-o rotatie de unghi
. Ration
and analog pentru ceilalti doi termeni ai sumei, rezulta c
a vectorul AP

AB + BP BC + CP CA se obtine din AB AP + BC BP + CA CP printr-o

rotatie de unghi , prin urmare AP AB + BP BC + CP CA = 0 . De aici,

(AP P B) AB + (BP P C) AC = 0 , asadar P A = P B = P C si atunci P

b ns
este centrul cercului circumscris ABC. In aceste conditii, m(BP
C) = 2m(A);
a

] = 180 m(C)
=
m(BP
C) = 180 m(P
BC) m(P
CB) = 180 [m(C)
b + m(B)
si deducem c
b B.
Analog se arat
= C,
adic
aA
a c
aB
a ABC este
m(A)
echilateral.

Clasa a X-a

x
X.116. O urn
a contine x bile rosii si n x bile verzi, unde x, n N , x < n.
3
x
bile rosii si
bile verzi sunt marcate cu 1, iar celelalte bile sunt marcate cu 2. Din
2
urn
a se extrage o bil
a si se consider
a evenimentele A: obtinem o bil
a rosie si B:
obtinem o bil
a marcat
a cu 1.
a) Determinati n si x pentru care A si B sunt evenimente independente.
b) Aflati P (B|A).
Laurentiu Modan, Bucuresti
..
..
x
Solutie. Se impune ca x . 3 si x . 2, deci x = 6k, k N , iar n x > , asadar
2


2n
x 1, 2, . . . ,
.
3
a) Evenimentele A si B sunt independente c
and P (A) P (B) = P (A B), adica
x
x 5x

=
. Obtinem c
a 5x = 2n, prin urmare x = 6k, n = 15k, k N .
n 6n
3n
P (A B)
nx
b) Avem c
a P (B|A) =
, unde P (A) = 1P (A) =
, iar P (AB) =
n
P (A)
x
nx 2
2n 3x
2n 3x
=
, deci P (B|A) =
.
n
2n
2(n x)

X.117. Demonstrati c
a n + 1 n > 3 n + 1 3 n, n N .
Ionut Iv
anescu, Craiova
Solutie. Amplific
and n ambii membri
cu
expresiile
conjugate,
observam ca ar fi

3
suficient sa dovedim c
a n + 1 + n < 3 (n + 1)2 + 3 (n + 1)n + n2 ; vom arata
174

3
a n < 3 (n + 1)n + n2 (2). Ridicand ambii
c
a n + 1 3 (n + 1)2 (1) si c
membri la puterea a sasea, (1) revine
la (n + 1)3 (n
+ 1)4 , adevarat pentru n N .

3
6
Apoi, din inegalitatea mediilor, 3 n(n + 1) + n2 2 6 n3 (n + 1) > n3 = n, deci
are loc si (2) si cu aceasta solutia problemei este complet
a.
X.118. Determinati n N pentru care

sin2n x + cos2n x + n sin2 x cos2 x = 1,

x R.

Ani Dr
aghici si Ileana M
andruleanu, Craiova
1 n
1

Solutie. Pentru x = , relatia din enunt devine n + n + = 1, sau 1 + (n 4)


4
2
2
4
n3
2
= 0. Aceasta egalitate nu poate avea loc pentru n 4. Pentru n = 0, conditia
din ipoteza devine 2 = 1, fals. Pentru n = 1, obtinem c
a 1 + sin2 x cos2 x = 1, x R,
imposibil. Se verific
a imediat c
a identitatea din enunt este adevarata pentru n = 2 si
n = 3, valori care constituie solutiile problemei.
X.119. Demonstrati c
a pentru orice z1 , z2 , z3 C are loc inegalitatea
|z1 + z2 |2 + |z2 + z3 |2 + |z3 + z1 |2 |z1 + z2 + z3 |2 + Im(z1 z2 + z2 z3 + z3 z1 ).
Dan Nedeianu, Drobeta Tr. Severin
Solutie. Se stie (sau se arata usor) c
a |z1 + z2 |2 + |z2 + z3 |2 + |z3 + z1 |2 |z1 +
z2 + z3 |2 = |z1 |2 + |z2 |2 + |z3 |2 , deci ram
ane sa dovedim c
a
()

|z1 |2 + |z2 |2 + |z3 |2 Im(z1 z2 + z2 z3 + z3 z1 ).


P

|z1 iz2 |2 0, care se scrie sub forma (z1


Pornim de la inegalitatea evident
a
P
P
a z z = 2iP
Im z, z C,
iz 2 )(z 1 + iz2 ) 0, sau 2 |z1 |2 + i(z1 z2 z1 z 2 ) 0. Ins
P
prin urmare i(z z) = 2Im z, z C, deci obtinem c
a 2 |z1 |2 2 Im z1 z2 0,
adica tocmai inegalitatea (*).

X.120. Fie ABCD un patrulater nscris n cercul C(M, R), av
and m(BOC)
90 , unde {O} = AC BD. Demonstrati c
a

AB 2 BC 2 + CD2 DA2 AC BD 4R2 .


C
and se realizeaz
a egalitatea?
Florin St
anescu, G
aesti
Solutie. Not
am: AB = a, BC = b, CD = c, DA = d, AC = e, BD = f ,
= 2x, m(BC)
= 2y, m(CD)
= 2z, m(DA)
= 2t. Cu teorema sinusurilor
m(AB)
n triunghiurile ABC si ACD, obtinem c
a a = 2R sin x, b = 2R sin y, c = 2R sin z,
d = 2R sin t. Cum ABCD este inscriptibil, ef = ac + bd (teorema lui Ptolemeu) si
atunci inegalitatea de demonstrat devine
4R2 (sin2 x sin2 y + sin2 z sin2 t) 4R2 (sin x sin z + sin y sin t 1)

cos(x z) cos(x + z)
1 cos 2x 1 cos 2y 1 cos 2z 1 cos 2t

+
2
2
2
2
2
175

cos(y t) cos(y + t)
1 (cos 2y + cos 2t) (cos2x + cos 2z)
2
cos(x z) + cos(y t) 2 2 cos(y + t) cos(y t) 2 cos(x + z) cos(x z)

cos(x z) + cos(y t) 2 cos(y t)[2 cos(y + t) 1] + cos(x z)


yt+xz
ytx+z
[2 cos(y + t) 1] 2 [2 cos(y + t) 1] 2 cos
cos

2
2
2 [2 cos(y + t) 1] sin(t + z) sin(y + z) 1.
(Pe parcurs am folosit faptul c
a x + y + z + t = .) Observam c
a 2 cos(y + t) 1 =

2 cos(BOC) 1 [1, 1], ntrucat cos(BOC) [0, 1], apoi sin(t + z) = sin B (0, 1],
sin(z + y) = sin A (0, 1], deci ultima inegalitate este adevarata.
= 0, sin B = 1 si sin A = 1, adic
=
Se realizeaz
a egalitatea c
and cos(BOC)
a BOC
b


atrat.
90 , B = 90 , A = 90 , prin urmare n cazul n care ABCD este p

Clasa a XI-a
XI.116. a) Fie R+ si A M2 (R) astfel nc
at det(I2 + A2 ) = 0; demonstrati
c
a det A = .
b) Ar
atati c
a exist
a R si A M2 (R) astfel nc
at det(I2 + A2 ) = 0 si
detA 6= .
Dan Popescu, Suceava

a b
Solutie. a) Fie A =
M2 (R), iar u = a + d = tr A, v = ad bc = det A.
c d

ua + ( v)
ub
,
Cum A2 = uAvI2 , atunci I2 +A2 = uA+(v)I2 =
uc
ud + ( v)
prin urmare det(I2 + A2 ) = u2 v + ua( v) + ud( v) + ( v)2 + ( v)2 =
u2 + ( v)2 . Deoarece u2 , , ( v)2 R+ , suma precedent
a este nula numai daca
v = 0, adica det A = .

1 0
b) De exemplu, putem considera A =
si = 1; atunci det (I2 + A2 ) =
0 2


0 0


0 3 = 0, iar detA = 2 6= .

1
a si
XI.117. Dac
a a, b R+ , ar
atati c
a ax ln(bx + 1), x , dac
b
numai dac
a a = b.
Dumitru S
avulescu, Bucuresti
Solutie. Dac
a a = b, cu notatia y = ax (1, ), ar trebui sa dovedim c
ay
1
ln(y + 1), fapt binecunoscut. Reciproc, fie f : ( , ) R, f (x) = ax ln(bx + 1).
b

1
Cum f (x) 0 = f (0), x , , rezulta c
a x = 0 este punct de minim local al
b
b
, deci f (0) = a b si deducem
lui f , prin urmare f (0) = 0. Dar f (x) = a
bx + 1
c
a a = b.
XI.118. Fie 1 , 2 , 3 , 4 numere reale pentru care limita l = lim (1 {x} +
x

176

1
2
+ 3 x +
+ 4 {3x}) este finit
a (unde {} desemneaz
a partea fractio2
3
nar
a). Ar
atati c
a 1 + 2 + 3 + 34 = 0.
Marius Dr
agan, Bucuresti
Solutie. Se stie c
a o functie continu
a si periodic
a ce
are
limita
finit

a la +

1
2
+ 3 x +
+
este constant
a; n cazul nostru, deducem c
a 1 {x} + 2 x +
3
3
2
1
4 {3x} = c, x R, unde c este o constant
a. Dand lui x valorile 0, si , obtinem
3
3
c
a 2 + 23 = 1 + 22 = 21 + 3 = 3c, de unde 1 = 2 = 3 = c. Atunci
1
2
c({x} + x +
+ x+
1) + 4 {3x} = 0 si, folosind identitatea lui Hermite,
3
3
rezulta c
a (c + 4 ){3x} = 0, adica 4 = c si de aici concluzia problemei.
2 x +

XI.119. Se consider
a sirul (un )n0 definit prin u0 = 1, u1 = 2, un+2 = 1 +
1
un+1

, n N . Ar
atati c
a sirul este convergent si aflati-i limita.
arctg
4
un
Moubinool Omarjee, Paris

1
3

= 1 + arctg 1;
Solutie. Observam c
a un 1,
, n 2. Fie a = 1 +
2
16
4
n
3
vom demonstra prin inductie c
a |un a| <
, n 2. Avem c
a |u2 a| =
4
2

1
u2
1 u2
1
1
3

| arctg 2arctg 1| |21| <
, |u3 a| = | arctg arctg 1| 1 =
4
4
4
4
2
4
2
3
1
1
1
3
| arctg 2 4| =
(4 arctg 2) <
4
. Presupunand relatia adevarata
32
32
32
4





1
un1
1 un1
arctg 1
1 <
pentru n 1 si n 2, obtinem |un a| = arctg
4
u
4 un2
 n2

n2 

1
1
3 n1
3
1 3 n2
1
|un1 un2 | (|un1 a| + |un2 a|) <
+

=
4
4
4
4
4
4 4
n

n
3
3
7
, ceea ce ncheie justificarea inegalitatii anuntate. Cum lim
= 0,
<
n
4
4
4

rezulta c
a lim un = a = 1 + .
n
16
XI.120. Fie k, a, b (0, ), a < b si sirurile (xn )n1 , (yn )n1 definite prin
k 2 + kxn + xn yn
k 2 + kyn + xn yn
x1 = a, y1 = b, xn+1 =
, yn+1 =
, n N .
yn
xn
k 2 + 2ka + ab
, n N .
a) Demonstrati c
a xn <
ba
b) Ar
atati c
a sirurile date au limite si determinati aceste limite.
Lucian Tutescu si Mircea Tereujanu, Craiova
(k + xn )(k + yn )
1
yn
Solutie. a) Avem c
a xn+1 =
k,
=
yn
k + xn+1
(k + xn )(k + yn )
xn
1
yn xn
1
1
=

=
. Atunci
=
si analog
k + yn+1
(k + xn )(k + yn )
k + xn+1 k + yn+1
(k + xn )(k + yn )
1
1
1
1

, prin urmare sirul an =

este constant. Cum


k + xn
k + yn
k + xn
k + yn
177

1
ba
1
ba
1
1
=
>

=
, rezulta c
a
+
k+a k+b
(k + a)(k + b)
k + xn
(k + a)(k + b) k + yn
(k + a)(k + b)
ba
, ntrucat yn > 0, n N . Deducem c
a xn + k <
, de
(k + a)(k + b)
ba
k 2 + 2ka + ab
unde xn <
, c.c.t.d.
ba
k 2 + kxn
k 2 + kyn
b) Deoarece xn+1 xn =
> 0 si yn+1 yn =
> 0, sirurile din
yn
xn

problem
a sunt strict cresc
atoare, deci au limita: l = lim xn , l = lim yn , cu l, l > 0.
a1 =

n


k 2 + 2ka + ab
, deci l R+ . Dac
a, prin
S
irul (xn )n1 este marginit: xn a,
ba
2
k + kxn + xn yn
absurd, l R+ , prin trecere la limita n relatia xn+1 =
rezulta c
a
yn
2

k + kl + ll
l=
, de unde k 2 + kl = 0; atunci l = k < 0, fals. R
am
ane c
a l = +.
l
ba
1
1
=
, obtinem c
a
+
Trec
and acum la limit
a n relatia
k + xn
(k + a)(k + b)
k + yn
ba
k 2 + 2ka + ab
1
=
, deci l =
.
k+l
(k + a)(k + b)
ba

Clasa a XII-a

XII.116. Determinati functia f : Z8 Z8 cu proprietatea c


a

f (3x) + f (5x) + f (7x) = 3x2 + 5x + 6, x Z8 .

Bogdan Chiriac, student, Iasi


Solutie. Inlocuim succesiv pe x cu 3x, 5x si 7x n relatia din enunt; adunand cele
trei egalitati obtinute si tinand seama de ipoteza, obtinem c
a 3f (x)+2(3x2 +5x+6) =
2
2
x + 3x + 2. Prin nmultire cu 3, rezulta c
a f (x) = x + 3x + 2, x
Z Z8 .

XII.117. Aflati n N pentru care exist


a R astfel nc
at

(2t3 + 3)dt = n.

Romeo Cernat, Iasi




t4
5

Solutie. Avem c
a
= 23 32 + 2 + .
(2t + 3)dt =
+ 3t
2
2
1
1

5
Ecuatia din enunt se scrie sub forma f () = 0, unde f () = 23 32 +2+
n .
2
Intrucat f () = 62 6 + 2 6= 0, R, ecuatia f () = 0 are o unic
a solutie
real
a, iar aceasta se afla n intervalul (, 0) daca si numai daca f (0) > 0. Conditia
5
f (0) = n > 0 conduce la valorile c
autate ale lui n, anume n {0, 1, 2}.
2
Z
Z

XII.118. Fie f : [0, 1] R o functie continu


a cu proprietatea c
a

f (x)dx =
0

a+1
, unde a (0, 1) este dat. Demonstrati c
a exist
a x0 (0, 1) astfel nc
at
a
1
1
.
< f (x0 ) <
x0 + 1
x0
Bogdan Victor Grigoriu, F
alticeni

ln

178

1
1
a+1
Solutie. Cum
f (x)
dx = ln
, functia f are proprietatea c
a
a
0 x+a
0

1
1
dx = 0. Fie g(x) = f (x)
, x [0, 1] functie continu
a, iar G o primitiva
x+a
x+a
sa; atunci G(1) = G(0) si, cum G este derivabila pe [0, 1], sunt ndeplinite conditiile
teoremei lui Rolle. Obtinem c
a exista x0 (0, 1) pentru care G (x0 ) = 0, adica
1
1
1
1
, de unde
. Deoarece a (0, 1), rezulta c
a
<
<
f (x0 ) =
x0 + a
x0 + 1
x0 + a
x0
cerinta problemei.

(n2 + 12 )(n2 + 22 ) . . . (n2 + n2 )

e2.
e2
Adrian Corduneanu, Iasi
Solutie. Consider
am functia continu
a, deci integrabila, f : [0, 1] R, f (x) =
XII.119. Demonstrati c
a lim

n2

ln(1 + x ). Integrand prin p


arti, obtinem c
aI=
Z

1

ln(1 + x )dx = x ln(1 + x )
2

x2
1
dx = ln 2 2 1
dx = ln 2 +
2. Considerand sirul
2
2
2
2
0 1+x
0 1+x
0
1
n
de diviziuni n = (x0 = , x1 = , . . . , xn = ) cu sirul normelor tinzand la
n
n
n 




n
1
1 P
12
22
n2
f (xk ) = ln 1 + 2
zero, observ
am c
a sn =
1 + 2 ... 1 + 2
=
n k=1
n
n
n
n

(n2 + 1)(n2 + 22 ) . . . (n2 + n2 )

este un sir de sume Riemann a c


arui limita este
n2
2

egala cu I. Astfel, limita sirului din enunt este eI = 2 e 2 .


e
XII.120. Fie f : R (0, ) o functie continu
a cu lim f (x) = 0, astfel nc
at

ln

x Z

oricare ar fi n N , exist
a un unic xn R+ pentru care

sin f (t)
xn
dt. Calculati lim
.
n n
f (t)

xn

ln(1 + f (t))
dt =
f (t)

Florin St
anescu, G
aesti
sin f (t)
dt, n N . Folosind lema Stolzf (t)
0
Z n+1
sin f (t)
sin f (cn )
Ces`aro si teorema de medie, obtinem lim an = lim
dt = lim
,
n
n n
n f (cn )
f (t)
cu n < cn < n + 1. Cum cn si lim f (x) = 0, rezulta c
a lim an = 1.
1
Solutie. Consider
am sirul an =
n

Utiliz
andZacum teorema lui Lagrange si inegalitatea ln(1 + t) t, t [0, ),
xn
ln(1 + f (t))
ln(1 + f (yn ))
dt = F (xn ) F (0) = xn F (yn ) = xn
<
deducem c
a
f (t)
f (yn )
0
ln(1 + f (t))
xn 1 = xn , n N (unde F (t) este o primitiva a functiei
. Astfel,
f (t)
Z xn
Z n
sin f (t)
ln(1 + f (t))
dt =
dt = n an , n N , prin urmare
xn >
f
(t)
f (t)
0
0
179

lim xn = .

Intrucat lim f (x) = 0, avem c


a (1 + f (t)) f (t) > 2 pentru t > t0 convenx
Z x

x
ln(1 + f (t))
ln 2 c
and x . In calculul limitei
abil ales, deci
>
f (t)
0
0
Z x
ln(1 + f (t))
1

lim
dt vom putea aplica regula lui l Hospital (cazul ) si obtinem
x x 0
f (t)

ln(1 + f (x))
= 1.
c
a limita este egala cu lim
x
f (x)
Z n
Z xn
sin f (t)
ln(1 + f (t))
xn 1
In relatia 1
dt =

dt facem pe n sa tind
a la
n 0
f (t)
n xn 0  f (t) 
xn
xn
= 1.
si, conform celor de mai sus, rezulta c
a 1 = lim
1, asadar lim
n n
n n

(continuare de la pag. 147)


2. Asteroizi cu nume de rom
ani - toti acestia fac parte din centura principala de
asteroizi situata ntre Marte si Jupiter
2331 Parvulesco (Constantin P
arvulescu (1890-1945) si copii sai Carina
si Antares), cu diametrul estimat ntre 11 si 24 km si cu o rotatie n jurul
Soarelui la 3,78 ani;
4268 Grebenikov (Eugeniu Grebenicov, n. 1932, Basarabia), cu diametrul 5-12 km si o rotatie n jurul Soarelui n 4,28 ani;
6429 Br
ancusi, cu diametrul de 4-9 km si o rotatie n 3,16 ani;

9253 Oberth (Herman Iulius Oberth, 1894-1989), cu diametrul p


ana la
6 km;
9403 Sanduleak (Nicolae Sanduleak, 1933-1990), cu diametrul p
ana la
6 km, o rotatie n 4 ani;

9493 Enescu, cu diametrul p


ana la 9 km;

9494 Donici (Nicolae Donici, 1875-1957, membru fondator al UAI);

9495 Eminescu, cu diametrul p


ana la 6 km si o rotatie n 3,23 ani;
10 034 Birlan (Mirel Brlan, n. 1963, Giurgiu);

12 498 Dragesco (Jean Dragesco, n. 1920), cu diametrul de cel mult


10 km.

3. Comete cu nume de rom


ani
Cometa Daimaca 1943c (Victor Daimaca, 1892-1969, Turnu Severin);

Cometa van Gent-Peltier-Daimaca 1943W1.


180

Solutiile problemelor pentru preg


atirea
concursurilor propuse n nr. 1/2011
A. Nivel gimnazial
G196. Fie M multimea numerelor naturale nenule scrise numai cu cifre pare,
care au cel mult 2011 cifre. Ar
atati c
a suma inverselor elementelor lui M este mai
mic
a dec
at 4.
Cecilia Deaconescu, Pitesti
Solutie. Pentru fiecare n {1, 2, . . . , 2011}, multimea M contine 45n1 elemente
. . 0} =
care se scriu cu n cifre pare si orice astfel de element este cel putin egal cu 2 0| .{z
n1 de 0

2 10n1 . Not
and cu Sn suma inverselor numerelor de n cifre din M , avem c
a Sn <

2011
2011
P
P
P
1
1
1
1
1
=
.
Atunci
=
4
=
S
<
45n1
1

< 4,
n
n2
2 10n1
2n2
22011
n=1
n=1 2
xM x
ceea ce ncheie rezolvarea.
G197. Determinati cea mai mare putere a lui 3 care divide num
arul N = 162011
2011
2011
2011
28
+34
22
+ 1.
Pedro H.O. Pantoja, Brazil
Solutie. Cum 29 = 1(mod 27) si 2011 = 223 9 + 4, atunci 22011 = (29 )223 24
16 11(mod 27), deci 42011 121 13(mod 27), 82011 13 11 8(mod 27), iar
162011 132 7(mod 27). Rezult
a c
a N 7 2 8 + 3 13 2 11 + 1 9(mod 27),
prin urmare N = 27k + 9, k N. Astfel, N se divide cu 32 , dar nu cu 33 , adica cea
mai mare putere a lui 3 care divide pe N este 32 .
G198. Rezolvati n numere naturale ecuatia 6n + 2800 = m6 .
Andrei Eckstein, Timisoara
n
n
Solutie. Deoarece 6 = (7 1) = M7 + (1)n , 2800 = M7 si orice p
atrat
perfect d
a la mpartire prin 7 unul dintre resturile 0, 1, 2 sau 4, deducem c
a n este
numar par, deci n = 2k, k N. Ecuatia devine (m3 6k )(m3 + 6k ) = 2800, de
unde 6k 2800, adica k {0, 1, 2, 3, 4}. Inlocuind n ecuatia initiala, obtinem c
a
m6 {2801, 2836, 4096, 49456, 1682416} si convine doar varianta m6 = 4096 = 46 . In
concluzie, unica solutie a ecuatiei date este m = n = 4.
G199. Determinati b N pentru care exist
a a N astfel nc
at a2 + ab + b2 s
a
fie p
atrat perfect.
Gheorghe Iurea, Iasi

2
2
2
Solutie. Fie a, b N pentru care a + ab + b p = 0, cu p N ; atunci
= 4p2 3b2 = q 2 , cu q N, prin urmare 3b2 = (2p q)(2p + q), cu p, q N. In
cazul n care b = 2t + 1, t N , scrierea precedent
a este posibila alegand, de exemplu,
2p q = 1, 2p + q = 3b2 , adica p = 3t2 + 3t + 1 si q = 6t2 + 6t + 1. Rezult
a c
a pentru
orice b 3 impar, b = 2t + 1, t N , exista a = 3t2 + 2t astfel nc
at a2 + ab + b2 sa
fie p
atrat perfect. Proced
and analog, pentru b = 4t, t N, t 2, g
asim (de exemplu)
a = 3t2 2t 1, iar pentru b = 4t + 2, t N , g
asim a = 6t2 + 4t astfel nc
at sa fie
ndeplinit
a cerinta problemei.
181

Dac
a b = 1, atunci a2 + ab + b2 = a2 + a + 1 nu este p
atrat c
and a N , ntrucat
2
2
2
2
2
a < a + a + 1 < (a + 1) . Dac
a b = 2, atunci a + ab + b = a + 2a + 4 nu este p
atrat
perfect pentru a N , deoarece (a + 1)2 < a2 + 2a + 4 < (a + 2)2 . Dac
a b = 4, atunci
a2 + ab + b2 = a2 + 4a + 16 are proprietatea c
a (a + 2)2 < a2 + 4a + 16 < (a + 4)2 si
2
2
a + 4a + 16 = (a + 3) , deci iarasi nu este p
atrat perfect.

In concluzie, valorile c
autate sunt b N \{1, 2, 4}.
2

G200. Ar
atati c
a
[0, 1].

a2
b2
c2
1
+
+
, a, b, c
3
3
3
3
3
3
2b + 2c + 5 2a + 2c + 5 2a + 2b + 5
3

Dan Nedeianu, Drobeta Tr. Severin


a2

Solutie. Intrucat c3 [0, 1], rezulta c


a 2c3 + 3 5, deci
3
2b + 2c3 + 5
3
2
P
a
a
. Scriind si analoagele, dupa sumare obtinem c
a

3
3
3
3
2(a + b + c ) + 3
2b + 2c3 + 5
a2 + b 2 + c2
. Dar 2a3 + 1 3a2 (echivalent cu (a 1)2 (2a + 1) 0, adevarat
3
2(a + b3 + c3 ) + 3
c
and a [0, 1]) si, similar, 2b3 +1 3b2 , 2c3 +1 3c2 , prin urmare 2(a3 +b3 +c3 )+3
a2 + b 2 + c3
1
3(a2 + b2 + c2 ), adica
, si de aici concluzia problemei.
2(a3 + b3 + c3 ) + 3
3
G201. Se consider
a triunghiul ABC cu AC 6= BC > AB. Dac
a exist
a un punct
1
D (BC) pentru care AB 2 = BD BC si AD2 = BD DC, ar
atati c
a
3
AB 2
1.
BC 2 AC 2
Ovidiu Pop, Satu Mare
c2
si AD2 = BD(a BD)
Solutie. Cu notatiile uzuale n triunghi, din BD =
a
c 2
obtinem c
a AD =
a c2 . Pe de alt
a parte, folosind formula lui Heron, avem c
a
a

2S
1
ha =
2a2 b2 + 2b2 c2 + 2c2 a2 a4 b4 c4 . Intrucat AD ha , rezulta c
a
=
a
2a
2 2
4
2 2
2 2
2 2
4
4
4
2
2
2
2
2
2
4a c 4c 2a b +2b c +2c a a b c , adica (3c +a b )(c +b a ) 0.
c2
1
Dac
a a < b, relatia precedent
a conduce la 3c2 + a2 b2 0, de unde 2

b a2
3
c2
1
< 0 < 1. Dac
a a > b, obtinem c
a c2 + b2 a2 0, prin urmare
si atunci 2
3
a b2
1
c2
<0< 2
1 si cu aceasta solutia este complet
a.
3
a b2
G202. Fie ABCD p
atrat, iar punctele M si N pe laturile AB, respectiv AD
sunt astfel nc
at AM = DN = k AB. Not
am {P } = CN DM si {Q} = AP
CD. Determinati valorile lui k pentru care P Q este bisectoare, respectiv median
a n
triunghiul CDP.
Neculai Roman, Mircesti (Iasi)
Solutie. Se arata usor c
a DM CN. Folosind teorema catetei n CDN ,
182

Q
D
NP
DN 2
C
2
obtinem c
a
= k . Cu teorema lui Menelaus n
=
PC
CD2
N P CQ DA
P

= N
CDN (transversala QP A) deducem c
a
P C QD AN
2
CQ
1
QD
k
1, deci k 2

= 1, prin urmare
=
.
QD k 1
QC
1k
M
A
B
PD
Din asem
anarea P CD DCN obtinem c
a
=
PC
DQ
PD
DN
= k. Atunci P Q este bisectoare n CDP daca si numai daca
=
,
DC
QC
PC
1
k2
= k, de unde k = 0 (care nu convine) sau k = , care este valoarea
adica
1k
2
k2
c
autata. Apoi, P Q este mediana n CDP daca si numai daca
= 1, cu solutia
1k

1 + 5
.
acceptabil
ak=
2
G203. Fie date numerele reale a si b cu a < b < 2a. Triunghiurile isoscele
ABC si A B C au aceeasi ax
a de simetrie, aceeasi dreapt
a suport a bazelor, iar
BC = a, AB = AC = b, B C = b, A B = A C = a. Dac
a {M } = AB A B ,
{N } = AC A C , ar
atati c
a M N este linie mijlocie n ABC dac
a si numai dac
a
a3 + b3 = 2a2 b.
Temistocle Brsan, Iasi
Solutie. Din motive de simetrie, avem c
a M N kBC. Not
am cu D mijlocul comun

A
al segmentelor
BC

s
i
B
C

s
i
fie
h
=
AD,
iar
h
=
A
D;
evir
r
2
2
a
b
dent c
a h = b2 , h = a2 . Aplic
and teorema lui
4
4

A
Menelaus n ABD, cu transversala A M B , obtinem c
a
MB
ba
h
M
N
si, cum M A + M B = b, deducem c
a
=

MA
b
h h
2

b (h h )
C C
D
AM =
. Din faptul c
a AM N ABC rezulta c
a B B
bh ah

ab( 4b2 a2 4a2 b2 )


ab(h h )
a

=
. Conditia ca M N sa fie
M N = AM =

b
bh ah
b 4b2 a2 a 4a2 b2

2
2
2
2
linie
ABC este 2M N
= a, ceea ce revine la 2b(
mijlocie n
4b a 4a b ) =
2
2
2
2
2
2
2
2
b 4b a a 4a b , adica b 4b a = (2b a) 4a b ; dupa ridicare la
p
atrat si reduceri, se obtine a3 + b3 = 2a2 b.
Analog se poate ar
ata c
a M N este linie mijlocie n A B C daca si numai daca
3
3
2
a + b = 2ab .
G204. Despre un punct de pe o muchie a unui tetraedru vom spune c
a este punct
bisector dac
a el este piciorul a dou
a bisectoare ale unor unghiuri ale fetelor. Ar
atati
c
a num
arul punctelor bisectoare ale unui tetraedru este 0, 2 sau 6.
Silviu Boga, Iasi
Solutie. Pe fiecare muchie exista cel mult un punct bisector. Dac
a M (AB) este
MA
CA
DA
punct bisector, atunci CM si DM sunt bisectoare, deci
=
=
. Rezult
a
MB
CB
DB
183

AC
BC
si CBD
se vor nt
alni
=
, prin urmare bisectoarele unghiurilor CAD
AD
BD
BC
PC
AC
=
=
, adica P este si el punct bisector;
ntr-un punct P (CD) nc
at
AD
BD
PD
deducem astfel c
a num
arul punctelor bisectoare este par. Dac
a un tetraedru ar avea
patru puncte bisectoare, fie acestea M (AB), P (CD), N (BC) si Q (AD),
din AC BD = AD BC si AC BD = AB CD urmeaza c
a AD BC = AB CD,
ceea ce arata c
a vom avea puncte bisectoare si pe muchiile AC si BC; astfel, numarul
punctelor bisectoare poate fi foar 0, 2 sau 6.
Evident, tetraedrul regulat are sase puncte bisectoare. Alegand fata ABC cu
laturile de lungimi distincte si varful D astfel nc
at proiectia sa pe planul (ABC) sa fie
centrul cercului circumscris acestei fete, obtinem un tetraedru fara puncte bisectoare.
In sfarsit, un tetraedru cu AB = BC = CA si DA 6= DB = DC 6= BC are exact
doua puncte bisectoare.
c
a

G205. Dou
a panouri luminoase sunt situate n plane paralele (verticale). Ele au
forma a dou
a dreptunghiuri identice, mp
artite fiecare n c
ate zece p
atrate congruente
cu ajutorul c
ate unei linii orizontale si a c
ate patru linii verticale.
In cele 18 v
arfuri
de p
atrate care se formeaz
a pe fiecare panou sunt instalate beculete. La un moment dat
pe fiecare panou se aprind c
ate dou
a beculete, la nt
amplare. Care este probabilitatea
ca cele patru beculete aprinse s
a se afle ntr-un acelasi plan?
Gabriel Popa si Cristian Laz
ar, Iasi
18 17
Solutie. Dou
a beculete dintre cele 18 aflate pe un panou pot fi alese n
=
2
2
153 moduri. Cele patru beculete pot fi alese n 153 moduri.
G
andim o pereche de beculete de pe un panou ca pe un segment cu capetele n
nodurile retelei de p
atrate. Dou
a segmente de pe cele doua panouri sunt coplanare
dac
a si numai dac
a sunt incluse n drepte paralele. Acest lucru se nt
ampla c
and sunt
ambele orizontale, ambele verticale sau ambele oblice, dar formand unghiuri egale cu
orizontala.
65
3 = 45 segmente orizontale, 3 6 = 18
Pe fiecare dreptunghi avem c
ate
2
2
segmente verticale, 5 segmente de o pant
a 2, 14 cu panta 1, 3 cu panta , 10 cu panta
3
2
1
1
1
1
, un segment cu panta , 6 cu panta , 4 cu panta si 2 segmente cu panta .
2
5
3
4
5
Rezult
a c
a num
arul cazurilor favorabile evenimentului urmarit este 452 + 182 + 52 +
142 + 32 + 102 + 1 + 62 + 42 + 22 = 2736.
304
0, 117.
Probabilitatea cerut
a este P =
2601

B. Nivel liceal
L196. Demonstrati c
a n orice triunghi ascutitunghic are loc inegalitatea
(ctg A + ctg B + ctg C)3
8 (1 + cos A cos B cos C)3

.
(ctg A + ctg B)(ctg B + ctg C)(ctg C + ctg A)
sin A sin B sin C
3 3
Gheorghe Costovici, Iasi
184

Solutie. Cum ctg A + ctg B =

sin(A + B)
sin C
=
, avem c
a
sin A sin B
sin A sin B

(ctg A + ctg B + ctg C)3


sin A sin B sin C
=

(ctg A + ctg B)(ctg B + ctg C)(ctg C + ctg A) (1 + cos A cos B cos C)3
(ctg A + ctg B + ctg C)3 (sin A sin B sin C)3
=
=
sin A sin B sin C (1 + cos A cos B cos C)3
(cos A sin B sin C + cos B sin C sin A + cos C sin A sin B)3
=
.
sin A sin B sin C(1 + cos A cos B cos C)3

E=

Intrucat cos C = cos(A + B) = cos A cos B sin A sin B, rezulta c


a
cos A sin B sin C + cos B sin C sin A + cos C sin A sin B =
= sin C sin(A + B) + cos C(cos C + cos A cos B) =
= sin2 C + cos2 C + cos A cos B cos C = 1 + cos A cos B cos C.
Atunci E =

27
1
27
1

A+B+C 3
sin A sin B sin C
(sin A + sin B + sin C)3
)
(3 sin
sin3
3

8
(am aplicat inegalitatea mediilor M G M A si inegalitatea lui Jensen pentru
3 3
h i
functia concava sin : 0,
[0, 1]).
2
Nota autorului. Problema este inspirata de cea a d-lui Marian Tetiva, aparuta
n AMM 2008, pg. 77:
8(tg A + tg B + tg C)3
(1 cos A)(1 cos B)(1 cos C)

.
cos A cos B cos C
27(tg A + tg B)(tg B + tg C)(tg C + tg A)
Asem
anator rezolv
a problema d-l Titu Zvonaru, Com
anesti. O alt
a solutie, primit
a
de la d-nii Mih`
aly Bencze, Brasov si Ioan Viorel Codreanu,
Maramure
s
,
reduce

3 3 2
inegalitatea de demonstrat la una cunoscuta, anume, S
R .
4
L197. Fie ABCDEF GH un paralelipiped dreptunghic, iar S o sfer
a prin A care
intersecteaz
a segmentele AB, AD, AE si AG n M, N, P , respectiv Q. Ar
atati c
a
AM AB + AN AD + AP AE = AQ AG.
Claudiu S
tefan Popa, Iasi
Solutie. Planul dreptunghiului ABCD si sfera S au n comun punctele necoliniare
A, M si N , deci intersecteaz
a S dupa cercul circumscris triunghiului AM N . Dreapta
AC intersecteaz
a cercul circumscris triunghiului AM N , deci si sfera S, ntr-un punct
R (AC] (n caz contrar, am avea c
a AR > AC > diametrul sferei, fals). Conform
cu L166 din RecMat 2/2009, obtinem c
a AM AB + AN AD = AR AC. Planul
dreptunghiului ACGE si S au n comun punctele A, P si R; repetand rationamentul,
deducem c
a AR AC + AP AE = AQ AG si de aici cerinta problemei.
Asa cum, n plan, problema L166 extinde teorema lu Pitagora, problema L197
extinde cunoscuta: p
atratul diagonalei unui paralelipiped dreptunghic este suma
185

p
atratelor dimensiunilor sale (care se obtine c
and paralelipipedul este nscris n sfera).
Probabil c
a o relatie asem
anatoare este valabila n Rn .
Not
a. Solutie corecta au dat si d-nii I.V. Codreanu si T. Zvonaru.
L198. Fie ABC un triunghi ascutitunghic nscris n cercul C. Cercul C este
tangent cercului C n punctul A si laturii BC n punctul D. Ar
atati c
a AD este

bisectoarea unghiului BAC.


Titu Zvonaru, Com
anesti
Solutie. Dac
a triunghiul ABC este isoscel cu b = c, concluzia este imediat
a.
Presupunem c
a b > c si fie T intersectia cu BC a tanA
gentei n A la cercul C. Cu puterea punctului T fata de
C, obtinem c
a T A2 = T B T C T A2 = T B(T B + a).
Aplic
and teorema cosinusului n T AB, deducem c
a
2
2
2

T A = T B + AB 2T B AB cos T BA si atunci T
C
B
D
T B 2 + a T B = T B 2 + c2 + 2c T B cos B, de unde
c2 + a2 2ac cos B
c2
, iar T B+a =
=
TB =
a 2c cos B
a 2c cos B
2
b
. Deoarece T A si T D sunt tangente cercului C , avem c
a T A = T D,
a 2c cos B
prin urmare

bc c2
=
a 2c cos B
ac
c(b c)a
ac(b c)
ab
=
= 2
= 2
CD =
a 2ac cos B
b c2
b+c
b+c

BD = T D T B =

T B(T B + a) T B =

si de aici concluzia rezulta imediat.


Not
a. Profesorul N. Roman semnaleaz
a c
a problema apare n lucrarea Transform
ari geometrice de D. Smaranda si N. Soare, la p. 104, cu numarul 67 (solutia de
la p. 216 foloseste omotetia).
A
L199. Fie O centrul cercului circumscris triunghiului ABC
si {M } = OB AC, {N } = OC AB. Dac
a OM = ON, ar
atati
c
a triunghiul este isoscel sau dreptunghic.
N
M
O
Temistocle Brsan, Iasi
Solutie (Gh. Iurea si T. Zvonaru). Se constata
C
B
usor c
a nici B, nici C nu poate fi obtuz sau drept. Intrucat

M OC N OB (L.U.L.), obtinem c
aM
BN M
CN . Dac
a

O Int ABC, atunci C = B, adica ABC este isoscel; daca O Ext ABC,
2
2

deducem c
a + C = + B, adica ABC este isoscel; n sfarsit, daca O (BC),
2
2
rezulta c
a ABC este dreptunghic si solutia este complet
a.
Not
a. Redactia regreta includerea acestei probleme n sectiunea pentru pregatirea
concursurilor.

3
,1 ,
L200.
In raport cu un reper cartezian xOy, se consider
a punctele M
2

3a 2
2a + 3
2 3
, unde a R\ 0, , , precum si familia de drepte
B
, 0 si C 0,
a
a
3 2
186

2 3m
, m R. Not
am cu Ca cercul circumscris triunghiului OBC.
2
a) Demonstrati c
a, pentru orice m R, dreapta dm intersecteaz
a Ca n dou
a
puncte distincte P si Q.
b) Ar
atati c
a produsul M P M Q este independent de a si m.
Gabriel Popa si Paul Georgescu, Iasi
13(a2 + 1)
2
Solutie. a) Observam c
a BC =
, deci raza cercului Ca este r =
a2

13(a2 + 1)
1
BC =
. Centrul cercului este mijlocul S al ipotenuzei BC, de coor2
2|a|

3a 2 2a + 3
13
2
2
, iar SM = (xS xM ) + (yS yM ) =
,
. Evident
donate S
2a
2a
2|a|
c
a SM < r, prin urmare M IntCa . Pe de alt
a parte, punctul M apartine oricarei
drepte dm din familia considerat
a si de aici cerinta problemei.
b) Faptul c
a M P M Q nu depinde de m rezulta din puterea punctului M fata de
Ca . Pentru a ar
ata c
a produsul nu depinde de a, vom demonstra c
a toate cercurile
Ca au aceeasi ax
a radical
a, iar M se afla pe aceasta ax
a radical
a. Observam c
a
2
13(a
+
1)
13
13
= r2 , prin urmare OM SM. Fie A(3, 2)
+ 2 =
OM 2 + SM 2 =
4
4a
4a2

2 3
simetricul punctului O fat
a de M ; deducem c
a A Ca , a R\ 0, , , deci axa
3 2
radical
a a tuturor cercurilor Ca este OA.
Not
a. Au rezolvat problema si d-nii I.V. Codreanu si T. Zvonaru.
dm : y = mx +

L201. Demonstrati c
a pentru orice num
ar prim p > 22k + 1, k N , num
arul

2k+1

p2

1 se divide cu 22(k+1)

m
Q

j=1

qj , unde {qj |j = 1, m} este multimea numerelor

prime din multimea {2i + 1|i = 1, 2k}.

Neculai Roman, Mircesti (Iasi)


Solutie. Avem c
ap
1 = (p + 1)(p
1) . . . (p2 + 1)(p + 1)(p 1).
Cum p este evident impar, fiecare factor din descompunerea precedent
a se divide cu
2k+1
2, deci p2
1 se divide cu 22(k+1) .
Pe de alt
a parte, pab 1 se divide cu pa 1 si cu pb 1. Cum 22k+1 = 2n 22k+1n ,
2k+1
n
2k+1n
a p2
1 se divide cu p2 1 si cu p2
1, pentru
cu n = 1, k, rezulta c
n = 1, k. Folosind mica teorema a lui Fermat, daca qj este un numar prim de forma
2k+1
2i + 1, atunci pqj 1 se divide cu qj si deci p2
1 se va divide cu qj . Cum numerele
22(k+1) , q1 , q2 , . . . , qm sunt relativ prime doua c
ate doua, rezulta concluzia problemei.
Not
a. Pentru k = 2 obtinem problema 8258 din G.M.-B. 6/1968. Pentru k = 4
g
asim c
a pentru orice num
ar prim p > 28 + 1 = 257, numarul p512 1 se divide cu
10
2 2 5 17 257.

n+2
n+2
L202. Determinati numerele reale a si b pentru care a
n
+
1

n+1
, n N .
n1b
n
Gheorghe Iurea, Iasi
22k+1

22k

187

22k1

(n + 1)( n + 2 n + 1)

Solutie. Ipoteza problemei este echivalent


a cu a
=
n+2

n( n + 2 n + 1)

= yn , n N . Se arata c
a sirurile (xn )n1
xn , n N si b
n+1
si (yn )n1 sunt strict cresc
atoare si atunci a x1 , iar b lim yn , prin urmare
n

1
2(3 6)
si b .
a
3
2
L203. Fie f, g polinoame cu coeficientii reali, nu ambele constante, iar P =
f + ig C[X]. Presupunem c
a r
ad
acinile lui P sunt numere complexe cu p
artile
imaginare strict negative. Dac
a , R, 2 + 2 6= 0, ar
atati c
a r
ad
acinile polinomului Q = f + g sunt reale.
Adrian Reisner, Paris
Solutie. Fie zk , k = 1, n, radacinile polinomului P , deci P = a

n
Q

k=1

(X zk ).

Dac
a z este un num
ar complex cu partea imaginara strict pozitiv
a, este imediat c
a
|z zk | > |z zk |, deci

n
Q

k=1

|z zk | >

m
Q

k=1

|z zk |, adica |P (z)| > |P (z)|. Dac


a

z este num
ar complex cu partea imaginara strict negativ
a, atunci |P (z)| < |P (z)|.
Conchidem c
a |P (z)| = |P (z)| daca si numai daca z R.
Fie z o radacina a lui Q; vom arata c
a |P (z)| = |P (z)|, de unde concluzia. Avem
|P (z)|2 |P (z)|2 = (f (z) ig(z))(f (z) ig(z)) (f (z) + ig(z))(f (z) ig(z)) =
2i(g(z)f (z) f (z)g(z)) = 4 Im (f (z)g(z)). Cum f (z) + g(z) = 0, unde , R,
2 + 2 6= 0, rezulta c
a imaginile geometrice ale lui f (z) si g(z) sunt coliniare cu
originea, deci f (z)g(z) R si astfel |P (z)| = |P (z)|, ceea ce ncheie rezolvarea.

L204. Fie A o matrice p


atratic
a de ordinul n av
and elementele aij din multimea
{0, 1} si urm
atoarele propriet
ati: i) aii = 0 pentru orice i {1, 2, . . . , n}; ii) dac
a
aij = 1 (pentru i 6= j din multimea {1, 2, . . . , n}), atunci aji = 0; iii) pentru fiecare
p {0, 1, . . . , n 1}, matricea are o linie pe care se afl
a exact p elemente egale cu 1.
S
a se arate c
a exist
a o permutare a multimii {1, 2, . . . , n} astfel nc
at, dac
a se
aplic
a aceast
a permutare liniilor matricei A si apoi coloanelor matricei astfel obtinute,
rezult
a n final o matrice cu toate elementele care sunt egale cu 1 situate deasupra
diagonalei principale. Care este polinomul caracteristic al unei asemenea matrice?
Marian Tetiva, B
arlad
Solutie. Fie ik linia matricei A pe care se g
asesc exact n k elemente egale cu
1 (pentru fiecare 1 k n) si permutarea definita prin (k) = ik , oricare ar fi
k {1, . . . , n}. Spunem c
a aplic
am liniilor matricei A permutarea daca formam o
noua matrice B obtinuta din A mutandu-i linia i1 pe prima pozitie, linia i2 pe pozitia
a doua si asa mai departe. Spunem asa deoarece, de fapt, B = P A, unde P este
matricea (numit
a de permutare) corespunz
atoare permutarii , adica matricea care
are toate elementele 0, cu exceptia elementelor din pozitiile (k, ik ), egale cu 1. Dac
a
o matrice se nmulteste cu P 1 (care este matricea de permutare corespunz
atoare
inversei lui la dreapta, precum si transpusa matricei P ) rezultatul este matricea
initiala asupra coloanelor c
areia s-a efectuat aceeasi permutare .
Facem acest lucru, adica formam din A matricea B obtinuta din permutarea liniilor
188

lui A conform permutarii . Desigur, B are n 1 elemente egale cu 1 pe prima linie,


n 2 elemente egale cu 1 pe linia a doua si tot asa (are toate elementele nule pe
ultima linie).
Fie jk num
arul elementelor egale cu 1 de pe coloana ik , pentru fiecare k, 1 k n.
Avem jk n 1 (n k) = k 1 pentru orice 1 k n (conform propriet
atilor
i) si ii)) si j1 + . . . + jn = n(n 1)/2 (conform propriet
atii iii) si faptului c
a numarul
total de elemente egale cu 1 de pe coloane este egal cu numarul elementelor egale
cu 1 num
arate pe linii). Din acestea doua obtinem c
a, de fapt, jk = k 1 pentru
orice 1 k n. Aplic
am coloanelor matricei B tot permutarea si obtinem astfel
matricea C = BP 1 care are pe linia i exact n i elemente egale cu 1 pentru fiecare
1 i n si are pe coloana j exact j 1 elemente egale cu 1, pentru fiecare 1 j n.
Mai mult, cele j 1 elemente de pe coloana j trebuie sa fie primele (de sus n
jos), altfel nu s-ar respecta num
arul elementelor de pe linii. Astfel, C este matricea
c
autata, obtinuta din A prin aplicarea permutarii asupra liniilor sale si apoi prin
aplicarea permutarii asupra coloanelor matricei B (care rezulta dupa permutarea
liniilor).
Cum C = P AP 1 , C si A au acelasi polinom caracteristic; dar C este (strict)
superior triunghiular
a, deci polinomul este X n - adica si polinomul caracteristic al lui
n
A este tot X .
Nota autorului. Aceasta problem
a reprezint
a o variant
a a problemei 11487 din
The American Mathematical Monthly 2/2010.
L205. Calculati

X
(1)k1 ln k
k=1

Marian Tetiva, B
arlad
Solutie. Seria este convergent
a conform criteriuluilui Leibniz.
Avem:

2n ln k
n ln 2k
2n
2n (1)k1 ln k
n 1
2n
P
P
P
P
P
P
ln k
ln k
=
2
=

ln 2 = =
k
k
2k
k
k
k=1
k=1
k=1
k=n+1
k=1
k=n+1 k


n 1
P
ln2 2n ln2 n
ln2 2

ln n ln 2+
. Dupa cum se stie, constanta lui Euler este
2
2
k=1 k
n 1
P
definita prin = lim (
ln n). Exact asa cum se justific
a existenta si finitudinea
n k=1 k
n ln k
P
ln2 n
acestei limite, putem ar
ata c
a sirul xn =

, n 1, este convergent.
2
k=1 k
ln x
si teorema lui Lagrange pentru a obtine ine(Folosim monotonia functiei x 7
x
2
2
ln(n + 1)
ln (n + 1) ln n
ln n
galit
atile
<
<
, n 3, iar aceste inegalitati folosn+1
2
n
esc pentru demonstrarea convergentei sirului mentionat.) Din aceasta convergenta
2n (1)k1 ln k
2n
P
P
ln k
ln2 2n ln2 n

) = 0 si atunci lim
=
rezulta c
a lim (
n k=1
n k=n+1 k
2
k
ln2 2
ln 2. Evident, aceasta va fi si suma seriei din enunt.
2

189

Probleme propuse1
Clasele primare
P.216. Fie numerele a = 1 + si b = 9 . Inlocuiti cercul si p
atratul cu cifre
corespunz
atoare astfel nc
at a + b = 15.
(Clasa I )
Amalia Munteanu, elev
a, Iasi
P.217. O elev
a a desenat un trenulet cu 23 vagoane pe care le-a colorat folosind,
pe rand, culorile rosu, galben, albastru, rosu, galben s.a.m.d. Ce culoare a folosit
pentru vagonul din mijloc?
(Clasa I )
Mihaela Glc
a, elev
a, Iasi
P.218. Mihaela are 14 ani. Ea s-a n
ascut c
and sora sa avea 7 ani, dar cu 5 ani
naintea fratelui sau. C
ati ani au mpreuna cei trei frati?
(Clasa a II-a)
Maria Racu, Iasi
P.219. Un cioban p
azea c
ateva oi si c
ateva capre, n total 24 picioare. Cate oi si
c
ate capre sunt, dac
a oile sunt mai multe decat caprele?
(Clasa a II-a)
Andreea Bzdg
a, elev
a, Iasi
P.220. Suma a doua numere este 2011. Dac
a stergem cifra miilor unuia dintre
ele, obtinem cel
alalt num
ar. Aflati cele doua numere.
(Clasa a III-a)
Mihaela Cianga, Iasi
P.221. Suma dintre un numar si succesorul sau este cu 10 mai mare decat predecesorul sau. Calculati produsul dintre numar si vecinii sai.
(Clasa a III-a)
Petru Miron, Pascani
P.222. Pe o farfurie sunt cirese si visine. Un copil mananca o treime din cirese si
o jum
atate din visine si constata c
a are 17 samburi. Pot ram
ane pe farfurie 20 fructe?
dar 34?
(Clasa a III-a)
Tatiana Ignat, elev
a, Iasi
P.223. Se consider
a numerele naturale x, 4x, 2x + 3, x + 2 si 3x + 2, unde x > 2.
a) Ordonati cresc
ator numerele.
b) Dac
a not
am cu m cel mai mic numar si cu M pe cel mai mare, care trebuie sa
fie valoarea lui x pentru ca sirul m, m + 1, . . . , M sa contina 130 numere?
(Clasa a IV-a)
Mariana Nastasia, elev
a, Iasi
P.224. Un elev si tine banii n doua buzunare. Dac
a ar cheltui un sfert din suma
din primul buzunar si o doime din cea din al doilea, suma total
a s-ar micsora cu 48
lei. Care ar fi suma total
a daca s-ar dubla suma din al doilea buzunar?
(Clasa a IV-a)
Petru Asaftei, Iasi
P.225. Aflati numerele de trei cifre distincte abc, daca abc + bca + cab = 666.
(Clasa a IV-a)
Nicolae Iv
aschescu, Craiova

1 Se

primesc solutii p
an
a la data de 31 decembrie 2011

190

Clasa a V-a
V.137. Se consider
a numerele naturale A = 12010 + 22010 + . . . + 92010 si B =
2011
2011
1
+2
+ ...+ 9
. Demonstrati c
a B A se divide cu 10.
Mariana M
arculescu si Dumitru Cotoi, Craiova
2011

V.138. G
asiti un multiplu al lui 13 a c
arui scriere n baza 10 contine doar cifre
de 1.
Nicolae Iv
aschescu, Craiova
V.139. Scrieti num
arul 17689 ca diferenta de doua p
atrate perfecte.
Liviu Smarandache, Craiova
V.140. Determinati numerele de forma 5abc care, mpatite la abc5, dau c
atul de
595 ori mai mic dec
at restul.
Petru Asaftei, Iasi
V.141. Se consider
a numerele naturale a1 , a2 , . . . , an astfel nc
at a1 = 1 si fiecare
numar, ncepand cu al doilea, este triplul sumei tuturor numerelor dinaintea lui. Dac
a
a1 + a2 + . . . + an = 220 , determinati n.
Mirela Marin, Iasi
. . 11} 22
. . 33} 44
. . 11} 0
. . 22} + 33
. . 44} 11
V.142. Ar
atati c
a num
arul A = 11
| .{z
| .{z
| .{z
| .{z
| .{z
2011

este p
atrat perfect.

2011

2011

2011

2011

Andrei Nedelcu, Iasi

V.143. Reconstituiti nmultirea


alaturat
a, stiind c
a literele distincte reprezint
a cifre distincte.
a
*

Clasa a VI-a
VI.137. Dac
a fractia
lui n.

b
b
*

c
c
c
*

d
d
d
d
*

e
e
e
e
e
*

a
*
f
f
f
f
f
f
*

b
*
a
a
a
a
a

c
*
b
b
b
b

d
*
c
c
c

e
*
d
d

f
*
e

* * * * *
C
at
alin Calistru, Iasi

3n + 7
, n N, este reductibil
a, determinati ultima cifra a
2n + 3
Eugeniu Bl
ajut, Bac
au

VI.138. Determinati numerele prime abc cu proprietatea c


a dintre cele cinci
numere (nu obligatoriu distincte) care se obtin prin permutarea cifrelor, exista macar
doua p
atrate perfecte.
Nicolae Iv
aschescu, Craiova
VI.139. Demonstrati c
a 287|(815 + 85 + 5)(42117 + 4239 + 39).
Bogdan Victor Grigoriu, F
alticeni
191

VI.140. Determinati numerele ntregi nenule n1 < n2 < . . . < n7 , daca 2n1 +
985
2n2 + . . . + 2n7 =
.
1024
Mihai Haivas, Iasi
VI.141. Demonstrati c
a ecuatia 12x + 15y + 20z = 73 nu are solutii (x, y, z) cu
toate componentele numere naturale, dar are o infinitate de solutii cu toate componentele numere ntregi.
Gheorghe Iurea, Iasi
VI.142. Fie ABC un triunghi isoscel cu AB = AC. Dac
a D este simetricul lui B
fat
a de C si mediana din B taie AD n E, aratati c
a CE este mediatoarea segmentului
BD.
Silviu Boga, Iasi
= 45 si m(CBA)
= 30 , iar
VI.143. Se consider
a triunghiul ABC cu m(ACB)
M este un punct pe segmentul AB. Aratati c
a M este mijlocul lui AB daca si numai

CB) = 15 .
dac
a m(M
Andrei Pasa, elev, si Narcisa Pasa, Iasi

Clasa a VII-a
VII.137. Dac
a x, y R, aratati c
a (x2 + x + 1)(y 2 + y + 1) x + y + 1.
Gheorghita St
anic
a si Iulian St
anic
a, Apele Vii (Dolj)
VII.138. Dac
a a, b, c sunt numere ntregi distincte, aratati c
a a2 + b2 + c2 ab +
ac + bc 1. C
and se realizeaz
a egalitatea?
Elena Iurea, Iasi

aa
N.
VII.139. Determinati cifrele a, b si c, daca
b, b(bc)
Romanta Ghit
a si Ioan Ghit
a, Blaj
VII.140. Determinati toate perechile (x, y) de numere ntregi cu proprietatea c
a
2
2
2x+y (2x +y + 1) = 1.
Neculai Stanciu, Buz
au
VII.141. Dac
a ABCD este un patrulater convex, aratati c
a exista un unic punct
M (BD) astfel nc
at triunghiurile ABM si CDM sa fie echivalente.
Cecilia Deaconescu, Pitesti
VII.142. Determinati valoarea minima a ariei unui paralelogram circumscris unui
cerc de raz
a r.
Adrian Corduneanu, Iasi

b = 60 se conVII.143. In interiorul triunghiului ascutitunghic ABC cu m(A)

C) = 150 . Un cerc ce trece prin A si M taie


sider
a un punct M astfel nc
at m(BM
(AB) n Q si (AC) n R, iar cercul circumscris triunghiului M QB taie (BC) n P .
Demonstrati c
a triunghiul P QR este dreptunghic.
Neculai Roman, Mircesti (Iasi)

192

Clasa a VIII-a
VIII.137. Fie V ABCD piramid
a patruletar
a regulata, M si N mijloacele muchiilor V A respectiv V D, iar P punctul n care naltimea V O a piramidei nteapa planul
(M BC). Ar
atati c
a V O = 3 OP.
Adrian Corduneanu, Iasi
VIII.138. Rezolvati n R ecuatia 25 {x}2 10x + 1 = 0.
Bogdan Chiriac, student, Iasi
VIII.139. Numerele naturale a1 , a2 , . . . , a100 au proprietatea c
a N = 6 a1 + 6 a2 +
a100
este p
atrat perfect. Ar
atati c
a numarul a1 + a2 + . . . + a100 se divide cu 5.
...+6
Andrei Eckstein, Timisoara
VIII.140. Fie n N si x1 , x2 , . . . , xn Z\{n} astfel nc
at n3 +x21 +x22 +. . .+x2n
n[1 + 2(x1 + x2 + . . . + xn )]. Demonstrati c
a x1 , x2 , . . . , xn N.
Dan Nedeianu, Drobeta Tr. Severin
b
a
+
+
VIII.141. Dac
a a, b, c, x, y, z > 0 si ax + by + cz = 1, demonstrati c
a
yz xz
c
27abc.
xy
D.M. B
atinetu-Giurgiu, Bucuresti
VIII.142. Fie a, b, c, d R si E(x, y) =
x, y R. Dac
a |ad| = |bc|, demonstrati c
a

E(x, z) F (x, z)

a2 x2 + b2 y 2 , F (x, y) =

E(x, y) F (x, y) +

c2 x2 + d2 y 2 ,

E(y, z) F (y, z), x, y, z R.

Valentina Blendea si Gheorghe Blendea, Iasi


a2011
b2011
c2011
VIII.143. Dac
a 2
=
=
, demonstrati c
a numerele reale
a + b2
b 2 + c2
c2 + a2
pozitive a, b si c sunt egale.
Cristina Ene, elev
a, Craiova

Clasa a IX-a


2

1 + sin8 x + cos8 x
, x, y R.
1 + sin8 y + cos8 y
Mih
aly Bencze, Brasov

a
1+ 5
IX.122. Fie a, b, c R cu b c > 0 si
. Dac
a numerele a, b, c pot fi
b
2
1 1 1
laturile unui triunghi, demonstrati c
a si , , pot fi laturi ale unui triunghi.
a b c
Ovidiu Pop, Satu Mare
IX.121. Ar
atati c
a

1 + sin4 x + cos4 x
1 + sin4 y + cos4 y

IX.123. Consider
am patrulaterul ABCD si fie M, N, P, Q mijloacele laturilor
AB, BC, CD respectiv DA, iar T un punct interior patrulaterului. Not
am cu G1 , G2 ,
G3 si G4 centrele de greutate ale patrulaterelor T N CP, T P DQ, T QAM , T M BN .

Aratati c
a AG1 + BG2 + CG3 + DG4 = 0 daca si numai daca {T } = M P N Q.
Florin St
anescu, G
aesti
193

IX.124. Dac
a ABCD este un patrulater inscriptibil, aratati c
a BC 2 SACD +
2
2
CD SABC = AC SBCD .
D.M. B
atinetu-Giurgiu, Bucuresti
IX.125. Dac
a x, y R, x > y > 1, aratati c
a xy + 4 > x + 3y.
Dan Nedeianu, Drobeta Tr. Severin

Clasa a X-a

X.121. Dac
a a R+ , rezolvati n R ecuatia (2a)x ax = 2.
Luminita Mihalache, Craiova
ha
+
X.122. Demonstrati c
a triunghiul ABC este echilateral daca si numai daca
ac
hc
ha
hb
hc
hb
+
=
+
+
(notatiile sunt cele uzuale).
ab
bc
bc
ac ab
Petru Asaftei, Iasi

X.123. Dac
a n N,
n 3 si x (1, 1), demonstrati inegalitatea n( n 1 + x +

n
1 x) 2( 1 + x + 1 x + n 2).
Lucian Tutescu si Petrisor Rocsoreanu, Craiova
X.124. Aflati numerele complexe nenule x, y, z cu proprietatea c
a
x(x + y)(x + z) = y(y + x)(y + z) = z(z + x)(z + y) = 1.
X.125. Dac
a x, y N sunt astfel nc
at numarul
este rational, ar
atati c
a x = y.

Vasile Chiriac, Bac


au

x2

+ 2y + 1+ 3 y 3 + 3x2 + 3x + 1
Gheorghe Iurea, Iasi

Clasa a XI-a



1


A
XI.121. Dat triunghiul ABC, aratati c
a sin
2

C

sin

XI.122.
XI.123.
Determinati
XI.124.
XI.125.
negativ
a.

C

2
B 1
1
sin .
2 2
B

sin
1
2
2
Bogdan Victor Grigoriu, F
alticeni
2
2
Fie A, B Mn (Q) cu A + B = 2In ; aratati c
a det(AB + BA) 0.
Dumitru Cr
aciun, F
alticeni
Fie A, B M2 (R) astfel nc
at detA = 10, detB = 12, tr A = tr B = 7.
numerele naturale n pentru care tr An = tr B n .
R
azvan Ceuc
a, elev, Iasi

Calculati lim
220 + 221 + 222 + . . . + 22n + 1.
n
Gheorghe Iurea, Iasi
x
x
x
x
Demonstrati c
a ecuatia 25 + 4 = 10 + 9 are cel putin o solutie reala
sin

A
2

sin

Ionut Iv
anescu, Craiova
194

Clasa a XII-a
XII.121. Fie a N si G = (a, +) pe care definim operatia x y = (x a)(y
a) + a, x, y G. Dac
a H este subgrup al lui G astfel nc
at N G H, aratati c
a
Q G H.
D.M. B
atinetu-Giurgiu, Bucuresti si Neculai Stanciu, Buz
au
XII.122. Fie n N, n 2 si polinomul f = X n 2nX n1 + (2n2 4)X n2 +
a3 X n3 + . . . + an C[X]. Demonstrati c
a f are toate radacinile reale daca si numai
daca n = 2.
Florin St
anescu, G
aesti
Z

arccos

XII.123. Calculati I =

65
9

tg x sin xdx.

Vasile Chiriac, Bac


au

XII.124. Fie f : R R o functie continu


a cu proprietatea c
a (f f )(x) = sin x,
Z

x R. Demonstrati c
a

f (x)dx 1.

Dumitru Cr
aciun, F
alticeni

XII.125. Fie f : [0, 1] R o functie derivabila cu f integrabila. Dac


a f (0) = 0,
Z

aratati c
a
0

(f (t)) dt

f 2 (t)dt.

Ciprian Baghiu, Iasi

Probleme pentru preg


atirea concursurilor
A. Nivel gimnazial
G206. C
ate submultimi ale multimii M = {1, 2, 3, . . . , 100} au 50 de elemente si
nu contin nicio pereche de numere consecutive?
Gheorghe Iurea, Iasi
G207. Ar
atati c
a num
arul N = 22009 + 32010 + 42011 nu este p
atrat perfect.
Andrei Eckstein, Timisoara
G208. Demonstrati c
a ecuatia x2 + y 2 = z(x + y + 1) are o infinitate de solutii n
multimea numerelor naturale.
Cosmin Manea si Dragos Petric
a, Pitesti
G209. Rezolvati n numere naturale ecuatia 4abc = (a + 2)(b + 1)(c + 1).
Titu Zvonaru, Com
anesti
a3n+2 a3n+1 + (1)n
este reductibil
a pentru
G210. Demonstrati c
a fractia 3n+8
a
a3n+7 + (1)n
orice a, n N, a 2.
Dan Popescu, Suceava
195

G211. Demonstrati c
a expresia

x2 (a1 + a2 ) + x3 a1
x1 (a1 + a2 ) + x3 a2
+ y2
x1 + x2 + x3
x1 + x2 + x3

2
x1 a1 x2 a2
1

+ y3

x1 + x2 + x3
y1 + y2 + y3

E = y1

x1 (a1 + a2 ) + x3 a2
x1 a1 x2 a2
x2 (a1 + a2 ) + x3 a1
+ y2
+ y3
y1
x1 + x2 + x3
x1 + x2 + x3
x1 + x2 + x3
unde ai , xi , yi R+ (i = 1, 2, 3), nu depinde x1 , x2 , x3 .

Mircea Brsan, Iasi

= 135 si m(C)
= 30 . Determinati
G212. Se consider
a triunghiul ABC cu m(B)

masurile unghiurilor triunghiului ABD, unde D este simetricul lui C fata de B.


Eugeniu Bl
ajut, Bac
au
G213. Se consider
a triunghiul ABC cu proprietatea c
a exista M si N puncte
n interiorul sau astfel nc
at BN = CM si ABM ACN . Demonstrati c
a
AB = AC.
Crisitan Laz
ar, Iasi
b
G214. Se consider
a triunghiul isoscel ABC cu AB = AC si m(A) < 90 . Construim naltimea CF si fie E mijlocul segmentului BF , iar D un punct pe segmentul
B,
ar
BC. Dac
a ADE
atati c
a D este mijlocul segmentului BC.
Claudiu S
tefan Popa si Gabriel Popa, Iasi
G215. In planele paralele P1 si P2 se consider
a cercurile C1 = C(O1 , R1 ), respectiv
C2 = C(O2 , R2 ). Fie K1 conul de varf O2 si baza C1 si K2 conul de varf O1 si baza C2 .
Ar
atati c
a intersectia celor doua conuri este un cerc si determinati pozitia centrului
si marimea razei acestuia.
Temistocle Brsan, Iasi

B. Nivel liceal
L206. Fie P un punct pe mediana din A a triunghiului ABC. Paralela prin P
la AC taie AB n M , iar simetricul lui P fata de mijlocul lui AC este N . Aratati c
a
M N kBC dac
a si numai daca P este centrul de greutate al triunghiului ABC.
Silviu Boga, Iasi
L207. Fie ABCD un patrulater convex si M, N, P puncte pe segmentele AB, CD
MB
ND
BP
respectiv BC astfel nc
at
=
=
= k. Dac
a R si S sunt mijloacele
AB
DC
BC
RS
.
segmentelor AP respectiv M N , calculati (n functie de k) raportul
AD
Titu Zvonaru, Com
anesti
L208. Un cilindru circular drept de ax
a d si raz
a R1 si o sfera de centru O si raz
a
R2 sunt tangente exterior n punctul A. Fie B simetricul lui A n raport cu d si fie
planul ce trece prin B, este perpendicular pe planul determinat de O si d si face
cu axa d un unghi de 30 . Calculati raportul razelor celor doua suprafete stiind c
a
sectiunile lor cu planul au arii egale.
Temistocle Brsan, Iasi
196

L209. Se consider
a triunghiul ABC si punctele M, N, P, Q, R, S definite prin

BM = k M C, CN = k N A, AP = k P B, AM = p M Q, BN = p N R,

1
CP = p P S, unde k, p R \{1}. Demonstrati c
a SMN P SABC , iar SQRS
4

p+3 2
SABC .
2p
Marius Olteanu, Rm. V
alcea
L210. Cercul A-exnscris triunghiului ABC este tangent prelungirilor laturilor
AB si AC n P , respectiv Q. Bisectoarele exterioare ale unghiurilor B si C intersecteaza dreapta P Q n S respectiv T . Demonstrati c
a P Q ST + BC.
Titu Zvonaru, Com
anesti

3
3
3 3
cos x
sin x

, x R.
+
L211. Ar
atati c
a
2
2
2
2
(1 + cos x)
16
(1 + sin x)
Mih`
aly Bencze, Brasov
P
3 P ab
(ab + c2 )2
L212. Demonstrati c
a +

(sumele fiind ciclice)


2
a2 + b 2
(a2 + c2 )(b2 + c2 )
pentru orice numere reale a, b, c printre care nu se g
asesc doua egale cu 0.
Marian Tetiva, B
arlad
L213. Fie m1 , . . . , mk numere naturale nenule si un numar irational.
[xk ]
[x1 ]
= ... =
.
a) Ar
atati c
a exista x1 , . . . , xk N astfel nc
at
m1
mk
b) Ar
atati c
a exista y1 , . . . , yk N astfel nc
at m1 [y1 ] = . . . = mk [yk ].
Marian Tetiva, B
arlad
L214. Fie A Mn (R) o matrice simetric
a al c
arei polinom caracteristic este X n .
Aratati c
a A este matricea nula.
Marian Tetiva, B
arlad
L215. Avem la dispozitie 2n + 1 pietricele (n 1) astfel nc
at orice submultime
de 2n pietricele poate fi mpartit
a n doua gramezi de c
ate n pietricele avand aceeasi
masa total
a. Demonstrati c
a toate pietricelele au aceeasi masa.
Adrian Reisner, Paris

Training problems for mathematical contests


A. Junior highschool level
G206. How many subsets of the set M = {1, 2, 3, . . . , 100} have 50 elements and
do not contain any pair of successive numbers ?
Gheorghe Iurea, Iasi
G207. Show that the number N = 22009 + 32010 + 42011 is not a perfect square.
Andrei Eckstein, Timisoara
G208. Show that the equation x2 + y 2 = z(x+ y + 1) has infinitely many solutions
in the set of natural numbers.
Cosmin Manea and Dragos Petric
a, Pitesti
197

G209. Solve, in the set of natural numbers, the equation 4 abc = (a + 2)(b +
1)(c + 1).
Titu Zvonaru, Com
anesti
3n+2
3n+1
n
a
a
+ (1)
is reducible for any
G210. Prove that that the fraction 3n+8
a
a3n+7 + (1)n
numbers a, n N, a 2.
Dan Popescu, Suceava
G211. Show that the expression

x2 (a1 + a2 ) + x3 a1
x1 (a1 + a2 ) + x3 a2
+ y2
x1 + x2 + x3
x1 + x2 + x3

1
x1 a1 x2 a2 2
+ y3

x1 + x2 + x3
y1 + y2 + y3

E = y1

y1

x2 (a1 + a2 ) + x3 a1
x1 (a1 + a2 ) + x3 a2
x1 a1 x2 a2
+ y2
+ y3
x1 + x2 + x3
x1 + x2 + x3
x1 + x2 + x3

where ai , xi , yi R+ (i = 1, 2, 3), does not depend of x1 , x2 , x3 .


Mircea Brsan, Iasi

= 30 . Determine
G212. Let ABC be a triangle with m(B) = 135 and m(C)
the measures of the angles of triangle ABD, where D is the symmetric of point C
with respect to point B.
Eugeniu Bl
ajut, Bac
au
G213. Let ABC be a triangle with the property that two points M and N exist
in its interior such that BN = CM and ABM ACN . Show that AB = AC.
Crisitan Laz
ar, Iasi
b <
G214. The isosceles triangle ABC is considered, with AB = AC and m(A)

90 . We build the altitude line CF and let E be the midpoint of the segment BF ,
B,
show that D is the midpoint
while D is a point on the segment BC. If ADE
of segment BC.
Claudiu S
tefan Popa and Gabriel Popa, Iasi
G215. In the parallel planes P1 and P2 the circles C1 = C(O1 , R1 ), respectively
C2 = C(O2 , R2 ) are considered. Let K1 be the cone of vertex O2 and base C1 , and K2
the cone of vertex O1 and base C2 . Show that the intersection of the two cones is
a circle and find the position of its center as well as the length of its radius.
Temistocle Brsan, Iasi

B. Highschool Level
L206. Let P be a point on the median from A of the triangle ABC. The parallel
through P to AC cuts AB at point M , and the symmetric point of P with respect to
the mipoint of AC is N. Show that M N kBC if and only if P is centroid of triangle
ABC.
Silviu Boga, Iasi
L207. Let ABCD be a convex quadrilateral and let M, N, P be points on the
ND
BP
MB
=
=
= k. If R
line segments AB, CD and respectively BC such that
AB
DC
BC
198

and S are the midpoints of segments AP , respectively M N, calculate (as a function


RS
.
of k) the ratio
AD
Titu Zvonaru, Com
anesti
L208. A right circular cone of axis d and radius R1 , and a sphere of center O and
radius R2 , are exterior-tangent at the point A. Let B be the symmetric of A with
respect to d and let be the plane that passes through B, is perpendicular on the
plane determined by O and d and forms an angle of 30 with axis d. Calculate the
ratio between the radii of the two surfaces knowing that their sections through the
plane have equal areas.
Temistocle Brsan, Iasi
L209. It is considered the triangle ABC and the points M, N, P, Q, R, S defined by

BM = k M C, CN = k N A, AP = k P B, AM = p M Q, BN = p N R, CP = p P S,

2
p+3
1
SABC .
where k, p R \{1}. Prove that SMN P SABC , and SQRS
4
2p
Marius Olteanu, Rm. V
alcea
L210. The A-excircle to the triangle ABC is tangent to the prolongations of the
sides AB and AC at P , respectively Q. The exterior angle bisectors of B and C
intersect the line P Q at S, respectively T . Prove that P Q ST + BC.
Titu Zvonaru, Com
anesti

3
3
sin x
3 3
cos x
L211. Show that

, x R.
+
2
2
2
2
(1 + cos x)
16
(1 + sin x)
Bencze Mih`
aly, Brasov
2
2
P
(ab + c )
3 P ab

(the sums being cyclic)


L212. Show that +
2
a2 + b 2
(a2 + c2 )(b2 + c2 )
for any real numbers a, b, c with no two numbers equal to 0 among them.
Marian Tetiva, B
arlad
L213. Let m1 , . . . , mk be nonzero natural numbers and an irrational number.
[xk ]
[x1 ]
= ... =
.
a) Show that x1 , . . . , xk N exist such that
m1
mk

b) Show that y1 , . . . , yk N exist such that m1 [y1 ] = . . . = mk [yk ].


Marian Tetiva, B
arlad
L214. Let A Mn (R) be a symmetric matrix whose characteristic polynomial is
X n . Show that A is the null matrix.
Marian Tetiva, B
arlad
L215. We have at our disposal 2n + 1 small stones (n 1) such that any subset
of small stones can be divided into two heaps of n small stones each and having the
same total sum. Prove that all the small stones have the same mass.
Adrian Reisner, Paris

199

Pagina rezolvitorilor

CAMPULUNG
MUSCEL
Liceul National cu program de atletism. Clasa a XI-a (prof. POPESCU Ionela).
GRUBINSCHI Anisoara-Marilena: IX(116,117), X.117, XI.117, XII.116.
CRAIOVA
Colegiul National Fratii Buzesti. Clasa a IX-a (prof. TUT
ESCU Lucian).
GOLEA Monica: VIII(132, 133), IX(116,117,119), X.117.
IAS
I
S
coala nr. 3 Al. Vlahut
a. Clasa a III-a (inst. MAXIM Gabriela). CARACAS

Sorin: P(206-210,213); DASCALU


Lorena: P(206-210,213); NICA Ioan-Daniel: P(206
210,213); ROBU Carmen: P(206-210,213); SERBANOIU
Alexandru: P(206-210,213).

Clasa a IV-a (nv. MARIUT


A Valentina). LUNGU Lucian: P(208,209,211-214);
Alin: P(208,209,
HERGHILIGIU-HENEA M
alina: P(208,209,211-214); HREAPCA
211-214); HUHU Paula: P(209,211-214); PERES Cristi: P(208,209,211-214); POPO Andreea-Maria: P(208,209,211VICI Teodor-Andrei: P(208,209,211-214); ROMILA
214). Clasa a V-a (prof. MARIN Mirela). CIOCOIU Alexandra: P(214,215), V(130132); MARIN Marius: P(214,215), V(130-132); VLAD Ioana: P(214,215), V(130132). Clasa a VI-a (prof. MARIN Mirela). CRET
U Cristiana-Paula: V(130-132),
A
Bianca: V(130-132), VI(131,134); GLIGA Dumitru: V(130VI(131,134); GAIN
132), VI(131,132). Clasa a VIII-a (prof. MARIN Mirela). ASAVEI Alexandra:
VI(131,132,134), VII.134, VIII.131; CELMARE Raluca-Iuliana: VI(131,132,134),
VII(130,132-134,136), VIII(131,132); TIBA S
tefana-Alexandra: VII(130,133,134,136),
VIII(131,132).
S
coala nr. 11 Otilia Cazimir. Clasa a III-a (nv. MARDARE Carmen).
AIAL

Dumitru). POPA
OLENIUC Iulian: P(206-212). Clasa a IV-a (inst. PAR
A
Ioana-Maria: P(196-215), V(131-134).

S
coala nr. 26 George Cosbuc. Clasa a II-a (inst. RACU Maria). CIOPEICA
Sebastian-Andrei: P(206-210); GROSU Victor-Alessandru: P(206,207,209-211); LU
CHIAN Maria-Clara: P(206-210); PASNICU Cosmin-Constantin: P(206-210); RAILEANU R
azvan-Constantin: P(206-210); VASILE Raluca-Andreea: P(206-210).
Stefan:
Colegiul National Iasi. Clasa a V-a (prof. POPA Gabriel). OBADA
V(124,126-128,131,135,136), VI(123,134). Clasa a VI-a (prof. POPA Gabriel).
BERBINSCHI Tudor: V(123-129,131-136), VI(123-132,134-136),VII(125,126), G(186,
187,191,193,196-198); DOMINTE S
tefan: V(123-129,131-133,135), VI(123-125,128,
134-136), VII.125, G(187,196); PALAGHIU Horatiu: VI(123-136). Clasa a VIII-a
(prof. POPA Gabriel). STOLERU Ingrid: VII(130-136), VIII(124,130,131), G(187,
R
189,190,193,195-198,205). Clasa a XI-a (prof. POPA Gabriel). CEUCA
azvan:
X(111-113,115,117-119), XI(111-114,116-118), XII.111.
PAS
CANI

S
coala Iordachi Cantacuzino. Clasa a III-a (nv. MIRON Petru). CRACIUN
S
tefana-Maria: P(206-212), V.113.
200


T
IGANAS
I (IAS
I)
S
coala cu clasele I-VIII M. Kog
alniceanu. Clasa I (nv. SAMSON Daniel
Mihai). DUCA Daria-C
alina-Stela: P(206-210); Clasa a II-a (prof.nv.primar BADI Aurica). DUCA Ema Stefania: P(206-210). Clasa a IV-a (nv. GALIA ParaschiT
A
va). CAZADOI Ioana-Cristina: P(206-210,213,214); DUCA Mihaela-Cristina: P(206210,213,214); ILIOAE Mihai-Iulian: P(206-210,213,214); PIU Naomi Stefana: P(206210,213,214); SANDU Ana Paula: P(206-210,213,214); SANDU Rebeca: P(206210,213,214).
Au mai trimis redactiei un num
ar mai mic de cinci probleme corect rezolvate
elevii urmatori: (clasa a V-a) Duca Cristina Diana, Gain
a Raluca Mihaela, Gioanca
Bianca Maria, Piu Raluca Diana, Toma Evelina; (clasa a VI-a) Duca Mirela Beatrice,
Ganeanu Tudor, Ghioanc
aS
tefan, Sandu Codrut, Verner Madalina Georgiana; (clasa
a VII-a) Iacob Oana Alisa, Piu Mioara Florentina, Tanase M
adalina; (clasa a VIII-a)
Ghioanc
a Andreea Lavinia, Ghioanc
a Claudia Oana.

Elevi rezolvitori premiati


1. ASAVEI Alexandra (clasa a VIII-a): 1/2010(5pb), 2/2010(5pb), 1/2011(7pb);
2. CARACAS
Sorin (clasa a III-a): 2/2010(5pb), 1/2011(5pb), 2/2011(6pb);
3. CELMARE Raluca-Iuliana (clasa a VIII-a): 1/2010(5pb), 2/2010(5pb),
1/2011(11pb);
Sebastian-Andrei (clasa a II-a): 2/2010(5pb), 1/2011(5pb),
4. CIOPEICA
2/2011(5pb);

5. CRACIUN
S
tefana-Maria (clasa a III-a): 1/2010(10pb), 2/2010(6pb),
1/2011(9pb);
6. CRET
U Cristiana-Paula (clasa a VI-a): 1/2010(5pb), 2/2010(5pb),
1/2011(5pb);
7. GROSU Victor-Alessandru (clasa a II-a): 2/2010(5pb), 1/2011(5pb),
2/2011(5pb);
8. IFTIME Ioana Evelina (clasa a VI-a): 1/2010(6pb), 2/2010(5pb),1/2011(7pb);
9. LUCHIAN Maria-Clara (clasa a II-a): 2/2010(5pb), 1/2011(5pb),
2/2011(5pb);
10. PASNICU Cosmin-Constantin (clasa a II-a): 2/2010(5pb), 1/2011(5pb),
2/2011(5pb);
11. PERES
Cristi (clasa a IV-a): 2/2010(6pb), 1/2011(7pb), 2/2011(6pb);

12. RAILEANU
R
azvan-Constantin (clasa a II-a): 2/2010(5pb), 1/2011(5pb),
2/2011(5pb);
13. SOFIAN Elena Marina (clasa a VI-a): 1/2010(5pb), 2/2010(5pb),
1/2011(5pb).
201

COLECT
IA RECREAT
II MATEMATICE
Asociatia Recreatii Matematice initiaz
a si stimuleaza
a publicarea de c
arti
de matematica elementar
a adresate tuturor iubitorilor acestui domeniu. Conditiile
de publicare sunt specificate n regulamentul colectiei (care poate fi solicitat la adresa
t birsan@yahoo.com).
Primul num
ar al Colectiei Recreatii Matematice, aflat n curs de tip
arire
este

1. D. Br
anzei, Al. Negrescu Probleme de pivotare.

IMPORTANT
In scopul unei leg
aturi rapide cu redactia revistei, pot fi utilizate urmatoarele
adrese e-mail: t birsan@yahoo.com si profgpopa@yahoo.co.uk . Pe
aceasta cale colaboratorii pot purta cu redactia un dialog privitor la materialele trimise acesteia, procurarea numerelor revistei etc. Sugeram colaboratorilor care trimit probleme originale pentru publicare sa le numeroteze
si sa-si retina o copie xerox a lor pentru a putea purta cu usurinta o discutie
prin e-mail asupra accept
arii/neaccept
arii acestora de c
atre redactia revistei.
La problemele de tip L se primesc solutii de la orice iubitor de matematici
elementare (indiferent de preocupare profesional
a sau v
arst
a ). Fiecare dintre
solutiile acestor probleme - ce sunt publicate n revist
a dupa un an - va fi
urmat
a de numele tuturor celor care au rezolvat-o.
Adres
am cu insistent
a rug
amintea ca materialele trimise revistei
s
a nu fie (s
a nu fi fost) trimise si altor publicatii.
Rugam ca materialele tehnoredactate sa fie trimise pe adresa redactiei
nsotite de fisierele lor (de preferinta n LATEX).
Pentru a facilita comunicarea redactiei cu colaboratorii ei, autorii materialelor sunt rugati sa indice adresa e-mail.

202

Revista semestrial RECREAII MATEMATICE este editat de ASOCIAIA


RECREAII MATEMATICE. Apare la datele de 1 martie i 1 septembrie i se
adreseaz elevilor, profesorilor, studenilor i tuturor celor pasionai de matematica
elementar.
n atenia tuturor colaboratorilor
Materialele trimise redaciei spre publicare (note i articole, chestiuni de metodic,
probleme propuse etc.) trebuie prezentate ngrijit, clar i concis; ele trebuie s prezinte
interes pentru un cerc ct mai larg de cititori. Se recomand ca textele s nu depeasc
patru pagini. Evident, ele trebuie s fie originale i s nu fi aprut sau s fi fost
trimise spre publicare altor reviste. Rugm ca materialele tehnoredactate s fie
nsoite de fiierele lor.
Problemele destinate rubricilor: Probleme propuse i Probleme pentru
pregtirea concursurilor vor fi redactate pe foi separate cu enun i demonstraie/rezolvare (cte una pe fiecare foaie) i vor fi nsoite de numele autorului, coala i
localitatea unde lucreaz/nva.
Redacia va decide asupra oportunitii publicrii materialelor primite.
n atenia elevilor
Numele elevilor ce vor trimite redaciei soluii corecte la problemele din rubricile
de Probleme propuse i Probleme pentru pregatirea concursurilor vor fi menionate
n Pagina rezolvitorilor. Elevii menionai de trei ori vor primi o diplom i un
premiu n cri. Elevii rezolvitori vor ine seama de regulile:
1. Pot trimite soluii la minimum cinci probleme propuse n numrul prezent
i cel anterior al revistei (pe o foaie va fi redactat o singur problem).
2. Elevii din clasele VI-XII au dreptul s trimit soluii la problemele propuse
pentru clasa lor, pentru orice clas mai mare, din dou clase mai mici i imediat
anterioare. Cei din clasa a V-a pot trimite soluii la problemele propuse pentru clasele a
IV-a, a V-a i orice clas mai mare, iar elevii claselor I-IV pot trimite soluii la
problemele propuse pentru oricare din clasele primare i orice clas mai mare. Orice
elev poate trimite soluii la problemele de concurs (tip G i L).
3. Vor fi menionate urmtoarele date personale: numele i prenumele, clasa,
coala i localitatea, precum i numele profesorului cu care nva.
4. Plicul cu probleme rezolvate se va trimite prin pot (sau va fi adus direct) la
adresa Redaciei:
Prof. dr. Temistocle Brsan
Str. Aurora, nr. 3, sc. D, ap. 6,
700 474, Iai
Jud. IAI
E-mail: t_birsan@yahoo.com

CUPRINS
Petru MINU (1936-2011) ................................................................................................. 99
Congresul al VII-lea al matematicienilor romni de pretutindeni (C. Corduneanu) ....... 101
Profesorul ION CREANG o sut de ani de la naterea sa (T. Brsan)........................ 103
SPIRU HARET (1851-1912) contribuia tiinific (V. Oproiu) .................................... 105

ARTICOLE I NOTE
T. ZVONARU, N. STANCIU Alte proprieti caracteristice
triunghiului echilateral .................... 108
D.M. BTINEU-GIURGIU, N. STANCIU Inegaliti geometrice
n poligoane convexe, de tip Bergstrm-Mitrinovi ................... 112
D.t. MARINESCU, I. ERDEAN n legtur cu o problem de concurs ...................... 116
L. TUESCU, M. DICU Aplicaii ale inegalitii mediilor
n rezolvarea unor probleme de minim ................... 119

NOTA ELEVULUI
D. DNIL Asupra unei inegaliti aproape clasic ..................................................... 121

CORESPONDENE
A. REISNER Autour du cardinal d'un ensemble de matrices binaires ................................ 124

CUM CONCEPEM ... CUM REZOLVM


N. ROMAN Comentarii pe marginea unei probleme .......................................................... 129

CHESTIUNI COMPLEMENTARE MANUALELOR


I. PTRACU Proprieti ale triunghiurilor n care 3a = b + c ......................................... 132

COLI I DASCLI
M. TETIVA Colegiul Naional Gheorghe Roca Codreanu din Brlad ....................... 134

CONCURSURI I EXAMENE
Concursul de matematic "Al. Myller", ed. a IX-a, 2011 ................................................. 137
Concursul de matematic "Florica T. Cmpan", ed. a XI-a, 2011 .................................... 140

PROBLEME I SOLUII
Soluiile problemelor propuse n nr. 2/2010 ......................................................................... 148
Soluiile problemelor propuse n nr. 1/2011 ......................................................................... 164
Soluiile problemelor pentru pregtirea concursurilor propuse n nr. 1/2011 ....................... 181
Probleme propuse ..................................................................................................................... 190
Probleme pentru pregtirea concursurilor ............................................................................... 195
Training problems for mathematical contests ........................................................................ 197
Pagina rezolvitorilor ................................................................................................................ 200
Elevi rezolvitori premiai ......................................................................................................... 201
ISSN 1582 1765

8 lei

S-ar putea să vă placă și